Você está na página 1de 171

FACULDADE DE MEDICINA DA UNIVERSIDADE DE LISBOA

Curso de Mestrado Integrado em Medicina 2010/2011


1 Semestre 1 Ano

BIOQUMICA
Mdulo II.I

Brigitta Cismasiu

O estudo atravs deste resumo deve ser complementado com os slides disponibilizados pelo
professor Miguel Castanho e com, pelo menos, um dos seguintes livros: Stryer, Devlin,
Grisham, Lehninger ou Lippincott.
Brigitta Cismasiu | FMUL 2010/2011

ndice

Noes Bsicas ...................................................................................... 13


Isomerismo ........................................................................................................ 13
Termodinmica ................................................................................................. 13
Enzimas.............................................................................................................. 14
Classificao ............................................................................................................ 15
Cintica Enzimtica ................................................................................................. 15
Equao de Michaelis-Menten................................................................................ 15
Linearizao de Lineweaver-Burk ........................................................................... 17
Factores que Influenciam a Velocidade da Reaco ............................................... 17
Tipos de Inibio...................................................................................................... 18
Receptorologia .................................................................................................. 19
Receptores .............................................................................................................. 20
Canais Inicos .......................................................................................................... 20
Enzimas.................................................................................................................... 20
Molculas Transportadoras .................................................................................... 21
Agonista ................................................................................................................... 22
Antagonista ............................................................................................................. 22
Receptores de Membrana e Mecanismos de Transduo de Sinais ...................... 22
Receptores Ionotrpicos ou Canais Inicos ............................................................ 23
Receptores Metabotrpicos ou GPCRs (Associados Protena G) ......................... 24
Receptores Associados a Protenas Cinases............................................................ 29
Receptores Intranucleares ...................................................................................... 34
Regulao dos Nveis Intracelulares de Clcio ........................................................ 35
Efeitos Biolgicos da Radiao .......................................................................... 36
Viso ........................................................................................................................ 37
Aminocidos e Protenas ....................................................................... 38
Aminocidos ...................................................................................................... 38
Estereoismeros...................................................................................................... 38
Propriedades Inicas ............................................................................................... 38

3
Brigitta Cismasiu | FMUL 2010/2011

Ponto Isoelctrico ................................................................................................... 39


Ligao Peptdica ..................................................................................................... 39
Protenas ........................................................................................................... 40
Estrutura Primria ................................................................................................... 40
Estrutura Secundria ............................................................................................... 40
Estrutura Terciria ................................................................................................... 41
Estrutura Quaternria ............................................................................................. 41
Estrutura Nativa ...................................................................................................... 41
Desnaturao .......................................................................................................... 41
Colagnio ................................................................................................................. 42
Insulina .................................................................................................................... 42
Correlaes Clnicas .......................................................................................... 43
Anemia Falciforme .................................................................................................. 43
Envenenamento por Insulina .................................................................................. 43
Lpidos, Membranas e Transporte ......................................................... 45
Importncia da Hidrofobicidade dos Triacilgliceris............................................... 45
Tipos mais Comuns de Lpidos ................................................................................ 46
Associaes Fosfolipdicas Possveis em Meio Aquoso........................................... 46
Colesterol ................................................................................................................ 46
Transporte Transmembranar ............................................................................ 48
Potencial Qumico ................................................................................................... 48
Potencial Elctrico ................................................................................................... 48
Transporte Mediado por Protenas ......................................................................... 49
Transporte Atravs da Membrana .......................................................................... 49
Trs Classes de Transporte ...................................................................................... 50
Transporte Activo .................................................................................................... 50
Transporte de Glucose nas Clulas Epiteliais Intestinais ........................................ 50
Glcidos ................................................................................................. 51
Monossacridos ................................................................................................ 51
Ciclizao ................................................................................................................. 52
Ligaes Glicosdicas ............................................................................................... 53

4
Brigitta Cismasiu | FMUL 2010/2011

Oligossacridos.................................................................................................. 53
Dissacridos ............................................................................................................. 53
Polissacridos .................................................................................................... 54
Homopolissacridos ................................................................................................ 54
Heteropolissacridos ............................................................................................... 54
Glicognio ................................................................................................................ 54
Proteoglicanos ......................................................................................................... 55
Glicoprotenas ......................................................................................................... 56
Vrus Influenza ................................................................................................... 56
Nucletidos ........................................................................................... 57
Funes ............................................................................................................. 57
Estrutura ............................................................................................................ 57
Bases Azotadas ........................................................................................................ 57
Pentoses .................................................................................................................. 58
Nuclesidos ............................................................................................................. 58
Nucletidos ............................................................................................................. 58
Nucletidos Cclicos................................................................................................. 59
Nuclesidos 5-Trifosfato ........................................................................................ 60
cidos Nucleicos ................................................................................................ 60
Diferenas entre DNA e RNA ................................................................................... 60
Introduo aos Metabolismos ............................................................... 62
Gliclise................................................................................................. 63
Reaces da Gliclise .............................................................................................. 64
Regulao da Gliclise ....................................................................................... 65
Etapa 1 ..................................................................................................................... 65
Etapa 3 ..................................................................................................................... 66
Etapa 10................................................................................................................... 66
Destinos Metablicos dos Produtos da Gliclise .................................................... 67
Via de Rapoport-Luebering ............................................................................... 68
Correlaes Clnicas .......................................................................................... 68
Anemia Hemoltica .................................................................................................. 68

5
Brigitta Cismasiu | FMUL 2010/2011

Enfarte Agudo do Miocrdio ................................................................................... 68


Ciclo dos cidos Tricarboxlicos ............................................................. 69
Entrada no Ciclo TCA ......................................................................................... 69
Descarboxilao do Piruvato a Acetil-CoA .............................................................. 69
Reaces da Descarboxilao Oxidativa ................................................................. 70
Regulao da Piruvato Desidrogenase .................................................................... 70
Ciclo TCA ............................................................................................................ 70
Reaces do Ciclo TCA............................................................................................. 71
Regulao do Ciclo TCA ..................................................................................... 72
Etapa 1 ..................................................................................................................... 73
Etapa 3 ..................................................................................................................... 73
Etapa 4 ..................................................................................................................... 73
Ciclo Anfiblico .................................................................................................. 73
Correlao Clnica .............................................................................................. 74
Beribri .................................................................................................................... 74
Cadeia Respiratria ............................................................................... 75
Componentes da Cadeia Respiratria ..................................................................... 75
Organizao ............................................................................................................. 76
ATP Sintase .............................................................................................................. 76
Shuttle Malato-Aspartato ........................................................................................ 77
Inibidores................................................................................................................. 77
Desacopladores ....................................................................................................... 77
Ciclo de Cori .......................................................................................... 79
Gluconeognese .................................................................................... 80
Reaces Exclusivas da Gluconeognese ................................................................ 81
Regulao da Gluconeognese ......................................................................... 81
Etapa 1 ..................................................................................................................... 81
Etapa 2 ..................................................................................................................... 82
Etapa 3 ..................................................................................................................... 83
Etapa 4 ..................................................................................................................... 83
Experincia ........................................................................................................ 84

6
Brigitta Cismasiu | FMUL 2010/2011

Qual seria a consequncia da existncia de fosfoenolpiruvato carboxicinase no


tecido muscular? ..................................................................................................... 84
Metabolismo do Glicognio................................................................... 86
Glicogenlise ..................................................................................................... 86
Glicognese ....................................................................................................... 87
Doenas Associadas ao Metabolismo do Glicognio ........................................ 88
Von Gierke (glucose 6-fosfatase) ............................................................................ 88
Pompe (-1,4 glicosidase) ....................................................................................... 88
Cori (enzima desramificadora) ................................................................................ 88
Andersen (enzima ramificadora) ............................................................................. 88
McArdle (glicognio fosforilase muscular).............................................................. 88
Hers (glicognio fosforilase heptica) ..................................................................... 88
Tarui (fosfofrutocinase muscular) ........................................................................... 88
Correlao Clnica .............................................................................................. 88
Doena de Von Gierke............................................................................................. 88
Via dos Fosfatos de Pentose .................................................................. 90
Importncia da Via .................................................................................................. 90
Reaces da Via dos Fosfatos de Pentose .............................................................. 90
Importncia do NADPH ........................................................................................... 91
Regulao da Via dos Fosfatos de Pentose ............................................................. 92
Correlao Clnica .............................................................................................. 93
Deficincia de G6PD ................................................................................................ 93
Metabolismo dos Nucletidos ............................................................... 94
Degradao de Nucletidos .............................................................................. 94
Degradao de Nucletidos Purnicos .................................................................... 94
Degradao de Nucletidos Pirimidnicos .............................................................. 95
Sntese de Nucletidos ...................................................................................... 95
Biossntese de Novo de Nucletidos Purnicos ....................................................... 96
Biossntese de Novo de Nucletidos Pirimidnicos ................................................. 97
Converso dos Monofosfatos de Nuclesidos em Trifosfatos de Nuclesidos ...... 98
Os Ribonucletidos so Percursores dos Desoxirribonucletidos.......................... 98

7
Brigitta Cismasiu | FMUL 2010/2011

Biossntese do Timidilato ........................................................................................ 99


Via da Reciclagem ................................................................................................ 99
Correlaes Clnicas .......................................................................................... 99
Gota ......................................................................................................................... 99
Sndrome de Lesch-Nyhan..................................................................................... 100
Frutlise .............................................................................................. 101
Correlao Clnica ............................................................................................ 101
Retinopatia Diabtica ............................................................................................ 101
Liplise ou -Oxidao ........................................................................ 102
Libertao dos cidos Gordos ......................................................................... 102
cidos Gordos da Dieta ................................................................................... 103
Activao dos cidos Gordos .......................................................................... 103
Entrada do Acil-CoA para a Matriz Mitocondrial ............................................ 103
-Oxidao ...................................................................................................... 104
Reaces da -Oxidao........................................................................................ 105
Balano Energtico ................................................................................................ 105
-Oxidao de cidos Gordos com Nmero mpar de Carbonos ......................... 105
-Oxidao de cidos Gordos Insaturados ........................................................... 105
-Oxidao no Peroxissomas ................................................................................ 106
Correlao Clnica ............................................................................................ 106
Deficincia de Carnitina ........................................................................................ 106
Cetognese ......................................................................................... 107
Corpos Cetnicos ............................................................................................ 107
Cetognese ...................................................................................................... 107
Reaces da Cetognese ....................................................................................... 108
Utilizao de Corpos Cetnicos ............................................................................. 108
Diferenas entre Sntese e Utilizao de Corpos Cetnicos ................................. 109
Os Animais no Conseguem Converter cidos Gordos em Glucose .................... 109
Lipognese .......................................................................................... 110
Transporte Atravs da Membrana Mitocondrial ............................................ 110
Formao de Malonil-CoA ..................................................................................... 111

8
Brigitta Cismasiu | FMUL 2010/2011

Sntese de cidos Gordos ................................................................................ 111


Reaces da Sntese de cidos Gordos ................................................................. 112
Sintase de cidos Gordos ...................................................................................... 112
Etapa Limitante ..................................................................................................... 113
Sntese de TAG ................................................................................................ 114
Gliceroneognese ........................................................................................... 115
Sntese de Fosfolpidos ........................................................................ 117
Colesterol ............................................................................................ 118
Sntese de Colesterol ....................................................................................... 118
Fases da Sntese do Colesterol .............................................................................. 119
Regulao da Sntese do Colesterol ...................................................................... 120
Transporte de Colesterol e TAG pelo Organismo ........................................... 121
Ciclo Exgeno ........................................................................................................ 122
Ciclo Endgeno ...................................................................................................... 122
Endocitose Mediada por Receptores .................................................................... 123
Aterosclerose e Doenas Cardacas ...................................................................... 124
cidos Biliares ....................................................................................................... 125
Correlao Clnica ............................................................................................ 126
Hipercolesterolmia Familiar ................................................................................ 126
Ciclo do Azoto ..................................................................................... 127
Metabolismo dos Aminocidos ........................................................... 128
Transaminao ................................................................................................ 128
Desaminao ................................................................................................... 129
Desaminao Directa ............................................................................................ 129
Desaminao Reversvel........................................................................................ 129
Desaminao Oxidativa ......................................................................................... 130
Catabolismo de Aminocidos .......................................................................... 130
Destino do Grupo Amina ....................................................................................... 131
Ciclo da Ureia ........................................................................................................ 131
As Reaces do Ciclo da Ureia............................................................................... 132
Regulao do Ciclo da Ureia .................................................................................. 133

9
Brigitta Cismasiu | FMUL 2010/2011

Interligao Metablica ........................................................................................ 133


Destino do Esqueleto de Carbono ........................................................................ 134
Ciclo da Alanina-Glucose ....................................................................................... 135
Ciclo do Metilo Activado ....................................................................................... 138
Derivados dos Aminocidos ............................................................................ 139
Creatina ................................................................................................................. 139
Glutatio ................................................................................................................ 139
Correlaes Clnicas ........................................................................................ 140
Fenilcetonria........................................................................................................ 140
Deficincia de Ornitina Transcarbamoilase .......................................................... 141
Hiperhomocisteinmia .......................................................................................... 141
Inter-relaes Metablicas .................................................................. 142
Introduo ....................................................................................................... 142
Fgado .................................................................................................................... 142
Crebro .................................................................................................................. 143
Msculo ................................................................................................................. 143
Tecido Adiposo ...................................................................................................... 144
Ciclo jejum-alimentado ................................................................................... 145
Estado Bem-Alimentado ....................................................................................... 145
Fases Iniciais de Jejum........................................................................................... 146
Jejum ..................................................................................................................... 146
Fases Iniciais do Estado Realimentado ................................................................. 148
Interaces Metablicas entre rgos ................................................................. 148
Necessidades Energticas, Reservas e Homeostasia Calrica .............................. 149
Homeostasia da Glucose ....................................................................................... 150
Estados Hormonais e Nutricionais .................................................................. 152
Obesidade.............................................................................................................. 152
Dieta ...................................................................................................................... 153
Diabetes Mellitus Tipo 2 ou Insulino-Resistente................................................... 154
Diabetes Mellitus Tipo 1 ou Insulino-Dependente ............................................... 154
Exerccio Aerbico e Anaerbico .......................................................................... 155
Consumo de lcool................................................................................................ 156

10
Brigitta Cismasiu | FMUL 2010/2011

Regulao do Metabolismo ................................................................. 159


Estratgias de Regulao ................................................................................ 159
Correlao Anatmica ..................................................................................... 159
Regulao Hormonal ....................................................................................... 160
Insulina e Glucagina na Regulao dos Metabolismos ......................................... 160
Vitaminas ......................................................................................................... 161
cido Flico ........................................................................................................... 161
Cobalamina............................................................................................................ 161
cido Ascrbico ..................................................................................................... 162
Piridoxina ............................................................................................................... 162
Tiamina .................................................................................................................. 162
Niacina ................................................................................................................... 162
Riboflavina ............................................................................................................. 162
Biotina ................................................................................................................... 163
cido Pantotnico ................................................................................................. 163
Vitamina A ............................................................................................................. 163
Vitamina D ............................................................................................................. 163
Vitamina K ............................................................................................................. 163
Vitamina E ............................................................................................................. 163
Correlaes Clnicas ........................................................................................ 164
Anemias Nutricionais ............................................................................................ 164
Anemia Perniciosa ................................................................................................. 164
Casos de Estudo .................................................................................. 165
Caso 1 .............................................................................................................. 165
Caso 2 .............................................................................................................. 165
Caso 3 .............................................................................................................. 165
Caso 4 .............................................................................................................. 166
Caso 5 .............................................................................................................. 166
Caso 6 .............................................................................................................. 166
Caso 7 .............................................................................................................. 167
Caso 8 .............................................................................................................. 167

11
Brigitta Cismasiu | FMUL 2010/2011

Caso 9 .............................................................................................................. 167


Caso 10 ............................................................................................................ 168
Solues .......................................................................................................... 168
Caso 1 .................................................................................................................... 168
Caso 2 .................................................................................................................... 169
Caso 3 .................................................................................................................... 169
Caso 4 .................................................................................................................... 169
Caso 5 .................................................................................................................... 170
Caso 6 .................................................................................................................... 170
Caso 7 .................................................................................................................... 170
Caso 8 .................................................................................................................... 170
Caso 9 .................................................................................................................... 171
Caso 10 .................................................................................................................. 171

12
Brigitta Cismasiu | FMUL 2010/2011

Noes Bsicas
Isomerismo

Ismeros Compostos diferentes que apresentam a mesma frmula molecular


mas diferente frmula de estrutura:
Planos diferem pelas frmulas estruturais planas
o De Cadeia apresentam diferentes tipos de cadeia
o De Funo pertencem a funes diferentes
o De Posio pertencem mesma funo e tm o mesmo tipo de
cadeia, mas apresentam diferente posio de um grupo funcional,
ramificao ou insaturao
o De Compensao (Metmeros) diferente posio do
heterotomo
o Dinmicos (Tautmeros) os ismeros coexistem em equilbrio
dinmico em soluo
Espaciais (Estereoismeros) diferem pelas frmulas estruturais espaciais
o Geomtricos (Cis-Trans) disposio espacial diferente dos
grupos ligados aos carbonos da ligao dupla
o pticos (Enantimeros) compostos assimtricos (quirais) que
apresentam efeito fisiolgico diferente e desviam a luz polarizada
para direces diferentes
Epmeros s diferem num carbono
No Epmeros diferem em vrios carbonos

Termodinmica

G
Variao da energia livre de Gibbs (energia que se pode libertar de um sistema
em transformao a presso constante). Descreve a direco para a qual a
reaco qumica vai tender e a concentrao de reagentes e produtos presentes
no equilbrio.
G>0 reaco endergnica
G<0 reaco exergnica

H
Variao de entalpia de um sistema, isto , o calor libertado ou absorvido quando
h quebra ou formao de ligaes, a presso constante.
H>0 reaco endotrmica
H<0 reaco exotrmica

S
Variao de entropia de um sistema, ou seja, mede o estado de ordem ou
desordem de um sistema termodinmica. Multiplicada pela temperatura, d a
medida de quantidade de energia que no se liberta no decurso de uma
transformao (energia degenerada).
A entropia tanto mais baixa quanto mais organizado for um sistema.

13
Brigitta Cismasiu | FMUL 2010/2011

G0
Variao da energia livre de Gibbs nas condies padro ( ; ;
;[ ] [ ] ).

G0'
Variao da energia livre de Gibbs nas condies padro ( ; ;
[ ] [ ] ), excepto o pH.

RELAO FUNDAMENTAL DA TERMODINMICA


Quando uma reaco qumica ocorre a temperatura constante:

(T = temperatura, em K)

G=0 reaco em equilbrio


G>0 reaco no espontnea
G<0 reaco que pode ser espontnea

OUTRAS RELAES

(R = 8,314472 J mol-1 K-1)

DIRECCIONAR UMA REACO


Para que uma reaco se d no sentido directo, podemos alterar os seguintes
parmetros:
[Reagentes]
[Produtos]
Temperatura
pH
Presso
Enzima
Acoplar outra reaco

Enzimas

Enzimas so protenas com actividade cataltica, isto , que aceleram as reaces


qumicas que se processam nos organismos.

As enzimas no so consumidas nas reaces que catalisam, e qualquer alterao


molecular que ocorra durante o processo cataltico reposta, de modo a obter-se
a enzima na forma molecular e/ou estrutural inicial. As enzimas no interferem no
estado de equilbrio e diminuem a energia de activao das reaces que
catalisam.

O centro activo constitudo pelo conjunto de aminocidos que entram em


contacto com o substrato. Compreende o local de fixao, que se combina com o

14
Brigitta Cismasiu | FMUL 2010/2011

substrato por ligaes fracas, e o centro cataltico, que actua sobre o substrato,
levando-o a sofrer a reaco qumica. Nas enzimas que actuam atravs de um
cofactor, que responsvel pela transformao estrutural do substrato, essa
estrutura encontra-se ligada enzima na vizinhana muito prxima do centro
cataltico.

Classificao
Holoenzima:
Exclusivamente proteica.
Constituda por:
o Apoenzima parte proteica
o Cofactor parte no proteica:
Io metlico;
Grupo prosttico composto orgnico no-proteico
ligado covalentemente;
Coenzimas pequenas molculas que transportam
grupos qumicos e que participam na catlise.

De acordo com o tipo de reaces que catalisam, podemos considerar seis


classes:
Oxirredutases transferncia de electres (oxidao-reduo). Ex.:
desidrogenases, oxidases.
Transferases transferncia de grupos funcionais (amina, fosfato, acil,
carboxilo,). Ex.: cinases, aminotransferases.
Hidrolases hidrlise de ligaes covalentes. Ex.: peptidases.
Liases quebra de ligaes covalentes e a remoo de molculas de gua,
amnia e dixido de carbono. Ex.: desidratases, descarboxilases.
Isomerases interconverso entre ismeros pticos ou geomtricos. Ex.:
epimerases, mutases.
Ligases formao de novas molculas a partir da ligao entre duas j
existentes, custa de energia (ATP). Ex.: aminoacil-tRNA sintetases.

Cintica Enzimtica
Uma enzima:
no altera K (constante de equilbrio);
s actua se G<0;
diminui a energia de activao, aumentando a sua velocidade.

Num estado de equilbrio, as velocidades das reaces directa e inversa mantm-


se ao longo do tempo.

Num estado estacionrio, algumas propriedades so mantidas constantes ao


longo do tempo, sem que haja equilbrio. A concentrao de ES permanece
constante (vformao = vdissociao).

Equao de Michaelis-Menten
Leonor Michaelis e Maud Menten concluram, em 1913, o estado quantitativo das
variaes da velocidade de uma reaco enzimtica em funo do aumento da
concentrao de substrato. A sua teoria baseava-se na suposio de uma enzima

15
Brigitta Cismasiu | FMUL 2010/2011

(E) e o seu substrato (S) se associam reversivelmente para formar um complexo


enzima-substrato (ES).
E enzima
v1 v2
E+S ES E+P S substrato
ES complexo enzima-substrato
v-1 v-2 P produto

A quantidade de P formada, assim como a velocidade da reaco, vo depender


directamente da concentrao em complexo ES, ou seja, da quantidade de enzima
que se liga ao substrato.

DEDUO DA EQUAO DE MICHAELIS-MENTEN:


1 1 [] []
1 1 []
2 2 [] ? ; [] ?
2 2 [] []

[] ?
[] constante (estado estacionrio)
[] []
1 2 1 1 [] [] 2 [] 1 []
[]
2 1
[] [] [] ([] []) []
1
[] [] [] [] [] []
[] []
[] [] ( []) [] []
[]

?
[] []
2 2 [] 2
[]
[]

[]

KM uma medida da afinidade de uma enzima


para o substrato: essa afinidade igual a 1/ KM. A
determinao de KM depende da determinao
de vmx, ou seja, da concentrao de S a partir da
qual a velocidade no aumenta. Assim, recorre-
se ao valor de vmx/2, de modo a obter um valor
mais prximo do real: KM igual concentrao
de substrato necessria para atingir vmx/2.

16
Brigitta Cismasiu | FMUL 2010/2011

Linearizao de Lineweaver-Burk
Em 1934, Hans Lineweaver e Dean Burk
forma os primeiros a transformar a
interpretao cintica de Michaelis-Menten
numa interpretao linear, obtendo uma
relao entre v e S que pode ser traduzida
graficamente por uma recta.

Invertendo ambos os membros da equao


de Michaelis-Menten e isolando 1/v, pode
obter-se a equao de Lineweaver-Burk:


[]

Factores que Influenciam a Velocidade da Reaco


CONCENTRAO DE SUBSTRATO

Velocidade mxima: a taxa ou velocidade de uma reaco


(v) o nmero de molculas de substrato convertidas em
produto por unidade de tempo. A velocidade
normalmente expressa em mol de produto formado por
minuto. A taxa de uma reaco catalisada por uma enzima
aumenta com a concentrao do substrato at atingir uma
velocidade mxima (vmx). Esse patamar reflecte a saturao
de todos os locais de ligao disponveis nas molculas de
enzima.

Forma hiperblica da curva de cintica


enzimtica: a maioria das enzimas apresentam
cintica de Michaelis-Menten, em que a curva do
grfico da velocidade inicial de reaco (vo) em
relao concentrao de substrato ([S])
hiperblica. No entanto, as enzimas alostricas
apresentam normalmente uma curva sigmoidal.

TEMPERATURA

Aumento da velocidade com a temperatura: a velocidade de reaco aumenta


com a temperatura at atingir um pico. Esse aumento resultado do aumento do
nmero de molculas com energia suficiente para ultrapassar a barreira de
energia e formar produtos da reaco.

Diminuio da velocidade a uma temperatura mais elevada: o aumento contnuo


da temperatura resulta numa reduo da velocidade de reaco, como resultado
da desnaturao da enzima induzida pela temperatura.

17
Brigitta Cismasiu | FMUL 2010/2011

A temperatura ptima para a maioria das enzimas humanas entre 35 e 40 C,


sendo que comeam a desnaturar a temperaturas acima de 40 C.

pH

Efeito do pH sobre a ionizao do centro activo: o processo cataltico geralmente


requer que a enzima e o substrato tenham grupos qumicos especficos no estado
ionizado ou no-ionizado que possam interagir. Por exemplo, a actividade
cataltica pode exigir que um grupo amina da enzima esteja na forma protonada
(NH3+). Em pH alcalino, este grupo desprotonado e a velocidade da reaco
diminui.

Efeito do pH sobre a desnaturao da enzima: extremos de pH tambm podem


levar desnaturao da enzima, pois a estrutura da protena cataliticamente
activa depende do carcter inico das cadeias laterais de aminocidos.

O pH ptimo varia para diferentes enzimas: o pH


no qual a actividade enzimtica mxima
atingida diferente para diferentes enzimas e
muitas vezes reflecte a concentrao de H+ a que
a enzima funciona no organismo. Por exemplo, a
pepsina, uma enzima digestiva no estmago,
optimamente activa a pH=2, enquanto que outras
enzimas, destinadas a funcionar a pH neutro, so
desnaturadas por esse ambiente cido.

Tipos de Inibio
Inibio enzimtica diminui a actividade da enzima e pode ser:
Reversvel o inibidor liga-se de modo no covalente, sendo a actividade
biolgica recuperada aps a dissociao:
o Competitiva o inibidor estruturalmente semelhante ao
substrato e compete pelo centro activo com o substrato; diminuiu
a afinidade (KM aumenta), mantm a vmx;
o Incompetitiva o inibidor liga-se apenas ao complexo ES, diminui
a formao de produto e altera o equilbrio ,
deslocando-o no sentido directo, de formao de ES e consumo
dos reagentes; diminuio de KM (aparente aumento de afinidade)
e da vmx;

18
Brigitta Cismasiu | FMUL 2010/2011

o Mista (No-Competitiva) o inibidor liga -se a um local diferente


do centro activo, pelo que a enzima pode continuar a ligar a si o
substrato mas, como a conformao do centro activo de algum
modo afectada, essa ligao efectua-se com menos facilidade;
diminuiu a vmx, mantm a afinidade.
Irreversvel h modificao covalente e definitiva no centro de ligao
ou no centro cataltico da enzima.

Receptorologia

Um sinal reconhecido atravs da interaco com um


componente celular, na maioria das vezes um receptor da
superfcie celular. A informao contida no sinal
convertida noutras formas qumicas ou transduzida. O
processo de transduo compreende, normalmente, muitas
etapas. O sinal amplificado antes de produzir uma
resposta. O mecanismo de feedback regula todo o processo
de sinalizao.

As vias de transduo de sinal seguem um circuito molecular que contm certas


etapas principais:
1. Libertao do 1o mensageiro: um estmulo desencadeia a libertao de
um sinal molecular;
2. Recepo do 1o mensageiro: protenas de membrana actuam como
receptores, ligando-se ao sinal (ligante) e transferindo a informao para
o interior da clula. Essa interaco altera a estrutura terciria ou
quaternria do receptor;
3. Entrega da mensagem no interior da clula por um 2o mensageiro:
outras pequenas molculas so usadas para transmitir a informao a
partir dos complexos ligante-receptor, alterando a sua concentrao
intracelular (ex.: cAMP, cGMP, Ca2+, IP3, DAG, etc). O uso de 2os
mensageiros tem algumas consequncias: (1) leva amplificao do sinal,
pois a activao de um nico receptor leva formao de muitas destas
molculas, (2) os 2os mensageiros difundem-se livremente para outros
compartimentos, influenciando vrios processos celulares, e (3) pode
haver conversa cruzada, pois a actuao de vrias vias de sinalizao
pode alterar a concentrao de um 2o mensageiro comum;
4. Activao de efectores que alteram directamente a resposta fisiolgica:
o ltimo efeito da via de sinalizao activar ou inibir bombas inicas,
enzimas e factores de transcrio gnica que controlam directamente vias
metablicas, activao de genes e alguns processos como a transmisso
nervosa;
5. Terminao do sinal: aps uma clula ter completado a sua resposta a
um sinal, o processo de sinalizao deve ser terminado ou a clula perde a
sua capacidade de resposta a novos sinais.

Um frmaco uma substncia qumica que, quando aplicada a um sistema


fisiolgico, afecta o seu funcionamento.

19
Brigitta Cismasiu | FMUL 2010/2011

De um modo geral, h quatro tipos de molculas reguladoras que normalmente


actuam como alvos farmacolgicos primrios:
1. Receptores;
2. Enzimas;
3. Molculas transportadoras;
4. Canais inicos.

Receptores
Elementos (sensores) que estabelecem comunicao qumica e
coordenam, assim, a funo de todas as clulas no organismo;
Protenas transmembranares que reconhecem e aos quais se ligam
substncias qumicas endgenas (como hormonas, neurotransmissores),
de modo especfico e reversvel;
A ligao traduz-se numa determinada resposta, com modificao do
comportamento celular transduo de sinal alteraes metablicas,
da expresso gentica,

Canais Inicos
Possuem um receptor na sua estrutura;
Podem ser ocupados por:
o Antagonistas (bloqueiam o canal, impedindo a passagem de ies);
o Agonistas (modulam a actividade do canal, aumentando ou
diminuindo a permeabilidade, consoante o canal inico em
causa).

Enzimas
Frmaco um substrato anlogo, que inibe competitivamente a enzima;

20
Brigitta Cismasiu | FMUL 2010/2011

Molcula do frmaco liga-se irreversivelmente e no competitivamente


enzima (ex.: aspirina);
Frmaco actua como falso substrato: sofre aco enzimtica e
transformado quimicamente, originando um produto metablico que
normalmente no produzido e que altera via metablica normal).

Nota: Alguns frmacos necessitam de ser degradados enzimaticamente para


ficarem activos pr-frmacos.

Molculas Transportadoras
Semelhantes a canais inicos, embora geralmente dem passagem a
molculas de maiores dimenses;
So importantes no transporte de molculas polares (e, por isso, pouco
lipossolveis) atravs das membranas celulares;
A actividade das molculas transportadoras est muitas vezes associada a
fenmenos de simporte ou antiporte, com transporte de Na+ na mesma
direco ou na direco oposta da molcula co-transportada,
respectivamente;
Exemplos:
1. Transportadores de glucose e aminocidos para o interior das
clulas;
2. Transportadores de Na+ e Ca2+ para o exterior das clulas;
3. Transportadores de neurotransmissores (ou precursores de
neurotransmissores) pelos terminais nervosos.

21
Brigitta Cismasiu | FMUL 2010/2011

Agonista
Ligante que se liga ao receptor e desencadeia uma resposta biolgica;
Para que haja resposta, a ligao ao receptor tem de ter uma
determinada afinidade (que garanta ligao ao receptor) e eficcia (que
traduza a resposta desencadeada);
Quando o agonista uma molcula hidrossolvel (ex.: hormonas
peptdicas), liga-se a receptores membranares e o sinal desencadeado
pela ligao transmitido e amplificado pela activao de molculas
intracelulares 2os mensageiros (ex.: cAMP, cGMP, IP3) sendo a
resposta rpida, ao nvel da actividade enzimtica;
Se o agonista for lipossolvel (ex.: hormonas esterides), entra na clula,
atravessando a bicamada fosfolipdica da membrana, para se ligar a
receptores intracelulares. O complexo ligante-receptor actua sobre o
DNA, desencadeando uma resposta mais lenta, ao nvel da expresso
gnica, por modificao da sntese proteica.

Antagonista
Substncia que se liga ao receptor, mas no desencadeia resposta
biolgica;
Tem afinidade (porque estabelece ligao), mas no tem eficcia (j que
no produz nenhuma resposta celular);
Utilizado terapeuticamente para corrigir situaes de
hiperactividade/hiperfuno (o antagonista bloqueia o receptor e diminui,
assim, a sua actividade).

Receptores de Membrana e Mecanismos de Transduo de Sinais


De acordo com a estrutura e mecanismo de transduo, existem 4 tipos de
receptores:
1. Receptores ionotrpicos;
2. Receptores metabotrpicos;
3. Receptores associados a protenas cinases;
4. Receptores intranucleares.

22
Brigitta Cismasiu | FMUL 2010/2011

Receptores Ionotrpicos ou Canais Inicos


Os receptores deste tipo controlam os eventos sinpticos mais rpidos do sistema
nervoso, nos quais um neurotransmissor actua na membrana ps-sinptica de
uma clula nervosa ou muscular e aumenta transitoriamente a sua
permeabilidade a ies particulares. A maioria dos neurotransmissores
excitatrios, como a acetilcolina, na juno neuromuscular, ou o glutamato, no
sistema nervoso central, causa um aumento na permeabilidade de Na+ e K+. Isto
causa uma despolarizao da clula e um aumento da probabilidade de ser
gerado um potencial de aco. A aco do transmissor atinge um pico numa
fraco de um milissegundo, e decresce em poucos milissegundos, o que significa
que o acoplamento entre o receptor e os canais inicos directo, o que
possibilitado pela estrutura molecular do complexo receptor-canal.

Receptores acoplados protena G podem controlar directamente canais inicos,


sem interveno de segundos mensageiros, estabelecendo-se interaco entre a
subunidade da protena G e o canal inico.

23
Brigitta Cismasiu | FMUL 2010/2011

Exemplos: receptores de acetilcolina* do msculo cardaco e receptores para


analgsicos opiides** nas clulas neuronais controlam a permeabilidade dos
canais de io K+.

* Acetilcolina um neurotransmissor que causa vasodilatao, diminuio da


frequncia cardaca e reduo de fora de contraco cardaca.

** Opiides so um grupo de frmacos que actuam nos receptores opiides


neuronais. Eles produzem aces de insensibilidade dor (analgesia) e so usados
principalmente na terapia da dor crnica e da dor aguda de alta intensidade.

RESUMINDO:
Esto envolvidos principalmente na transmisso sinptica rpida.
Existem vrias famlias estruturais, a mais comum sendo conjuntos
heteromricos de quatro ou cinco subunidades, com hlices
transmembranares organizadas em torno de um canal central aquoso.
A ligao de um ligante e a abertura do canal ocorre numa escala de
tempo de milissegundos.

Receptores Metabotrpicos ou GPCRs (Associados Protena G)


A famlia dos GPCR inclui os receptores muscarnicos, adrenrgicos, de dopamina,
opiides, de pptidos e de purinas e os quimiorreceptores envolvidos no olfacto e
deteco de feromonas.

Muitos neurotransmissores, alm de pptidos, podem interagir tanto com GPCRs


como com canais inicos, permitindo que a mesma molcula produza uma ampla
variedade de efeitos. As hormonas peptdicas individuais, por outro lado, actuam
tanto em GPCRs como em receptores ligados a cinases, mas raramente em
ambos, e uma selectividade semelhante se aplica a muitos ligantes que atuam
sobre receptores nucleares.

Um GPCR consiste numa cadeia polipeptdica at 1100 aminocidos, sendo a sua


estrutura composta por sete hlices transmembranares, com um domnio N-
terminal extracelular de comprimento varivel e um domnio C-terminal
intracelular.

Os GPCRs so divididos em trs famlias distintas, que tm em comum a estrutura


hepta-helicoidal, mas diferem noutros aspectos, principalmente no comprimento
do domnio N-terminal extracelular e da localizao do domnio de ligao ao
agonista.

A rodopsina uma protena abundante na retina e produz uma resposta nos


bastonetes (hiperpolarizao, associada inibio da condutncia de Na+) atravs
de um mecanismo que envolve uma protena G. No entanto, neste caso, um
foto, e no uma molcula agonista, que produz a resposta. Ou seja, a rodopsina
incorpora a sua prpria molcula agonista, o retinal, que isomerisa da
configurao trans (inactiva) para a cis (activa) quando absorve um foto.

24
Brigitta Cismasiu | FMUL 2010/2011

As protenas G constituem uma famlia de protenas membranares cuja funo


reconhecer GPCRs activados e passar a mensagem aos sistemas efectores que
geram uma resposta celular. So chamadas de protenas G por causa de sua
interaco com os nucletidos da guanina, GTP e GDP. So constitudas por trs
subunidades: , e . Os nucletidos de guanina ligam-se subunidade , que
tm actividade enzimtica, catalisando a converso de GTP em GDP. As
subunidades e permanecem juntas num complexo . As trs subunidades
esto ancoradas membrana atravs de uma cadeia de cidos gordos, acoplados
protena G por meio de uma reaco conhecida como prenilao.

No estado de repouso, a protena G est sob a forma de um trmero solto,


com um GDP ocupando a subunidade . Quando um GPCR activado por um
agonista, ocorre uma mudana conformacional, envolvendo o domnio
citoplasmtico do receptor, fazendo-o adquirir elevada afinidade para .
Associao do ao receptor faz com que o GDP se dissocie e seja substitudo
por GTP, que, por sua vez, provoca a dissociao do trmero da protena G,
libertando as subunidades -GTP e . Estas formas activas difundem-se na
membrana e podem associar-se a vrias enzimas e canais inicos, causando
activao do alvo.

25
Brigitta Cismasiu | FMUL 2010/2011

Quando a subunidade , atravs da sua actividade GTPsica, hidrolisa o GTP a


GDP, a sinalizao termina. A subunidade -GDP resultante dissocia-se do efector
e rene-se com a subunidade , completando o ciclo.

A ligao da subunidade a um efector aumenta a sua actividade GTPsica. A


magnitude deste aumento diferente para diferentes tipos de efectores. Uma vez
que a hidrlise do GTP a etapa que termina a capacidade da subunidade
produzir o seu efeito, a regulao da sua actividade GTPsica pela protena
efectora significa que a activao do efector tende a ser autolimitada. O
mecanismo resulta numa amplificao, pois um nico complexo agonista-receptor
pode activar vrias protenas G, e cada uma delas pode permanecer associada
enzima efectora por tempo suficiente para produzir vrias molculas de produto.
O produto muitas vezes um 2o mensageiro e a amplificao adicional ocorre
antes de a resposta celular final ser produzida.

Nota: A fosforilao o mecanismo regulador de grande parte dos processos


metablicos em clulas eucariticas. As cinases so enzimas que catalisam
preferencialmente reaces de fosforilao, com converso de ATP a ADP (ATP
ADP + Pi), reaco muito favorvel por ser muito exergnica. As fosfatases
catalisam geralmente reaces de desfosforilao, que no envolvem passagem
de ATP a ADP, visto tratar-se de uma reaco pouco favorvel do ponto de vista
energtico. O ATP o dador mais comum de grupos fosfato.

Funes das GPCRs:


Aco e secreo hormonal;
Transmisso nervosa;
Quimiotaxia (migrao de clulas, sobretudo leuccitos, em direco a um
gradiente qumico);
Exocitose;
Embriognese;

26
Brigitta Cismasiu | FMUL 2010/2011

Crescimento e diferenciao celulares;


Funes sensitivas (olfacto, paladar, viso);Infeco viral.

Os principais alvos das protenas G, atravs dos quais os GPCRs controlam os


diferentes aspectos da funo celular, so:
Guanilato ciclase, responsvel pela formao de cAMP;
Fosfolipase C, responsvel pela formao de fosfato de inositol e
diacilglicerol;
Canais inicos, particularmente de clcio e potssio;
RhoA/Rho-cinase, um sistema que controla a actividade de muitas vias de
sinalizao, controlando o crescimento e a proliferao celular, a
contraco do msculo liso, etc.

GUANILATO CICLASE
O cAMP um nucletido sintetizado na clula a partir do ATP pela aco de uma
enzima ligada membrana, a guanilato ciclase. produzido continuamente e
inactivado pela hidrlise de 5'-AMP, atravs da aco de uma famlia de enzimas,
as fosfodiesterases. Diversos medicamentos, hormonas e neurotransmissores
actuam nos GPCRs e produzem os seus efeitos aumentando ou diminuindo a
actividade cataltica da guanilato ciclase, aumentando ou diminuindo a
concentrao de cAMP dentro da clula. Existem diversas isoformas moleculares
da enzima, algumas das quais respondem selectivamente a Gs ou Gi.

O AMP cclico regula vrios aspectos da funo celular, incluindo, por exemplo,
enzimas envolvidas no metabolismo, a diviso e a diferenciao celular, o
transporte de ies, os canais inicos e as protenas contrcteis do msculo liso.
Estes efeitos variados so, porm, provocados por um mecanismo comum, ou
seja, a activao de protenas cinases pelo cAMP. As protenas cinases regulam a
funo de vrias protenas celulares por fosforilao de protenas que controlam.
O cAMP hidrolisado no interior das clulas por fosfodiesterases. Existem vrios
subtipos de fosfodiesterases, alguns dos quais so cAMP-selectivas, enquanto
outras so cGMP-selectivas.

27
Brigitta Cismasiu | FMUL 2010/2011

FOSFOLIPASE C
O fosfatidilinositol (PIP2) um importante 2o mensageiro intracelular. O PIP2 o
substrato de uma enzima ligada membrana, a fosfolipase C (PLC), que se divide
em diacilglicerol e trifosfato de inositol (IP3), os quais funcionam como 2os
mensageiros. A activao da PLC por vrios agonistas mediada pela protena G.
Aps a clivagem do PIP2, o DAG fosforilado para formar o cido fosfatdico,
enquanto que o IP3 desfosforilado e reassociado ao cido fosfatdico para
formar PIP2 de novo.

CANAIS INICOS
Os GPCRs podem controlar directamente a funo de canais inicos por
mecanismos que no envolvem 2os mensageiros, como o cAMP ou o fosfato de
inositol.

RhoA/Rho-CINASE
Esta via de transduo de sinal activada por GPCRs (e tambm por mecanismos
no-GPCR) que de ligam a protenas G. A subunidade da protena G interage
com um factor de troca de nucletido de guanosina, que facilita a troca de GDP
por GTP numa outra GTPase, a Rho. A Rho-GDP, na forma de repouso, est
inactiva, mas quando a troca GDP-GTP ocorre, a Rho activada que, por sua vez,
activa a Rho-cinase. A Rho-cinase fosforila vrias protenas do substrato e controla
uma grande variedade de funes celulares, incluindo a contraco e proliferao
do msculo liso, a angiognese e a remodelao sinptica.

28
Brigitta Cismasiu | FMUL 2010/2011

RESUMINDO:
So estruturas compostas por -hlices que atravessam a membrana sete
vezes, muitas vezes sob a forma dimrica.
Uma das alas intracelulares maior do que as outras e interage com a
protena G.
A protena G uma protena da membrana composta por trs
subunidades (, e ), a subunidade possuindo actividade GTPsica.
Quando o trmero se liga ao receptor ocupado por antagonista, a
subunidade dissocia-se e activa um efector (uma enzima de membrana
ou um canal inico). Nalguns casos, a subunidade o activador.
A activao do efector termina quando a molcula de GTP ligada
hidrolisada, o que permite que a subunidade se recombine com a
subunidade .
Existem vrios tipos de protena G, que interagem com diferentes
receptores e controlam de diferentes efectores.

Receptores Associados a Protenas Cinases


Esses receptores de membrana mediam as aces de uma ampla variedade de
mediadores de protenas, incluindo factores de crescimento, citocinas e hormonas
como a insulina e a leptina, cujos efeitos so exercidos principalmente ao nvel de
transcrio gnica.

A maioria desses receptores so protenas constitudas por uma nica cadeia at


1000 aminocidos, com uma nica hlice atravessando a membrana, associada a
um grande domnio extracelular ligante e um domnio intracelular de tamanho e
funo variveis.

Os principais tipos de receptores associados a protenas cinases so os seguintes:


Receptores de tirosina cinase (RTKs): a sua estrutura incorpora uma
tirosina cinase na regio intracelular. Eles incluem os receptores para
vrios factores de crescimento;

29
Brigitta Cismasiu | FMUL 2010/2011

Serina/treonina cinases: estruturalmente semelhantes aos RTKs, mas


fosforilam resduos de serina e/ou treonina, em vez de tirosina;
Receptores de citocinas: no apresentam actividade enzimtica
intrnseca. Quando ocupados, associam-se e activam uma tirosina cinase
citoslica;
Receptores ligados a guanilato ciclase: estruturalmente semelhante aos
RTKs, mas a poro enzimtica uma guanilato ciclase e exercem os seus
efeitos estimulando a formao de cGMP.

A fosforilao de protenas um mecanismo fundamental para controlar a funo


das protenas (ex.: enzimas, canais inicos, receptores, protenas transportadoras)
envolvidas na regulao dos processos celulares. A fosforilao e a desfosforilao
so realizadas pelas cinases e fosfatases, respectivamente.

Em muitos casos, a ligao do ligante ao receptor leva dimerizao. A associao


dos dois domnios cinase intracelulares permite uma autofosforilao mtua dos
resduos de tirosina intracelulares. Os resduos de tirosina fosforilados servem
como locais de alta afinidade de encaixe a outras protenas intracelulares, que
formam a prxima etapa da cascata de transduo de sinal.

Um grupo importante dessas protenas so as protenas de domnio SH2, que


possuem uma sequncia altamente conservada de cerca de 100 aminocidos,
formando um local de reconhecimento para os resduos de fosfotirosina do
receptor.

30
Brigitta Cismasiu | FMUL 2010/2011

VIA PROTENA CINASE A

Ligao do ligante GTP hidrolisado a GDP


ao receptor GPCRs

Subunidade Deixa de ser


Protena G activada cessa activao da produzido cAMP
adenilato ciclase a partir de ATP

Libertao GTP liga-se


do GDP subunidade Reassociao da
subunidade com as
subunidades e

Subunidade separa-se
das restantes
Dissociao do complexo
ligante-receptor

Formao do Formao do
complexo -GTP complexo -

Activao da Activao de
adenilato ciclase alvos (enzimas,
canais inicos)

4 cAMP activam a PKA liberta as duas


Sntese de cAMP,
protena cinase A subunidades catalticas (vo
a partir de ATP
(PKA) fosforilar outras protenas)

A protena cinase A (PKA) constituda por dois tipos de subunidades:

1. Subunidade reguladora (R);


2. Subunidade cataltica (C).

Na ausncia de cAMP, a PKA forma o complexo


R2C2 (enzimaticamente inactivo).

Na presena de cAMP (4 molculas), h


dissociao do complexo R2C2 em:
1 subunidade R2;
2 subunidades C livres, aptas a fosforilar
protenas-alvo.

31
Brigitta Cismasiu | FMUL 2010/2011

As respostas luta ou fuga que, em situaes de stress, preparam o msculo para


a aco so o resultado da actividade da PKA. Nestas situaes, a glndula supra-
renal produz a hormona adrenalina (ou epinefrina), que lanada na corrente
sangunea e activa receptores -adrenrgicos do corao e do msculo liso. Com
a activao dos receptores, aumenta a produo de cAMP e a PKA activada,
fosforilando protenas-alvo, provocando assim aumento da frequncia cardaca,
dilatao das pupilas, mobilizao de energia (liplise dos adipcitos,
glicogenlise e gluconeognese) e aumento do fluxo sanguneo no msculo
esqueltico.

VIA PROTENA CINASE C

Mecanismo de transduo
Ligao do ligante Activao da
pela protena G
ao receptor GPCRs fosfolipase C (PLC)
(semelhante ao da via PKA)

Formao de dois PLC catalisa hidrlise de


2os mensageiros fosfatidilinositol (PIP2)

IP3 desfosforilado a
inositol, que se liga ao
Diacilglicerol Trifosfato de
cido fosfatdico,
(DAG) inositol (IP3)
convertendo-se em PIP2
(pronto para novo ciclo)

Protena Cinase C Difunde-se pelo


(Ca2+ dependente) citosol e liga-se a um
activada pelos nveis receptor prprio na
de Ca2+, pelo DAG e membrana das
pela fosfatidilserina vesculas do RE

Protena Cinase C Libertao de


(PKC) activa vai Ca2+ no citosol
fosforilar enzimas
especficas no citosol

Ca2+ e calmodulina
contribuem para juno de
vesculas com membrana
plasmtica nos processos de
exocitose

32
Brigitta Cismasiu | FMUL 2010/2011

VIA PROTENA CINASE G


O sistema da protena cinase G (PKG) cGMP-dependente, ou seja, a produo da
protena PKG estimulada pelo aumento dos nveis de cGMP (2 mensageiro) no
citosol, por aco da guanilato ciclase.

A PKG formada por aco de cGMP vai fosforilar enzimas no citosol, provocando,
entre outros efeitos, relaxamento muscular e vasodilatao, aumentando o
aporte de sangue e a disponibilidade de oxignio.

O monxido de azoto (NO) um composto amplamente utilizado que estimula


esta via, ao aumentar a actividade da guanilato ciclase.

Monxido de estimula
Guanilato ciclase
azoto (NO)

Estimula produo de
cGMP, a partir de ATP

Activao da Protena
Cinase G (PKG)

RESUMINDO:
Receptores de vrios factores de crescimento incorporam uma tirosina
cinase no seu domnio intracelular.
Receptores de citocinas tm um domnio intracelular que se liga e activa a
cinase citoslica quando o receptor est ocupado.

33
Brigitta Cismasiu | FMUL 2010/2011

Os receptores possuem uma estrutura comum, com um grande domnio


extracelular ligado a ligante, associado ao domnio intracelular atravs de
uma hlice que atravessa a membrana uma nica vez.
A transduo de sinal geralmente envolve dimerizao de receptores,
seguido por autofosforilao dos resduos de tirosina. Os resduos de
fosfotirosina agem como aceptores para os domnios SH2 de uma
variedade de protenas intracelulares, permitindo assim um controle de
vrias funes celulares.
Eles esto envolvidos principalmente em eventos de controlo do
crescimento e diferenciao celular, e indirectamente atravs da
regulao da transcrio gnica.
Alguns receptores hormonais tm uma estrutura semelhante mas esto
associados a guanilato ciclase.

Receptores Intranucleares
H pelo menos 48 membros da famlia de receptores nucleares no genoma
humano, o que representa uma percentagem bastante pequena de receptores,
mas so alvos importantes de medicamentos e desempenham um papel vital na
sinalizao endcrina, bem como na regulao metablica.

Os receptores nucleares so factores de transcrio activados por ligantes que


esto envolvidos na transduo de sinais por modificao da transcrio gnica.
Estes receptores no esto incorporados na membrana. Alguns, como os
receptores esterides, tornam-se mveis na presena de seu ligante e podem
translocar-se do citoplasma para o ncleo, enquanto outros, como o RXR, esto
provavelmente confinados no interior do compartimento nuclear. Muitos
receptores nucleares agem como sensores de lpidos e esto intimamente
envolvidos na regulao do metabolismo lipdico dentro da clula.

Os receptores nucleares tm um domnio N-terminal muito heterogneo, que se


liga a outros factores de transcrio e modifica sua prpria ligao ou actividade,
um ncleo altamente conservado, composto por uma estrutura responsvel pelo

34
Brigitta Cismasiu | FMUL 2010/2011

reconhecimento e pela ligao ao DNA, e um domnio C-terminal especfico para


cada classe de receptor.

RESUMINDO:
Famlia de 48 receptores solveis que reconhecem sinais lipdicos e
hormonais e modulam a transcrio gnica.
Existem duas classes principais de receptores nucleares:
o Classe I: esto presentes no citoplasma, formam homodmeros e
migram para o ncleo. Os seus ligantes so principalmente de
natureza endcrina (ex.: hormonas esterides);
o Classe II: esto permanentemente no ncleo e formam
heterodmeros. Os seus ligantes so geralmente lpidos (ex.:
cidos gordos);
o Subclasse: esto envolvidos na transduo de sinais,
principalmente endcrinos, mas funcionam como heterodmeros
(ex.: hormona tiroideia).
Os complexos ligante-receptor desencadeiam alteraes na transcrio
gnica, ligando-se a elementos de resposta hormonal em promotores de
genes e recrutando factores co-activatores ou co-repressores.

Regulao dos Nveis Intracelulares de Clcio


A concentrao intracelular de clcio desempenha um papel importante na
regulao da funo celular (cinases e fosfatases, canais, transportadores,
factores de transcrio, protenas das vesculas sinpticas, etc.).

Intracelularmente, a maior parte do Ca2+ est retido em organelos (retculo


endoplasmtico, retculo sarcoplasmtico, mitocndrias) e os nveis de Ca2+ livre
no citosol so mantidos baixos, para impedir a precipitao de compostos
carboxilados e fosforilados, que forma sais muito pouco solveis quando se
combinam com Ca2+. Estas baixas concentraes so conseguidas atravs de
mecanismos de transporte que expulsam Ca2+ da clula.

O nvel de Ca2+ intracelular determinado por trs mecanismos reguladores que


controlam:
1. Entrada de Ca2+: canais de Ca2+ controlados por voltagem (semelhantes aos
canais de Na+ e K+) e canais de Ca2+ regulados por ligantes (a maioria so
activados por neurotransmissores excitatrios);
2. Sada de Ca2+ da clula: o Ca2+ transportado activamente por uma ATPase
dependente de Ca2+. Tambm pode ocorrer troca de Na+/Ca2+ (troca de 3
Na+ por 1 Ca2+; o gradiente electroqumico de Na+ fornece energia
necessria para sada do Ca2+, sem hidrlise de ATP);
3. Libertao de Ca2+ contido nos organelos celulares: a principal via pela qual
os ies Ca2+ so libertados do RE/RS o receptor de trifosfato de inositol
(IP3R). O receptor activado pelo trifosfato de inositol (IP3), um dos 2os
mensageiros resultantes da activao da fosfolipase C (PLC), que, uma vez
activado, funciona como canal de Ca2+, permitindo que os ies sejam
libertados no citosol.

35
Brigitta Cismasiu | FMUL 2010/2011

O PAPEL DA CALMODULINA NA FUNO REGULADORA DO CLCIO


Na maioria dos casos, necessria uma protena ligante que sirva de
intermedirio entre Ca2+ e as protenas funcionais que este regula.

Um exemplo dessas protenas intermedirias a calmodulina (dmero com 4 stios


de ligao para Ca2+). Quando todos os centros de ligao esto ocupados, expe
um domnio hidrofbico aderente que se dispe ao redor de regies especficas
das protenas alvo de regulao, induzindo alteraes de conformao.

O complexo Ca2+-calmodulina estimula uma vasta gama de enzimas, canais inicos


e outras protenas-alvo, como, por exemplo, as protenas cinases dependentes de
calmodulina (CaM cinases).

As CaM cinases fosforilam muitas protenas diferentes e esto envolvidas na


regulao do metabolismo energtico, da permeabilidade inica e da sntese e
libertao de neurotransmissores.

Efeitos Biolgicos da Radiao

O espectro de radiao emitido pelo Sol muito amplo em comprimento de onda.


Assim, muitos organismos vivos adaptaram-se e tiram partido da luz visvel. Uma
dessas adaptaes consiste no processo de viso.

No mundo biolgico, o
aproveitamento de largas gamas de
energia de radiao ditado pela
capacidade das molculas em sofrer
alteraes controladas quando
interactuam com a radiao.
Enquanto a radiao gama, por
exemplo, pode quebrar ligaes

36
Brigitta Cismasiu | FMUL 2010/2011

qumicas indiscriminadamente, as ondas rdios no tm qualquer efeito marcante


sobre as molculas. Compreensivelmente, ambas no so utilizadas pelos seres
vivos. Na zona de energia intermdia, as molculas podem ter os seus electres
excitados e mudar de conformao em consequncia disso. esta a chave do
processo de viso. A radiao infravermelha excita alguns terminais nervosos e a
sua aco biolgica uma percepo de calor.

Viso
A mudana de conformao (cis-trans) do retinal desencadeia o processo
molecular de viso nas membranas das clulas cones e bastonetes dos olhos. O
retinal est ligado protena rodopsina.

Os raios ultravioleta esto na fronteira entre a radiao visvel e a radiao que


causa quebra de ligaes covalentes (dita ionizante). Embora no extremamente
ionizantes, podem, sobretudo em doses elevadas, quebrar ligaes e provocar
danos em tecidos, inclusive ao nvel de cidos nucleicos, com risco muito
acrescido de cancro.

A derme e os olhos, estando muito expostos,


requerem especial ateno. Os culos escuros
e os cremes protectores solares so as formas
de defesa mais simples e comuns. Ambos se
baseiam num princpio comum: interpor entre
a fonte de radiao (sol) e os tecidos
(olhos/derme) molculas que absorvem a
radiao UV, no a deixando chegar aos
tecidos. As lentes so compostas por materiais
vtreos ou plsticos que absorvem radiao UV,
enquanto os protectores solares se baseiam
em emulses de molculas aromticas, como derivados de benzofenonas e
benzoatos. Tambm so por vezes usadas molculas inorgnicas.

37
Brigitta Cismasiu | FMUL 2010/2011

Aminocidos e Protenas
Aminocidos

Os aminocidos so as unidades estruturais bsicas das protenas. So


constitudos por um tomo de carbono central (carbono ), ligado a um grupo
amina, um grupo de carboxilo, um tomo de hidrognio, e um grupo R (cadeia
lateral).

Estereoismeros
A quiralidade do carbono origina uma configurao tetradrica assimtrica e,
portanto, existem 2 enantimeros (isomerismo ptico):

L-aminocido D-aminocido

Com a ligao CH perpendicular ao plano do papel, o tomo de hidrognio para


trs do mesmo plano e o grupo R para cima, um aminocido com o grupo amina
projectado para a esquerda tem configurao L e um aminocido com o grupo
amina projectado para a direita tem configurao D.

As definies L e D esto relacionadas com a capacidade de desviar a luz


polarizada.

Apenas os L-aminocidos so constituintes das protenas.

Propriedades Inicas
Os aminocidos em soluo de pH neutro existem predominantemente como ies
dipolares ou zwitteries. Na forma dipolar, o grupo amina est protonado (NH3+)
e o grupo carboxilo est desprotonado (COO-). O estado de ionizao de um
aminocido varia de acordo
com o pH. Numa soluo
cida, o grupo amina est
protonado (NH3+) e grupo
carboxilo no est dissociado
(COOH). medida que o pH
aumenta, o cido carboxlico
o primeiro grupo a ceder
um proto (pKa=2). Numa
soluo bsica, o grupo amina
protonado grupo perde um
proto (NH2).

38
Brigitta Cismasiu | FMUL 2010/2011

Ponto Isoelctrico
O ponto isoelctrico o valor de pH para o qual o aminocido atinge a sua
concentrao mxima sob a forma de zwitterio e as formas catinica e aninica
esto em igual concentrao. Como tal, a carga total nula e, se for sujeito a um
campo elctrico, o aminocido no migra.

Numa electroforese, dependendo da relao entre o pH da soluo-tampo e do


ponto isoelctrico do aminocido, este mover-se- em direco a um dos plos:
Ctodo (plo negativo), se estiver carregado positivamente, forma cida,
logo ;
nodo (plo positivo), se estiver carregado negativamente, forma bsica,
logo ;
ou permanece estacionrio, se a sua carga for nula, logo .

Assim, quanto maior a diferena entre pH e PI, maior vai ser a migrao.

Ligao Peptdica
Os aminocidos podem ser polimerizados para formar cadeias, atravs de uma
reaco de desidratao. A ligao resultante denominada ligao peptdica.

uma ligao covalente com carcter intermdio entre simples e dupla, no


ocorrendo rotao em torno desta ligao, uma vez que os tomos ligados ao
carbono-azoto esto todos no mesmo plano. O grupo formado pela ligao o
grupo amida.

Numa cadeia polipeptdica,


existe uma cadeia principal,
regular e repetida, e uma cadeia
varivel, que corresponde s
cadeias laterais, diferentes
consoante os tipos de
aminocidos.

As cadeias polipeptdicas apresentam orientao, sendo o grupo amina a


extremidade inicial e o grupo carboxilo a extremidade final.

39
Brigitta Cismasiu | FMUL 2010/2011

Protenas

Uma protena uma macromolcula de elevada massa molecular, sendo


constituda por uma ou mais cadeias polipeptdicas e apresentando conformao
tridimensional definida e especfica (devido aos grupos R). formada por mais de
50 resduos de aminocidos unidos entre si por ligaes peptdicas.

Estrutura Primria
Sequncia linear de resduos de aminocidos unidos por ligaes peptdicas e est
relacionada com o tipo de aminocidos e a sua sequncia na cadeia polipeptdica.
Determina a estrutura da protena, o seu mecanismo de aco ao nvel molecular
e a sua relao com outras protenas.

um tipo de organizao muito instvel, que se rompe com facilidade, e que


permite que a cadeia polipeptdica se dobre em estruturas tridimensionais
especficas, que vo conferir protena as suas propriedades qumicas e
fisiolgicas.

As ligaes predominantes so ligaes covalentes (peptdicas), embora alguns


autores considerem a existncia de pontes dissulfito.

Estrutura Secundria
Conformao que a cadeia polipeptdica adopta no espao, de modo a conferir-
lhe maior organizao estrutural. Dependendo dos valores dos ngulos de rotao
das ligaes psi ( entre C e C do grupo carboxilo) e phi ( entre C e N do
grupo amina), uma cadeia polipeptdica pode apresentar os seguintes arranjos
espaciais:
Estrutura em hlice : quando se estabelecem ligaes por pontes de H
entre o grupo aceitador de protes (O do grupo carboxilo) de um
aminocido e o grupo doador de protes (NH do grupo amina) de outro
aminocido. As cadeias laterais orientam-se para o exterior da hlice e o
nmero de aminocidos por volta 3,6. Normalmente, a espiral curva para
a direita por ser mais estvel. Esta estrutura confere s protenas
estabilidade e forma alongada, sendo responsvel pela formao de
protenas fibrosas.
Estrutura em folha pregueada: vrias cadeias polipeptdicas estendem-se
de modo regular e semelhante a pregas e estabelecem ligaes por pontes
de H entre o O do grupo carboxilo de um aminocido e o H do grupo amina
de outro aminocido de cadeias adjacentes (alinhadas de forma paralela ou
antiparalela). As cadeias laterais so projectadas para cima e para baixo da
estrutura.

As ligaes predominantes so as ligaes no covalentes as ligaes por pontes


de H e as interaces hidrfobas.

Os grupos R no so considerados neste nvel de organizao estrutural.

40
Brigitta Cismasiu | FMUL 2010/2011

Estrutura Terciria
Estrutura tridimensional de uma protena, incluindo as interaces entre regies
distantes da cadeia e as interaces entre as cadeias laterais. As cadeias laterais
hidrfobas orientam-se para o interior da estrutura proteica, longe da gua,
enquanto que as cadeias laterias hidrfilas situam-se superfcie da estrutura
proteica, contactando com a gua.

As ligaes predominantes so as ligaes no-covalentes interaces das


cadeias laterais por pontes de H, interaces hidrfobas, interaces
electrostticas (inicas), ligaes de Van der Waals e ligaes por pontes
dissulfito (intercadeia e intracadeia; tm um papel importante na estabilizao
das estruturas proteicas).

Estrutura Quaternria
Associao no-covalente de subunidades polipeptdicas numa protena
oligomtrica (multicadeia).

Estrutura Nativa
De acordo com a sequncia de aminocidos (estrutura primria), as protenas
assumem uma nica conformao (secundria, terciria ou quaternria),
determinando a sua estrutura nativa. A passagem da estrutura primria
estrutura nativa ocorre espontaneamente atravs do estabelecimento de ligaes
no-covalentes. Esta estrutura nativa assegura a maior estabilidade e o nvel mais
baixo de energia cintica cadeia polipeptdica em determinadas condies de pH
e temperatura.

Desnaturao
A desnaturao proteica consiste na alterao da conformao espacial das
protenas, por quebra de ligaes de menor energia (ligaes no-covalentes),
levando perda da estrutura nativa e, consequentemente, da funo.

A mudana na estabilidade de uma ligao no-covalente pode ser causada por


mudanas no pH, na fora inica ou na temperatura. A ligao de grupos
prostticos*, cofactores e substratos tambm pode afectar essa estabilidade.

As ligaes peptdicas no so afectadas, sendo a desnaturao um processo


reversvel em determinadas condies de pH e temperatura (a cadeia pode
reorganizar-se e restabelecer a estrutura nativa inicial).

* Grupo prosttico: grupo no proteico (orgnico ou inorgnico) que se liga


covalentemente a protenas, originando uma protena conjugada ou
heteroprotena. Consoante a natureza do grupo prosttico, as heteroprotenas
so designadas por lipoprotenas, glicoprotenas, fosfoprotenas, nucleoprotenas,
etc.

41
Brigitta Cismasiu | FMUL 2010/2011

Colagnio
a protena mais abundante nos mamferos e o principal componente fibroso da
pele, ossos, tendes, cartilagens e dentes. Esta protena extracelular constituda
por trs cadeias polipeptdicas helicoidais, cada uma com cerca de 1000 resduos.
A glicina aparece a cada terceiro resduo da cadeia e a sequncia glicina-prolina-
hidroxiprolina* frequente.

A hlice do colagnio tem propriedades diferentes das da hlice . As ligaes de


H intracadeia esto ausentes. Em vez disso, a hlice estabilizada pela repulso
estrica dos anis da prolina e hidroxiprolina. As trs cadeias enrolam-se entre si,
formando uma superhlice estabilizada por ligaes de H entre cadeias. As
ligaes de hidrognio formam-se entre os grupos NH dos resduos de glicina e os
grupos CO dos resduos das cadeias laterais. Os grupos hidroxilo dos resduos da
hidroxiprolina tambm participam nas ligaes de H, sendo que a sua ausncia
causa escorbuto*.

O interior da cadeia em tripla hlice muito estreito e s a glicina cabe nessa


posio interior, razo pela qual necessria uma glicina em cada terceira
posio. Os resduos de aminocidos de ambos os lados da glicina esto
localizados na parte exterior da superhlice.

Estrutura primria: Gly-Pro-Hyp (repete-se)


Estrutura secundria: -hlice
Estrutura quaternria: 3 hlices formam superhlice

* A hidroxiprolina, que crtica para a estabilidade do colagnio, criada pela


modificao de aminocidos da prolina aps a construo da cadeia de colagnio.
Essa reaco requer vitamina C para facilitar a adio de oxignio. Uma vez que o
nosso organismo no sintetiza a vitamina C, caso no a ingeramos o suficiente
atravs da dieta, a sua deficincia diminui a produo de hidroxiprolina e impede
a construo de um novo colagnio, causando escorbuto. Os sintomas do
escorbuto perda de dentes e hematomas fceis so causados pela falta de
colagnio para reparar o desgaste dos tecidos.

Insulina
A insulina produzida nas clulas de Langerhans a
partir da molcula precursora proinsulina, que
consiste numa cadeia simples polipeptdica de 86
aminocidos e trs pontes dissulfito intercadeia.
transformada em insulina biologicamente activa por
clivagem proteoltica aps a sua secreo. A
proinsulina clivada em dois locais: entre os resduos
30 e 31 e entre os resduos 65 e 66. Esta reaco
liberta duas molculas: o pptido C, um fragmento de
35 resduos, e a insulina, que consiste em duas
cadeias polipeptdicas (A e B) de 21 aminocidos e 30
aminocidos, respectivamente, covalentemente
ligados por duas pontes dissulfito e a cadeia A
contendo uma ponte dissulfito intracadeia. O pptido

42
Brigitta Cismasiu | FMUL 2010/2011

C posteriormente processado por proteases, que hidrolisam um dipptido das


extremidades N e C. O pptido C modificado segregado com a insulina.

Correlaes Clnicas

Anemia Falciforme
A hemoglobina (HbA) um tetrmero formado por 2 cadeias e 2 cadeias . Na
hemoglobina S (HbS) ocorre substituio no conservativa (troca de um
aminocido por outro de polaridade diferente), na sexta posio da cadeia , de
glutamato por valina.

O grupo polar (hidrfilo) da cadeia lateral do glutamato substitudo por um


grupo apolar (hidrfobo) da valina. Assim, a valina vai fazer com que a HbS
estabelea interaces hidrfobas entre valinas de outras molculas de HbS.
Consequentemente, ocorre uma precipitao da Hb dentro dos eritrcitos, o que
leva a que estes assumam uma configurao em forma de foice, resultando numa
elevada taxa de hemlise por perda de elasticidade dos eritrcitos. Assim,
durante a circulao desses eritrcitos deformados, os capilares vo ficar
obstrudos.

Envenenamento por Insulina


O envenenamento por insulina (hiperinsulinmia sbita) pode resultar em
sintomas severos, incluindo coma hipoglicmico, comprometimento neurolgico e
morte.

Existem alguns casos de suicdio por este mtodo, que mais frequente em
pessoal mdico, farmacutico, parentes de diabticos e pacientes diabticos, pois
tm acesso fcil aos medicamentos antidiabticos.

Sintomas:
Tonturas;
Perda de sentidos;
Leses cerebrais;
Hipocalmia ( K)*;
Hipofosfatmia ( P);
Hipomagnesmia ( Mg).

*A insulina actua nas Na+/K+ ATPases, pelo que h uma descida de K no meio
extracelular.

Alm do tipo de insulina, a gravidade e a velocidade do processo depende:


Tecido onde injectada (no tecido adiposo, o processo acelerado);
Profundidade da injeco;
Alimentao em fase crtica;
Consumo de etanol (inibe a gluconeognese).

43
Brigitta Cismasiu | FMUL 2010/2011

Tratamento:
1. Procurar sintomas dores de cabea, sensao de fraqueza, sudorese e
aumento do apetite. Em fases mais avanadas, verifica-se sonolncia,
dificuldade de concentrao e convulses. Eventualmente, coma
diabtico e morte;
2. Ingesto de acar o envenenamento por insulina provoca hipoglicmia
sbita. A forma mais rpida de a tratar o aumento da glicmia (acar,
sumos, chocolate, etc.);
3. Medicao de glucose glucose instantnea (pasta comestvel), injeco
de glucose ou infuso intravenosa de glucose:
4. Monitorizao dos nveis sanguneos de glucose.

Outras abordagens teraputicas:


Hemodilise (melhora a eliminao, mas no encurtara durao da aco
da insulina, que depende da taxa de absoro e no de eliminao);
Administrao de glucagina (a quantidade de glicognio armazenado no
sangue pode no ser suficiente para repor os nveis de glucose);
Exciso do tecido injectado.

44
Brigitta Cismasiu | FMUL 2010/2011

Lpidos, Membranas e Transporte


Os lpidos desempenham uma grande variedade de papis nas clulas. Eles so a
principal forma de armazenamento de energia e os constituintes fundamentais
das membranas celulares.

Lpidos especializados actuam como pigmentos (retinal, caroteno), cofactores


(vitamina K), detergentes (cidos biliares), transportadores (dolicol), hormonas
(derivados da vitamina D, hormonas sexuais), mensageiros extracelulares e
intracelulares (eicosanides, derivados do fosfatidilinositol) e ncoras lipdicas
para ligao de protenas membranares (cidos gordos ligados covalentemente,
grupos prenil, fosfatidilinositol).

CIDOS GORDOS
Constitudos por uma cadeia linear, no ramificada, de tomos de carbono, com
um grupo carboxilo numa das extremidades (hidrfila) e uma cadeia carbonada na
outra extremidade (hidrfoba). Os cidos gordos so saturados se s possurem
ligaes simples. Caso contrrio, so insaturados, sendo que as ligaes duplas
baixam o ponto de fuso e aumentam a fluidez da membrana e as trocas atravs
dela.

GLICEROL
Molcula constituda por trs tomos de carbono com um grupo hidroxilo ligado a
cada um.

ACILGLIGERIS
Uma molcula de glicerol ligada covalentemente (ligao ster) a um, dois ou trs
cidos gordos (monoacilglicerol, diacilglicerol e triacilglicerol, respectivamente).

Importncia da Hidrofobicidade dos Triacilgliceris


Os TAG e outros complexos lipdicos tm solubilidade limitada na gua, uma vez
que a longa cadeia carbonada constituinte dos cidos gordos no estabelece
pontes de hidrognio. Consequentemente, tendem a associar-se entre si ou com
outros grupos hidrfobos, tal como os esteris e as cadeias laterais dos
aminocidos. Esta insolubilidade na gua essencial para o armazenamento dos
TAG e para a formao de membranas biolgicas.

Os TAG formam reservas de energia muito mais eficientes do que o glicognio,


uma vez que da sua oxidao completa so formadas 2 vezes mais ATP do que a
partir da mesma massa de glicognio. Por outro lado, os TAG so armazenados
sem molculas de gua associadas, enquanto o glicognio, sendo hidrfilo, liga o
dobro da sua massa de gua.

Ao contrrio das reservas de glicognio e aminocidos, que so limitadas, os TAG


conseguem armazenar uma quantidade quase ilimitada, dependendo do consumo
de calorias.

45
Brigitta Cismasiu | FMUL 2010/2011

Tipos mais Comuns de Lpidos

Associaes Fosfolipdicas Possveis em Meio Aquoso

Colesterol
O colesterol uma molcula anfiptica, com uma cabea polar (o grupo hidroxilo
em C-3) e uma zona apolar (o ncleo esteride e a cadeia lateral carbonada em C-
17). Por vezes, o grupo OH pode ser substitudo por um cido gordo, ficando a
molcula apolar.

O colesterol especialmente abundante nas estruturas mielinizadas do sistema


nervoso central, mas est presente em pequenas quantidades na membrana
externa da mitocndria. No plasma, a maior parte do colesterol est sob a forma
esterificada, mas na membrana celular encontra-se na forma livre.

A excreo do colesterol feita atravs do fgado e da vescula biliar pelo


intestino, sob a forma de cidos biliares. O colesterol o percursor imediato dos
cidos biliares; estes facilitam a absoro de TAG provenientes da dieta e de
vitaminas lipossolveis.

O colesterol ainda percursor de vrias hormonas esterides, como a


progesterona, a testosterona, o estradiol e o cortisol.

46
Brigitta Cismasiu | FMUL 2010/2011

Estrutura das Membranas Biolgicas


As membranas so extremamente dinmicas e assimtricas na sua composio (a
camada intracelular tem carga negativa e a extracelular neutra), de modo a
formar domnios de especializao, normalmente associados a protenas, e variam
a sua composio e distribuio de organelo para organelo e de clula para clula.

A estrutura e flexibilidade da bicamada lipdica dependem da temperatura e do


tipo de lpidos constituintes. A uma temperatura relativamente baixa, os lpidos
da bicamada esto numa fase semi-slida de gel, na qual as molculas lipdicas
esto fortemente unidas: fase cristalina. A temperaturas relativamente elevadas,
as cadeias carbonadas dos cidos gordos esto em constante movimento
(rotaes em torno nas ligaes C-C): fase fluida. Nas membranas, o colesterol
orienta-se paralelamente aos cidos gordos dos fosfolpidos e o grupo hidroxilo
interage com os grupos polares dos mesmos. Assim, aumenta a rigidez e a
organizao das camadas internas, conferindo membrana uma estrutura de
mosaico fluido.

47
Brigitta Cismasiu | FMUL 2010/2011

H uma distribuio heterognea de lpidos e protenas na membrana, formando


microdomnios, os rafts lipdicos, enriquecidos por colesterol e esfingolpidos. A
funo dos rafts segregar e concentrar protenas especficas para facilitar a sua
actividade. A camada externa dos rafts tem uma maior concentrao de
ceramidas e glicoesfingolpidos, criando reas com maior espessura. A camada
interna contm uma grande quantidade de fosfolpidos com cadeias carbonadas
saturadas que permitem um maior empacotamento. Assim, os lpidos dos rafts
esto mais ordenados e empacotados do que a bicamada envolvente. Alm disso,
as interaces electrostticas entre as cabeas polares, as interaces hidrfobas
das molculas de colesterol com alguns fosfolpidos e glicolpidos e as interaces
lipoproteicas condicionam o movimento dos desses lpidos.

Algumas protenas membranares esto exclusivamente associadas aos rafts,


outras associam-se e desassociam-se e outras esto apenas presentem nas
restantes pores da membrana.

Transporte Transmembranar

A entrada de um soluto (genericamente: ies atmicos, nutrientes e outras


molculas necessrias ao metabolismo celular) pode seguir vrias vias,
correspondendo s propriedades qumicas e fsicas do soluto e necessidade de
regular as trocas de matrias e energia da clula com o exterior.

Potencial Qumico Potencial Elctrico

48
Brigitta Cismasiu | FMUL 2010/2011

Transporte Mediado por Protenas


Para atravessar a membrana, um soluto
polar ou carregado tem de quebrar as
ligaes das molculas de gua que o
hidratam e difundir-se atravs do solvente
no qual pouco solvel (lipdico),
processo que requer uma grande
quantidade de energia.

A passagem pela membrana cria um


estado intermdio de passagem
transmembranar que requer uma
quantidade considervel de energia (G),
de tal ordem que uma bicamada lipdica
pura totalmente impermevel.

As permeases so protenas membranares


que baixam a energia de activao, ou
seja, oferecem uma via alternativa a
solutos especficos atravs da membrana.

A energia gasta reposta quando o composto atravessa a membrana e


reidratado.

Transporte Atravs da Membrana

49
Brigitta Cismasiu | FMUL 2010/2011

Trs Classes de Transporte


Uniporte: o transportador transloca apenas um substrato;
Simporte: o transportador transloca os dois substratos na mesma direco;
Antiporte: o transportador transloca os dois substratos em direces
opostas.

Transporte Activo
Primrio: a acumulao de soluto acoplada a uma reaco exergnica, tal
como a converso do ATP em ADP e Pi;
Secundrio: o transporte endergnico de um soluto acoplado ao
transporte exergnico de um soluto diferente, que foi previamente
bombeado por transporte activo primrio.

Transporte de Glucose nas Clulas Epiteliais Intestinais


A glucose co-transportada com o
Na+, atravs da membrana plasmtica
apical, para o citosol da clula
epitelial. A glucose atravessa a clula
at superfcie basal, onde passa
para o sangue atravs do GLUT2, um
transportador de glucose passivo. A
Na+/K+ ATPase continua a bombear
Na+ para o exterior, de modo a
conservar o gradiente de Na+ que
mantm a captao de glucose.

50
Brigitta Cismasiu | FMUL 2010/2011

Glcidos
Monossacridos

Aldedos ou cetonas de cadeia no ramificada com grupos hidroxilo em cada um


dos carbonos que no formam carbonilos. Geralmente tm como formula
molecular (CH2O)n.

Os monossacridos mais simples so as trioses:

A configurao absoluta dos monossacridos determinada pela estereoqumica


do tomo de carbono assimtrico mais afastado do carbono do grupo carbonilo,
ou seja, pela configurao do carbono quiral.

Com base na posio do OH do carbono quiral, um monossacrido D se o OH se


projectar para a direita e L se se projectar para a esquerda.

L-glucose D-glucose

Todas as oses que intervm no metabolismo apresentam configurao D.

Em soluo aquosa, aldedos e cetonas reagem com grupos hidroxilo para formar
hemiacetais e hemicetais, respectivamente.

51
Brigitta Cismasiu | FMUL 2010/2011

Ciclizao
As pentoses e as hexoses podem formar hemiacetais e hemicetais cclicos.

Numa aldohexose (ex.: glucose), o aldedo em C1 na cadeia aberta reage com o


hidroxilo em C5 para formar um hemiacetal intramolecular. O hemiacetal cclico
resultante, um anel de 6, chamado piranose. Esta ciclizao ocorre, aps
hidratao, por eliminao de uma molcula de gua entre o OH de C1 e o OH de
C5.

Numa cetohexose (ex.: frutose), a cetona em C2 na cadeia aberta reage com o


hidroxilo em C6, formando um anel hemicetal cclico de 6 (piranose), ou com o
hidroxilo em C5, formando um anel hemicetal cclico de 5 (furanose).

Aps ocorrerem ciclizaes, gerado um novo carbono quiral (o carbono


carbonil), designado de carbono anomrico, que possibilita a existncia de 2
formas ismeras, os anmeros. Segundo Haworth, o ismero se o grupo OH e
o grupo CH2OH nos dois tomos de carbono ligados pela ponte osdica estiveram
em trans um em relao ao outro e se estiverem em cis.

52
Brigitta Cismasiu | FMUL 2010/2011

Os monossacridos existem sob a forma de anel 99% das vezes em soluo


aquosa, uma vez que uma forma mais estvel. Assim, so facilmente
reconhecidos por enzimas, facilitam a formao de ligaes O-glicosdicas, tornam
a polimerizao mais fcil e, como so solveis em gua, tornam o seu transporte
mais fcil.

Ligaes Glicosdicas
Os monossacridos podem reagir com vrios tipos de molculas (outros
monossacridos, lcoois, aminas, protenas, etc.) para formar monossacridos
modificados, ligando-se a estas covalentemente, por ligaes glicosdicas.

A ligao formada entre o tomo de carbono anomrico de uma ose e o tomo de


oxignio do grupo hidroxilo de outra molcula chamada de ligao O-glicosdica
e o produto pode ser um dissacrido ou um glicosdeo.

Se o tomo de carbono anomrico de uma ose estiver ligado ao tomo de azoto


de uma amina, origina uma ligao N-glicosdica e o produto denominado
glicosilamina (ex.: nuclesidos como a adenina, citosina, adenosina).

Oligossacridos

Polmeros at 8 monossacridos.

Dissacridos
So os oligossacridos mais simples. Consistem com 2 molculas de
monossacridos ligadas covalentemente por uma ligao O-glicosdica.

53
Brigitta Cismasiu | FMUL 2010/2011

Polissacridos

Polmeros com mais de 8 monossacridos.

Homopolissacridos
Contm um nico tipo de monmeros. Alguns homopolissacridos servem como
formas de armazenamento de monossacridos que so utilizados como
combustvel (amido e glicognio), enquanto outros servem como elementos
estruturais em paredes celulares vegetais e exosqueletos animais (celulose e
quitina).

Heteropolissacridos
Contm dois ou mais tipos diferentes de monmeros. Os heteropolissacridos
fornecem proteco, forma e suporte extracelular a clulas, tecidos e rgos.

Glicognio
Homopolmero de glucose ramificado, cujos resduos esto ligados,
maioritariamente, por ligaes -1,4-glicosdicas. Os ramos provm de ligaes -
1,6-glicosdicas.

armazenado no fgado e no msculo como reserva de glucose, para manuteno


dos nveis de glucose no sangue e produo de ATP, respectivamente.

Variao dos nveis de glicognio no fgado durante o jejum e as refeies:

54
Brigitta Cismasiu | FMUL 2010/2011

Aumentam aps as refeies e depois decrescem lentamente, uma vez que o


glicognio usado para manter os nveis de glucose no sangue.

O exerccio mobiliza o glicognio dos msculos para a formao de ATP.


As fibras musculares vermelhas so muito vascularizadas, contendo uma
grande quantidade de mioglobina e mitocndrias. O glicognio mobilizado
nestas clulas convertido a piruvato, sendo este convertido a CO2 e H2O
no ciclo TCA;
As fibras musculares brancas contm menos mioglobina e mitocndrias. A
glicogenlise nestas clulas apenas fornece substrato para a gliclise, com o
produto final sendo lactato.

Destino do glicognio degradado no fgado e nos msculos:

Porqu armazenar glucose como glicognio? Porque no armazenar como TAG?


1. Os cidos gordos no conseguem ser libertados dos TAG to rapidamente
quanto a glucose do glicognio;
2. Os TAG no podem ser usados como fonte de energia na ausncia de
oxignio;
3. Os TAG no podem ser convertidos a glucose para manter os nveis de
glucose no sangue necessrios ao crebro.

Proteoglicanos
So macromolculas da superfcie celular ou da matriz extracelular nas quais
uma ou mais cadeias de glicosaminoglicanos (um ou dois monossacridos so
cido urnico e o outro uma N-acetilglicosamina, derivada da glicosamina,
derivada da glucose) esto ligados covalentemente a uma membrana ou protena
segregada.

55
Brigitta Cismasiu | FMUL 2010/2011

Funes:
So os principais compostos dos tecidos conjuntivos, uma vez que as
ligaes no-covalentes com outros proteoglicanos, protenas ou
glicosaminoglicanos oferecem resistncia e resilincia;
Constituem a cartilagem, porque os glicosaminoglicanos so muito
hidrfilos, logo, rodados por molculas de gua.

Glicoprotenas
So oligossacridos ligados covalentemente a uma protena. So encontradas no
lado extracelular da membrana plasmtica, na matriz extracelular e no sangue.
Dentro das clulas, existem em organelos especficos como o complexo de Golgi,
grnulos secretores e lisossomas.

Funes:
As regies oligosacridas so ricas em informao, formando locais
altamente especficos de reconhecimento e grande afinidade de ligao a
outras protenas.

Vrus Influenza

Alguns vrus conseguem entrar nas clulas de um hospedeiro especfico, aderindo


aos glcidos da superfcie celular.

Por exemplo, o vrus influenza reconhece os resduos de cido silico presentes


em glicoprotenas da superfcie celular.

A hemaglutinina (HA), protena da superfcie


viral, liga-se a esses resduos, permitindo a
entrada do vrus na clula hospedeira, por
endocitose induzida. O vrus , assim, envolvido
num endossoma. Nesse pH cido, ocorre uma
mudana conformacional na protena HA,
expondo uma sequncia chamada pptido de
fuso, que vai permitir a penetrao da HA na
membrana endossomal. Uma vez que a fuso
se complete, ocorre a libertao do contedo
viral no citoplasma da clula hospedeira.

Uma outra protena viral, neuraminidase (NA), cliva as ligaes glicosdicas dos
resduos de cido silico, permitindo que o vrus seja libertado a partir da clula
hospedeira e infecte clulas vizinhas. Inibidores da NA, como o oseltamivir
(Tamiflu) e o zanamivir (Relenza), so importantes agentes antigripais.

Quando dois vrus diferentes infectam a mesma clula, aparecem novas estirpes
do vrus influenza. As cpias dos seus RNA misturam-se e d-se uma combinao
gentica das estirpes iniciais. Desta forma, uma estirpe do vrus influenza animal
ou aviria pode ganhar genes que lhe confiram a capacidade de difundir-se com
maior capacidade nos humanos.

56
Brigitta Cismasiu | FMUL 2010/2011

Nucletidos
Os cidos nucleicos so macromolculas presentes em todas as clulas, tanto na
forma livre como combinados com protenas para formar nucleoprotenas. So
constitudos por nucletidos e desempenham um papel fundamental na
informao gentica. Os dois tipos bsicos de cidos nucleicos so o cido
desoxirribonucleico (DNA) e o cido ribonucleico (RNA).

Funes

Os nucletidos desempenham inmeras funes bioqumicas:


Monmeros para sntese de cidos nucleicos DNA ou RNA;
Transportadores de energia qumica, como ATP e GTP;
+
Componentes de cofactores, como NAD , FAD, coenzima A, etc.;
Mediadores fisiolgicos, como cAMP e cGMP;
Precursores de 5 cap de mRNAs (GTP), tetrahidrobiopterina (GTP);
Intermedirios activados, como UDP-glucose, S-adenosilmetionina, etc.;
Efectores alostricos.

Estrutura

Um nucletido uma associao entre uma base azotada, uma pentose e um


grupo fosfato. O DNA constitudo por desoxirribonucletidos e o RNA por
ribonucletidos.

Bases Azotadas
As bases azotadas so derivadas das estruturas pirimidina e purina, sendo
designadas de bases pirimdicas ou purnicas, respectivamente.

Tanto o DNA e como o RNA tm as bases purinas adenina (A) e guanina (G) e a
pirimidina citosina (C). A outra pirimidina a timina (T) no DNA e o uracilo (U) no
RNA. Existem outras bases purinas, como a hipoxantina e a xantina, que se
encontram apenas em alguns cidos nucleicos (nomeadamente no tRNA) e em
pequenas quantidades, e o cido rico, a forma mais oxidada das purinas.

57
Brigitta Cismasiu | FMUL 2010/2011

Pentoses
Os cidos nucleicos tm dois tipos de pentoses: a desoxirribose (2-desoxi-D-
ribose) no DNA e a ribose (D-ribose) no RNA. Ambos os tipos de pentoses esto na
forma de -furanose.

Nuclesidos
Os nuclesidos so compostos formados pela ligao entre uma base azotada e
uma pentose, atravs de uma ligao glicosdica. A base azotada est
covalentemente unida, pelo N1 nas pirimidinas e N9 nas purinas, por uma ligao
N--glicosil, ao C1 da pentose. A ligao N--glicosil formada pela remoo do
grupo hidroxilo da pentose e do hidrognio da base.

Os nuclesidos comuns so denominados desoxiadenosina, desoxiguanosina,


desoxitimidina e desoxicitidina, no DNA, e adenosina, guanosina, uridina e
citidina, no RNA. O nuclesido formado por hipoxantina e ribose tem o nome de
inosina.

Nucletidos
Um nucletido formado quando um grupo fosfato esterificado com o grupo
OH da pentose de um nuclesido. O anel da ribose tem trs grupos OH disponveis
para a esterificao e o da desoxirribose tem apenas dois. Um composto formado
pela ligao de um grupo fosfato ao carbono 5 da pentose de um nuclesidos
denominado nuclesido 5-monofosfato (dNMP ou NMP).

A ligao de um segundo grupo fosfato ao fosfato de um NMP origina um


nuclesido 5-difosfato (dNDP ou NDP). Se um terceiro fosfato se ligar ao
segundo forma-se um nuclesido 5-trifosfato (dNTP ou NTP).

58
Brigitta Cismasiu | FMUL 2010/2011

Tanto no DNA como no RNA, os nucletidos esto ligados por ligaes


fosfodister 3-5, isto , ligaes covalentes entre o grupo hidroxilo do C3 da
pentose de um nucletido e o grupo fosfato ligado ao hidroxilo do C5 da pentose
do outro nucletido. Na formao da ligao fosfodister libertado um grupo
pirofosfato rico em energia.

As cadeias de DNA e de RNA possuem estrutura primria idntica. Embora as


bases azotadas no participem no encadeamento, a sequncia de bases
caracterstica de um cido nucleico, pois nela que reside a informao gentica.

Nucletidos Cclicos
Nucletidos monofosfatados em que o cido fosfrico esterificado por dois dos
grupos hidroxilo disponveis da ribose so encontrados em todas as clulas. A
formao de duas ligaes ster com um fosfato resulta numa estrutura cclica. O
3,5-AMP cclico (cAMP) e o seu anlogo 3,5-GMP cclico (cGMP) so importantes
reguladores do metabolismo celular.

59
Brigitta Cismasiu | FMUL 2010/2011

Nuclesidos 5-Trifosfato
Os NTPs so agentes indispensveis no metabolismo, porque as ligaes que
possuem entre os grupos fosfato so uma fonte essencial de energia qumica para
as reaces biolgicas. O ATP a moeda energtica da clula, o GTP a
principal fonte de energia para a sntese proteica, o CTP um metabolito
essencial na sntese de fosfolpidos e o UTP forma intermedirios activados com
oses que passam a servir como substrato na biossntese de glcidos complexos e
oligossacridos.

A evoluo do metabolismo levou especializao de cada um destes quatro


NTPs a cada um dos ramos principais do metabolismo. Alm disso, os quatro NTPs
e os seus homlogos dNTPs so os substratos para a sntese de cidos nucleicos.

cidos Nucleicos

As duas principais classes de cidos nucleicos so o DNA e o RNA.

O DNA tem apenas uma funo biolgica, mas a mais central. As informaes
para fazer todas as macromolculas funcionais da clula so preservadas no DNA
e acessveis atravs da transcrio da informao em cpias de RNA. Nas clulas
eucariticas, as molculas de DNA organizam-se em duplas hlices, unidas por
complementaridade de bases, muito condensadas, formando cromossomas.
Existem duas cpias de cada um cromossoma no ncleo e vrias cpias nas
mitocndrias e nos cloroplastos, que codificam algumas das protenas e RNA
nicos desses organelos.

Em contraste, o RNA ocorre em cpias mltiplas e diferentes forma e tm uma


srie de funes biolgicas importantes: o RNA mensageiro (mRNA), que
transporta a informao codificada nos genes para os locais de sntese proteica,
onde traduzida numa sequncia polipeptdica, RNA ribossmico (rRNA),
constituinte dos ribossomas, RNA de transferncia (tRNA), transportador de
resduos de aminocidos para a sntese proteica, e pequenos RNA nucleares
(snRNA), importantes no processamento do mRNA.

Diferenas entre DNA e RNA


1. O DNA contm 2-desoxirribose em vez de ribose
O grupo 2-OH da ribose do RNA, ausente no DNA, faz com que a ligao
fosfodister seja mais susceptvel clivagem nucleoflica, pelo que o DNA
selectivamente vantajoso para a conter a forma hereditria da informao
gentica.

2. O DNA contm timina em vez de uracilo


A citosina desamina-se para formar uracilo, a uma taxa finita. Uma vez que
a citosina de uma cadeia de DNA emparelha com a guanina da outra cadeia,
enquanto que o uracilo emparelharia com a adenina, a converso de uma
citosina por um uracilo pode potencialmente resultar numa alterao
hereditria de um par CG por um par UA, o que originaria uma mutao no
DNA. Para evitar que essa reaco provoque mudanas na sequncia de

60
Brigitta Cismasiu | FMUL 2010/2011

nucleotdeos, um mecanismo de proofreading substitui qualquer uracilo


decorrente da desaminao, substituindo-o por uma citosina. Se o DNA
normalmente contivesse uracilo em vez de timina, este sistema de
reparao no poderia distinguir um uracilo formado por desaminao da
citosina de um correctamente emparelhado com a adenina. A timina , na
verdade, um uracilo modificado por adio de um grupo metilo.

Por causa dessas diferenas, o DNA uma forma polimrica mais estvel do que o
RNA, o que coerente com os papis que estas macromolculas assumem na
hereditariedade e na transferncia de informao.

61
Brigitta Cismasiu | FMUL 2010/2011

Introduo aos Metabolismos


Nem todos os metabolismos operam nos mesmos tecidos nem ao mesmo tempo.
Os principais metabolismos so:
Glicognese
Glicogenlise
Gluconeognese
Gliclise
Sntese de cidos gordos
Lipognese
Liplise ou -oxidao
Gliceroneognese
Oxidao de cidos gordos
Glutaminlise
Ciclo dos cidos tricarboxlicos (TCA)
Cetognese
Oxidao dos aminocidos
Sntese proteica
Protelise
Sntese de ureia

importante saber (1) quais os tecidos mais activos nestes processos, (2) quando
esto estes processos mais activos e (3) como so controlados e coordenados nos
diferentes estados metablicos e de doena.

62
Brigitta Cismasiu | FMUL 2010/2011

Gliclise
A gliclise tem 3 etapas principais:
Fase preparatria: converso de glucose em frutose 1,6-bisfosfato, com
consumo de 2 molculas de ATP;
Fase de "splitting": converso de frutose 1,6-bisfosfato a trioses-fosfato;
Fase de produo de energia: converso de gliceraldedo 3-fosfato a
lactato, com formao de ATP e NADH.

63
Brigitta Cismasiu | FMUL 2010/2011

Reaces da Gliclise
FASE PREPARATRIA
Fosforilao (endergnica)
A glucose activada, atravs da sua fosforilao em C6, produzindo glucose 6-
fosfato, com o ATP como doador do grupo fosfato. Esta reaco, irreversvel em
condies intracelulares, catalisada pela hexocinase.
Isomerizao
A converso reversvel de glucose-6-fosfato, uma aldose, em frutose 6-fosfato,
uma cetose, catalisada pela fosfoglucoisomerase.
Fosforilao (endergnica)
Transferncia de um grupo fosfato de uma molcula de ATP para a frutose 6-
fosfato, originando frutose 1,6-bisfosfato, pela fosfofrutocinase.

FASE DE "SPLITTING"
Clivagem
A frutose-bisfosfato aldolase catalisa a clivagem da frutose 1,6-bisfosfato em
trioses-fosfato: gliceraldedo 3-fosfato, uma aldose, e diidroxiacetona fosfato,
uma cetose.
Interconverso
Apenas uma das trioses-fosfato formado pela aldolase, o gliceraldedo 3-fosfato,
pode ser directamente degradado nas etapas subsequentes. O outro produto,
diidroxiacetona fosfato, rpida e reversivelmente convertido em gliceraldedo 3-
fosfato pela triose-fosfato isomerase.

FASE DE PRODUO DE ENERGIA


Oxidao
Converso do gliceraldedo 3-fosfato a 1,3-bisfosfoglicerato (1,3-BPG) pela
gliceraldedo 3-fosfato desidrogenase, com formao de NADH. A uma reaco
exergnica muito favorvel (oxidao de um aldedo a cido carboxlico)
acoplada uma reaco endergnica desfavorvel (reduo do NAD+ a NADH).
Fosforilao
A fosfoglicerato cinase transfere o grupo fosfato de alta energia carbonilo do 1,3-
BPG para o ADP, formando ATP e 3-fosfoglicerato.
Desidrogenao
A fosfoglicerato mutase catalisa um deslocamento reversvel do grupo fosfato
entre C2 e C3 do glicerato, originando 2-fosfoglicerato a partir de 3-fosfoglicerato,
numa reaco que tem Mg2+ como cofactor.
Desidratao
A enolase promove a remoo reversvel de uma molcula de H2O do 2-
fosfoglicerato, originando fosfoenolpiruvato. Esta reaco tambm tem Mg2+
como cofactor, que se liga molcula de H2O quando esta eliminada.
Hidrlise (endergnica)
Transferncia do grupo fosfato do fosfoenolpiruvato para o ADP, catalisada pela
piruvato cinase, que exige K+ e Mg2+ ou Mn2+ como cofactores, originando
piruvato. Devido converso espontnea da forma enol do piruvato para a forma
cetona, esta reaco irreversvel.

64
Brigitta Cismasiu | FMUL 2010/2011

Regulao da Gliclise

As etapas irreversveis (1, 3 e 10) so pontos fundamentais da regulao da


gliclise.

Etapa 1
A glucose uma molcula neutra e difunde-se atravs da membrana celular, mas
a fosforilao confere-lhe uma carga negativa e a membrana plasmtica
essencialmente impermevel glucose 6-fosfato. Alm disso, a converso rpida
de glucose em glucose-6-fosfato mantm baixa a concentrao intracelular de
glucose, favorecendo a entrada de glucose na clula por difuso.

HEXOCINASE
Existem 4 diferentes isoenzimas* da hexocinase expressas nos tecidos: I, II, III e IV.
As hexocinases encontradas na maioria dos tecidos (I, II e III) tm grande
afinidade para a glucose, relativamente sua concentrao no sangue.
Asseguram um constante fornecimento de energia e so inibidas
alostericamente pela glucose 6-fosfato;
A glucocinase (hexocinase IV) exclusiva do fgado e das clulas de
Langerhans (pncreas) e tem uma S0,5 muito maior, pelo que pouco
afectada pela glucose 6-fosfato. Actua velocidade mxima apenas quando
os nveis de glucose so muito elevados, fosforilando a glucose, impedindo-
a de atravessar a membrana e utilizando-a na sntese de glicognio e
lpidos.

Hexocinase: actua sempre


velocidade mxima

Glucocinase: resposta
proporcional v/[glucose]

*Enzimas que catalisam a mesma reaco, mas tm cintica e composio


qumica diferentes.

65
Brigitta Cismasiu | FMUL 2010/2011

Etapa 3
A reaco da fosfofrutocinase compromete a clula a metabolizar a glucose em
vez de a converter noutro glcido ou a armazen-la, sendo este o ponto de
regulao mais importante na via glicoltica.

FOSFOFRUTOCINASE
a enzima que controla a taxa de gliclise. O ATP um inibidor alostrico
desta enzima, sendo esse efeito reforado pela presena de citrato (intermedirio
do ciclo TCA). Na presena de altas concentraes de ATP e citrato, a
fosfofrutocinase comporta-se de forma cooperativa e o KM para a frutose
6-fosfato aumenta. Assim, quando os nveis de ATP no citoplasma esto
suficientemente elevados, a gliclise "desliga-se". Na maioria das
condies celulares, no entanto, a concentrao de ATP no varia em
intervalos grandes. A taxa de gliclise, porm, varia muito mais. O AMP
inverte a inibio provocada pelo ATP, e os nveis de AMP nas clulas
podem subir drasticamente quando os nveis de ATP diminuem, devido
aco da enzima adenilato cinase, que interconverte rapidamente ADP,
ATP e AMP, de modo a manter o equilbrio, atravs da reaco:

Etapa 10
Por cada molcula de glucose que entra na via glicoltica, so formadas duas
molculas de ATP na reaco catalisada pela piruvato cinase.

PIRUVATO CINASE
Esta enzima possui centros alostricos para vrios efectores. activada por AMP e
frutose 1,6-bisfosfato e inibida por ATP, acetil-CoA e alanina. Alm disso, a
piruvato cinase heptica regulada por modificao covalente. Hormonas como a
glucagina activam uma protena cinase dependente de cAMP, que transfere um
grupo fosfato do ATP para a enzima. A forma fosforilada da piruvato cinase mais
fortemente inibida por ATP e alanina e tem maior KM para o fosfoenolpiruvato,
pelo que, na presena de nveis fisiolgicos de fosfoenolpiruvato, a enzima
inactiva. O fosfoenolpiruvato utilizado como substrato para a sntese de glucose
na gluconeognese, em vez de ir seguir pela gliclise e pelo ciclo TCA.

66
Brigitta Cismasiu | FMUL 2010/2011

Destinos Metablicos dos Produtos da Gliclise


Alm do ATP, os produtos da gliclise so o NADH e
o piruvato. O seu processamento depende de outras
vias celulares. O NADH deve ser reciclado a NAD+ no
caso de se tornar limitante na gliclise. Esta
reciclagem pode ser feita tanto de modo aerbio
como de modo anaerbio. Assim, o que uma
determinada clula faz com o piruvato produzido na
gliclise depende, em parte, da disponibilidade de
oxignio.

Em condies aerbias, o piruvato pode ser enviado


para o ciclo TCA, onde oxidado a CO2, com a
produo de novos NADH (e FADH2). O NADH
produzido na gliclise e no ciclo TCA reoxidado a
NAD+ na mitocndria, na cadeia transportadora de
electres.

Em condies anaerbias, o piruvato produzido na


gliclise reduzido a lactato, atravs da
fermentao lctica. Essa reduo ocorre em tecidos pouco irrigados e no
msculo esqueltico de rpida contratao. Quando o msculo esqueltico
submetido a exerccio intenso, a disponibilidade de oxignio nos tecidos baixa, e o
piruvato gerado pela gliclise deixa de poder ser oxidado no ciclo TCA. Em vez
disso, o excesso de piruvato reduzido a lactato pela lactato desidrogenase. A
maior parte desse lactato deve ser conduzida para fora do msculo e
transportada para o fgado atravs da corrente sangunea, onde pode ser
ressintetizada em glucose na gluconeognese.

S h este modo de
produzir/consumir
L-lactato.

67
Brigitta Cismasiu | FMUL 2010/2011

Via de Rapoport-Luebering

Como no eritrcito no h mitocndrias, no pode haver


ciclo TCA nem cadeia respiratria. Deste modo, toda a
energia ter que ser fornecida pela gliclise.

O 2,3-BPG tem grande afinidade com a Hb desoxigenada e


menor afinidade com a Hb oxigenada. Assim, quando se
liga Hb parcialmente oxigenada, promove,
alostericamente, a libertao de todo o oxignio ligado
Hb, aumentando a capacidade de libertao do oxignio
pelos eritrcitos junto dos tecidos.

Todavia, no eritrcito h uma grande concentrao


de 2,3-BPG que se forma por um desvio da gliclise,
o ciclo de Rapoport-Luebering, podendo-se formar,
quer pela aco de uma mutase sobre o cido 1,3-
bisfosfoglicrico, quer pela aco de uma cinase
sobre o cido 3-fosfoglicrico.

Correlaes Clnicas

Anemia Hemoltica
provocada pela hemlise prematura dos eritrcitos, sem que a medula consiga
compensar essas perdas.

Os eritrcitos exigem certas condies intracelulares (diferentes das


extracelulares), que s so conseguidas com transporte activo de certas
substncias e, logo, com gasto de energia. A ausncia das enzimas da gliclise
provoca uma produo ineficaz de ATP, pelo que essas condies no so
mantidas, comprometendo a integridade da membrana e levando lise celular.

Enfarte Agudo do Miocrdio


um processo que pode levar morte de parte do msculo cardaco por falta de
transporte adequado de nutrientes e oxignio. causado por uma reduo no
fluxo sanguneo coronrio. A causa habitual desta reduo uma isqumia
(deficincia de O2) no msculo cardaco por ocluso de uma artria coronria. A
ocluso d-se pela formao de um cogulo sobre uma rea comprometida por
aterosclerose, causando estreitamentos luminais de dimenses variadas.

A obstruo de uma artria diminui o aporte de oxignio aos eritrcitos, que


deixam de produzir ATP e de controlar o seu equilbrio osmtico, levando sua
ruptura e libertao da LDH no sangue. Assim, o LDH usado como marcador da
morte dos tecidos.

68
Brigitta Cismasiu | FMUL 2010/2011

Ciclo dos cidos Tricarboxlicos


Entrada no Ciclo TCA

Em aerobiose, o NADH reoxidado atravs do sistema transportador de electres


e o piruvato completamente oxidado a CO2 e H2O, oxidao esta que ocorre no
ciclo TCA.

Descarboxilao do Piruvato a Acetil-CoA


Para entrar neste ciclo, o piruvato tem de ser previamente convertido em acetil-
CoA por descarboxilao oxidativa, que tem lugar na matriz mitocondrial.

Esta transformao processa-se em vrias etapas, catalisadas por um complexo


enzimtico designado piruvato desidrogenase, do qual fazem parte trs enzimas
(das quais a enzima central a piruvato desidrogenase, alostrica) e vrias
coenzimas.

69
Brigitta Cismasiu | FMUL 2010/2011

Reaces da Descarboxilao Oxidativa


Descarboxilao do piruvato a acetaldedo, com a coenzima pirofosfato de
tiamina ligada ao complexo enzimtico por interaces no covalentes. O Mg+ o
cofactor.
Oxidao do acetaldedo pela coenzima cido lipico na forma oxidada
(dissulfurada). Formao de um tioster. Reduo do cido lipico.
Transferncia do grupo acetilo para a coenzima A, com formao da acetil-
CoA.
Reoxidao do cido lipico pelo FAD e NAD+. O NADH formado reoxidado
pela cadeia transportadora de electres.

Nota: Contrariamente ao habitual, o NAD+ que reoxida o FADH2, uma vez que
nestas circunstncias o potencial redox do FAD/FADH2 inferior ao do
NAD+/NADH.

A descarboxilao oxidativa e o ciclo TCA ocorrem ao nvel da matriz mitocondrial.


Tambm as enzimas implicadas no transporte de electres e na fosforilao
oxidativa se encontram localizadas ao nvel das mitocndrias, o que refora a
eficcia do acoplamento da oxidao da acetil-CoA com a produo de ATP.

Por cada molcula de glucose oxidada a duas molculas de piruvato formam-se,


em aerobiose, 2 acetil-CoA e 2 NADH. A reoxidao do NADH na cadeia
transportadora de electres conduz formao de 5 ATP.

Regulao da Piruvato Desidrogenase


A piruvato desidrogenase regulada por uma protena cinase no dependente do
cAMP e por uma fosfatase. A cinase, ao fosforilar a enzima, inactiva-a; a fosfatase,
que desfosforila a enzima, activa-a.

A piruvato desidrogenase cinase tambm activada por quocientes elevados


ATP/ADP e acetil-CoA/CoA, e por um baixo quociente NAD+/NADH. Nessas
condies, a piruvato desidrogenase inactivada. O piruvato, por seu lado, inibe a
piruvato desidrogenase cinase.

Ciclo TCA

O ciclo TCA traduz-se na converso da acetil-CoA em 2 molculas de CO2 e na


perda de 8 tomos de hidrognio para os aceitadores NAD+ e FAD.

O ciclo TCA permite a oxidao completa da glucose, dos cidos gordos (a CO2 e
H2O, formando acetil-CoA), dos aminocidos (formando piruvato e, depois, acetil-
CoA) e dos prprios intermedirios do ciclo (-cetoglutarato, succinato, fumarato
e oxaloacetato).

70
Brigitta Cismasiu | FMUL 2010/2011

Reaces do Ciclo TCA


Condensao (endergnica)
Transferncia do grupo acetilo do acetil-CoA para o oxaloacetato para formar
citrato, sendo libertada a coenzima A.

Desidratao/Hidratao
Transferncia do grupo OH da posio 3 para a posio 4, formando-se isocitrato.
Origina-se um composto intermdio, o cis-aconitato, mas no libertado do
centro activo da enzima aconitase.

Descarboxilao oxidativa (endergnica)


O isocitrato desoxigenado ao nvel do OH para formar o -cetoglutarato. A
isocitrato desidrogenase pode usar como coenzima o NAD+ ou o NADP+.

Descarboxilao oxidativa (endergnica)


O -cetoglutarato sofre uma descarboxilao oxidativa por aco de um complexo
enzimtico designado -cetoglutarato desidrogenase, que tem como coenzimas o
pirofosfato de tiamina, o cido lipico e o FAD. O Mg+ o cofactor.

71
Brigitta Cismasiu | FMUL 2010/2011

Fosforilao
Sntese de ATP directamente custa da hidrlise do substrato (succinil-CoA), um
composto de potencial energtico elevado. O grupo fosfato inorgnico est ligado
covalentemente enzima, sendo transferido para o GDP e da para o ADP.

Desidrogenao
O succinil oxidado a fumarato por aco da succinato desidrogenase, cuja
coenzima o FAD.

Hidratao
O fumarato recebe um grupo OH por hidratao, originando malato.

Desidrogenao (fortemente endergnica)


O malato oxidado a oxaloacetato pela malato desidrogenase, encerrando-se
assim o ciclo. a reaco mais endergnica, mas encontra-se acoplada reaco
mais exergnica (condensao do acetil-CoA com o oxaloacetato). H produo
de uma grande quantidade de malato e NAD+, de modo a empurrar a reaco no
sentido directo, para a formao de oxaloacetato.

Todas as enzimas so solveis na matriz mitocondrial, excepto a succinato


desidrogenase, que se encontra no espao interno da membrana interna.

O CO2 formado nas reaces 3 e 4 pode ser utilizado em diversas snteses (ureia,
nucletidos, carboxilaes,) ou eliminado pela via pulmonar aps transporte
pelo sangue.

Balano energtico: 3 oxidaes por enzimas de NAD+ ou NADP+ + 1 oxidao por


enzima de FAD + ciso do succinil-CoA = 7,5 + 1,5 + 1 = 10 ATP

Regulao do Ciclo TCA

Uma vez que se situa entre a


gliclise e da cadeia respiratria,
o ciclo TCA deve ser
cuidadosamente regulado pela
clula. Se o ciclo no fosse
controlado, grandes quantidades
de energia metablica poderiam
ser desperdiadas em excesso de
produo de coenzimas reduzidas
e ATP; por outro lado, se se
procedesse muito devagar, o ATP
no poderia ser produzido com
rapidez suficiente para satisfazer
as necessidades da clula.

A regulao do ciclo efectua-se ao nvel das trs reaces irreversveis (1, 3 e 4)


por activao alostrica pelo ADP e NAD+ e inibio pelo ATP e NADH.
Exteriormente ao ciclo, exerce-se uma regulao ao nvel da produo de acetil-
CoA.

72
Brigitta Cismasiu | FMUL 2010/2011

Etapa 1
a nica reaco atravs da qual os tomos de carbono entram no ciclo TCA.

CITRATO SINTASE
O NADH, um produto do ciclo TCA, um inibidor alostrico da citrato sintase, tal
como o succinil-CoA.

Etapa 3
Esta reaco fornece a primeira ligao entre o ciclo TCA com a cadeia
transportadora de electres e a fosforilao oxidativa, atravs da produo de
NADH. Ocorre a primeira descarboxilao.

ISOCITRATO DESIDROGENASE
As elevadas razes NAD+/NADH e ADP/ATP estimulam a isocitrato desidrogenase
e a actividade do ciclo TCA. Este um importante ponto de regulao j que a
razo citrato/isocitrato controla a taxa de produo de acetil-CoA citoslico, uma
vez que este derivado do citrato exportado da mitocndria.

Etapa 4
Ocorre a segunda descarboxilao.

-CETOGLUTARATO DESIDROGENASE
Esta enzima inibida por succinil-CoA e NADH, produtos da reaco que catalisa,
e tambm por grandes concentraes de ATP.

Ciclo Anfiblico

O ciclo TCA a via final comum oxidao de todas as molculas combustveis.


No entanto, as espcies produzidas neste ciclo tambm so combustveis
para uma variedade de processos biossintticos.

O -cetoglutarato, o succinil-CoA, o fumarato e o oxaloacetato so


precursores de importantes metabolitos celulares. Uma reaco de
transaminao converte -cetoglutarato directamente a glutamato, que
pode servir como um precursor verstil da prolina, da arginina e da
glutamina. O succinil-CoA fornece a maior parte dos tomos de carbono
das porfirinas. O oxaloacetato pode ser transaminado para produzir
aspartato. O aspartato em si um precursor de nucleotdeos de pirimidina
e, alm disso, um precursor essencial para a sntese de asparagina,
metionina, lisina, treonina e isoleucina. O oxaloacetato pode tambm ser
descarboxilado para produzir fosfoenolpiruvato, que um elemento-chave
de vrias vias.

73
Brigitta Cismasiu | FMUL 2010/2011

Finalmente, o citrato pode ser exportado da mitocndria e degradado pela


ATP citrato liase para produzir oxaloacetato e acetil-CoA, um precursor dos
cidos gordos. O oxaloacetato produzido nesta reaco rapidamente
reduzido a malato, que pode ento ser processado de duas maneiras: (1)
pode ser transportado para dentro da mitocndria, onde reoxidado a
oxaloacetato, ou (2) sofrer descarboxilao oxidativa a piruvato pela
enzima mlica, com captao mitocondrial subsequente de piruvato. Este
ciclo permite o fornecimento de acetil-CoA a partir de citrato, com retorno
dos subprodutos malato e piruvato mitocndria.

Correlao Clnica

Beribri
uma doena causada por deficincia de tiamina e caracterizada por nveis
elevados de piruvato e -cetoglutarato em circulao, especialmente aps o
consumo de refeies ricas em glucose.

A tiamina, ou vitamina B12, encontra-se no organismo sob a forma de pirofosfato


de tiamina (TPP). O TPP uma coenzima que est envolvida no catabolismo de
glcidos e aminocidos, nomeadamente na descarboxilao do piruvato a
acetaldedo (liga-se ao complexo piruvato desidrogenase) e na descarboxilao
oxidativa do -cetoglutarato a succinil-CoA (liga-se ao complexo -cetoglutarato
desidrogenase). Essas reaces so irreversveis pelo que a ausncia da coenzima
TPP e consequente inactividade dos complexos enzimticos leva a uma
acumulao dos reagentes dessas reaces.

74
Brigitta Cismasiu | FMUL 2010/2011

Cadeia Respiratria
A respirao ocorre na membrana interna das mitocndrias. Este um processo
gerador de ATP no qual o O2 o aceitador final de electres. Dos mais de 80 polipptidos
da cadeia respiratria,
A cadeia respiratria uma das vias envolvidas na fosforilao oxidativa. Os apenas 13 so codificadas
pelo genoma mitocondrial.
electres so transportados do NAD+ + H+ ou da ubiquinona reduzida (QH2) at ao
Os restantes esto
oxignio molecular. Devido grande diferena entre o potencial redox do doador codificados nos genes
(NAD+ + H+ ou QH2) e do aceitador (O2), a reaco fortemente exergnica. A nucleares e so importados
maior parte da energia libertada usada para estabelecer um gradiente para a mitocndria depois
electroqumico atravs da membrana interna, que usado na sntese de ATP, com na sua sntese no
citoplasma.
a ajuda da ATP sintase ou ATPase.

A transferncia de electres processa-se por etapas sucessivas, atravs de uma


srie de transportadores, o que se revela muito favorvel do ponto de vista
energtico. Os transportadores so, na sua maioria, grupos prostticos agrupados
em vrios complexos e encontram-se, partida, no estado oxidado, sendo
reduzidos pelo hidrognio, pelos electres provenientes do substrato ou pelo
transportador precedente. Em seguida, so reoxidados pelo transportador
seguinte. A sua funo promover a oxidao dos diferentes compostos com
reduo do oxignio a H2O e produo de ATP.

Componentes da Cadeia Respiratria


A cadeia transportadora de electres consiste em trs complexos proteicos, os
complexos I (NADH desidrogenase), III (ubiquinol-citocromo c redutase) e IV
(citocromo c oxidase), que esto integrados na membrana mitocondrial interna, e
em duas molculas transportadoras mveis, a ubiquinona (coenzima Q) e o
citocromo c. A succinato desidrogenase, que na verdade pertence ao ciclo TCA,
tambm participa na cadeia respiratria como complexo II. A ATPase (por vezes
designada complexo V) no participa no transporte de electres.

Todos os complexos da cadeia respiratria so constitudos por numerosos


polipptidos e contm uma srie de diferentes coenzimas redox ligadas a
protenas. Estas incluem flavinas (FMN flavin mononucleotide ou FAD flavin
adenine dinucleotide nos complexos I e II), centros de ferro-enxofre (I, II e III) e
grupos heme (II,III e IV).

75
Brigitta Cismasiu | FMUL 2010/2011

Os electres integram a cadeia respiratria por diferentes vias: (1) na oxidao do


NAD+ pelo complexo I, os electres passam atravs de FMN e centros Fe/S para a
ubiquinona; (2) os electres libertados durante a oxidao de succinato, acil-CoA
e outros substratos passam para a ubiquinona pela succinato desidrogenase ou
outra desidrogenase mitocondrial atravs da sua ligao com o FADH2 e (3)
atravs de uma flavoprotena.

O ubiquinol passa electres para o complexo III, que os transfere atravs de


grupos heme e um centro Fe/S para o citocromo c. Este, por sua vez, transporta
os electres para o complexo IV (citocromo c oxidase), que contm componentes
activos redox na forma de dois centros de cobre e heme, atravs dos quais os
electres atingem o O2. Como resultado da reduo do O2, forma-se o anio muito
bsico O2-, que convertido em gua por ligao com dois protes. A
transferncia de electres est acoplada formao de um gradiente
electroqumica pelos complexos I, III e IV.

Organizao
O transporte de protes via complexos I, III e IV ocorre da matriz para o espao
intermembranar. Quando os electres so transportados atravs da cadeia
respiratria, a concentrao de H+ nesse espao aumenta, com consequente
diminuio do valor de pH. Por cada molcula de H2O formada, cerca de 10 ies
H+ so bombeados para o espao intermembranar. Se a membrana interna estiver
intacta, apenas a ATPase consegue permitir o fluxo dos protes para dentro da
matriz.

Os complexos I, II, III e IV e a ATPase no esto em contacto, uma vez que os


electres so transferidos pela ubiquinona e pelo citocromo c. Com a sua longa
cadeia lateral apolar, a ubiquinona move-se livremente na membrana. O
citocromo c hidrfilo e est localizado no exterior da membrana interna.

A oxidao do NADH atravs do complexo I ocorre na matriz, onde esto


localizadas as fontes de NADH (ciclo TCA e -oxidao). A reduo do O2 e a
formao de ATP tambm tm lugar na matriz.

ATP Sintase
A ATP sintase ou ATPase consiste em duas partes: uma bomba de protes
integrada na membrana (com trs centros activos) e uma unidade cataltica que
se projecta na matriz.

O ciclo cataltico pode ser dividido em trs fases:


1. Ligao entre ADP e Pi;
2. Formao da ligao anidrido;
3. Libertao do produto.
Em cada passo, os protes atravessam atravs do
canal para a matriz, alterando a conformao dos
centros activos para o prximo estado.

76
Brigitta Cismasiu | FMUL 2010/2011

Shuttle Malato-Aspartato
A maior parte do NADH utilizado no transporte de electres produzido na matriz
mitocondrial, uma vez que oxidado pelo complexo I no lado interno da
membrana interna. Alm disso, a membrana interna da mitocndria
impermevel a NADH. Contudo, que o NADH resultante da gliclise produzido
no citosol. Se esse NADH no fosse oxidado para regenerar o NAD+, a via glicoltica
deixaria de funcionar. Assim, as clulas eucariticas tm uma srie de sistemas de
transporte que recolhem dos electres do NADH citoslico para a sua entrega na
mitocndria, sem que o NADH seja realmente transportado atravs da membrana
interna.

No shuttle malato-aspartato, o oxaloacetato reduzido a malato no citosol,


adquirindo os electres do NADH, que oxidado a NAD+. O malato transportado
atravs da membrana interna,
onde reoxidado pela malato
desidrogenase, convertendo
+
NAD em NADH na matriz. Esse
NADH mitocondrial facilmente
entra na cadeia transportadora
de electres. O oxaloacetato
produzido nesta reaco no
pode atravessar a membrana
interna e transaminado para
formar aspartato, que pode ser transportado atravs da membrana para o lado
citoslico. O aspartato transaminado no citosol para reciclar o oxaloacetato.

Este ciclo reversvel e actua apenas se a razo NADH/NAD+ no citosol for


superior na matriz. Uma vez que este shuttle produz NADH na matriz, so
recuperados 2,5 ATP por NADH.

Inibidores
So substncias que actuam ao nvel dos complexos e que paralisam
irreversivelmente o fluxo de electres na cadeia respiratria. Uma vez que deixa
de se gerar potencial redutor, no h consumo de O2 e no se forma ATP.

Exemplos de inibidores:
Complexo I: rotenona e amital;
Complexo II: succinato;
Complexo III: antimicina A;
Complexo IV: cianeto (CN-) e monxido de carbono (CO).

Desacopladores
So substncias que actuam ao nvel da ATP sintase e que diminuem a sntese de
ATP, sem paralisar o fluxo de electres e o consumo de O2. Assim, as oxidaes
prosseguem, mas as fosforilaes so inibidas, de modo a que a energia, em vez
de ser utilizada na formao de ATP, dissipada na forma de calor.

Exemplos de inibidores: oligomicina, 2,4-dinitrofenol e tiroxina.

77
Brigitta Cismasiu | FMUL 2010/2011

Efeito de inibidores

Efeito de desacopladores

78
Brigitta Cismasiu | FMUL 2010/2011

Ciclo de Cori
O exerccio fsico vigoroso pode levar a uma escassez de oxignio (anaerobiose), e
as necessidades energticas musculares devem ser satisfeitas pelo aumento dos
nveis de gliclise. Sob tais condies, a gliclise converte NAD+ em NADH, mas o
O2 no est disponvel para a regenerao do NAD+ atravs da respirao celular.
Em vez disso, grandes quantidades de NADH so reoxidadas pela reduo do
piruvato a lactato. O lactato assim produzido pode ser transportado a partir de
msculos para o fgado, onde reoxidado pela lactato desidrogenase, originando
piruvato, que finalmente convertido em glucose. Desta forma, o fgado interfere
no stress metablico criado pelo exerccio vigoroso, exportando glucose para os
msculos.

Este processo conhecido como ciclo de Cori. O fgado, com uma elevada razo
NAD+/NADH, produz mais glucose do que aquela que utiliza. Quando o msculo
esta a ser exercitando vigorosamente entra anaerobiose e sofre uma diminuio
na razo NAD+/NADH, o que favorece a reduo de piruvato a lactato.

79
Brigitta Cismasiu | FMUL 2010/2011

Gluconeognese
A capacidade de sintetizar glucose a partir de metabolitos comuns muito
importante para a maioria dos organismos. Se a glucose no obtida atravs da
dieta, o corpo deve produzir glucose a partir de novos precursores no-glicdicos,
processo denominado gluconeognese.

Alm disso, os msculos consomem grandes quantidades de glucose atravs da


gliclise, produzindo grandes quantidades de piruvato. No exerccio vigoroso, as
clulas musculares tornam-se anaerbias e o piruvato convertido a lactato. A
gluconeognese reutiliza esse piruvato e lactato, reconvertendo-os em glucose.

Alm de piruvato e lactato, outros


precursores podem ser utilizados
como substrato para a
gluconeognese. Estes incluem a
maioria dos aminocidos, bem
como o glicerol e todos os
intermedirios do ciclo TCA. Por
outro lado, os cidos gordos, a
lisina e a leucina no so substratos
para a gluconeognese em animais,
porque da sua degradao resulta
apenas acetil-CoA. O acetil-CoA s
substrato para a gluconeognese
quando est presente tambm
oxaloacetato.

Os rgos que consomem mais


glucose, ou seja, o crebro e o
msculo, tm uma taxa muito baixa
de sntese de glucose. A
gluconeognese ocorre
principalmente no fgado e nos rins
e essa glucose libertada no
sangue e posteriormente absorvida
pelo crebro, corao, msculos e
eritrcitos, para satisfazer as suas
necessidades metablicas. Por sua
vez, o piruvato e o lactato
produzidos nesses tecidos so
devolvidos ao fgado e aos rins para
serem usados como substratos
gluconeognicos.

80
Brigitta Cismasiu | FMUL 2010/2011

A gluconeognese no o inverso exacto da gliclise, porque, nesse caso, seria


um processo fortemente endergnico e no poderia ocorrer espontaneamente.
Por outro lado, a gliclise e a gluconeognese devem ser reguladas de um modo
recproco para que, quando uma dessas vias est activa, a outra seja inibida.
Ambas as limitaes so superadas pela existncia de certas reaces exclusivas a
cada uma das vias. As trs reaces exergnicas da gliclise, reguladas pelas
enzimas glucocinase, fosfofrutocinase e piruvato cinase, so substitudas por
reaces alternativas na gluconeognese.

Reaces Exclusivas da Gluconeognese


Carboxilao
O piruvato e convertido a oxaloacetato atravs da aco da piruvato carboxilase,
com consumo de ATP.

Descarboxilao fosforilante
A fosfoenolpiruvato carboxicinase converte o oxaloacetato a fosfoenolpiruvato,
na presena de GTP.

Hidrlise
A frutose 6-fosfato formada a partir da frutose 1,6-bisfosfato, por hidrlise do
ster de fosfato em C1, reaco catalisada pela frutose 1,6-bisfosfatase.

Hidrlise
formada glucose atravs da hidrlise da glucose 6-fosfato, numa reaco
catalisada pela glucose 6-fosfatase.

Regulao da Gluconeognese

As etapas exclusivas da gluconeognese asseguram um melhor controlo do


processo.

Etapa 1
A gluconeognese inicia-se na reaco catalisada pela piruvato carboxilase,
converso de piruvato a oxaloacetato. A reaco envolve ATP e bicarbonato como
substratos e utiliza biotina como coenzima e acetil-CoA como activador alostrico.

PIRUVATO CARBOXILASE
Enzima tetramrica em que cada monmero possui uma ligao covalente com a
biotina no centro activo. A piruvato carboxilase alostericamente activada pelo
acetil-CoA, o que constitui um importante ponto regulao fisiolgica. O acetil-
CoA o substrato primrio para o ciclo de TCA e o oxaloacetato (formado pela
piruvato carboxilase) um importante intermedirio, tanto do ciclo TCA como da
gluconeognese. Se os nveis de ATP e/ou acetil-CoA (ou outros acil-CoA) forem
baixos, o piruvato dirigido principalmente para o ciclo TCA, que eventualmente
promove a sntese de ATP. Se os nveis de ATP e acetil-CoA forem elevados, o
piruvato convertido a oxaloacetato e consumido na gluconeognese.

81
Brigitta Cismasiu | FMUL 2010/2011

Uma outra caracterstica da piruvato carboxilase que ela s encontrada na


matriz mitocondrial. Em contrapartida, a enzima seguinte da gluconeognese,
fosfoenolpiruvato carboxicinase, est localizada no citosol e na mitocndria. O
piruvato transportado para a matriz mitocondrial, onde pode ser convertido a
acetil-CoA (para utilizao no ciclo TCA) e, em seguida, em citrato (para a sntese
de cidos gordos). Como alternativa, pode ser convertido directamente a
oxaloacetato pela piruvato carboxilase e utilizado na gluconeognese. No
entanto, o oxaloacetato no pode ser transportado para o citosol directamente
atravs da membrana mitocondrial. Em vez disso, deve ser primeiro transformado
em malato pela malato desidrogenase para o transporte atravs da membrana
mitocondrial interna, e novamente convertido a oxaloacetato antes de continuar
a via gluconeognica.

Etapa 2
A converso de oxaloacetato a fosfoenolpiruvato produz um metabolito de alta
energia.

FOSFOENOLPIRUVATO CARBOXICINASE
Os requisitos energticos so supridos de duas maneiras. Primeiro, o CO2
adicionado ao piruvato na etapa da piruvato carboxilase removido na reaco
da fosfoenolpiruvato carboxicinase, atravs de um processo muito exergnico. Em
segundo lugar, consumido um fosfato de alta energia do GTP.

O lactato produzido pela gliclise nos eritrcitos ou no msculo convertido a


piruvato no fgado, com produo de NADH, pelo que a exportao de
equivalentes redutores (como o malato) a partir da mitocndria desnecessria.
Aps a converso do malato em oxaloacetato pela piruvato carboxilase, este
convertido directamente a fosfoenolpiruvato por uma isoenzima mitocondrial da
fosfoenolpiruvato carboxicinase, e transportado para fora da mitocndria para
continuar no caminho gluconeognico. As isoenzimas mitocondrial e citoslica da
fosfoenolpiruvato carboxicinase so codificadas por genes diferentes nos
cromossomas nucleares.

82
Brigitta Cismasiu | FMUL 2010/2011

Etapa 3
A hidrlise da frutose 1,6-bisfosfato em frutose-6-fosfato termodinamicamente
favorvel nas condies fisiolgicas hepticas.

FRUTOSE 1,6-BISFOSFATASE
A frutose 1,6-bisfosfatase alostericamente regulada pela citrato bisfosfatase,
que estimula a sua actividade. O AMP tambm inibe a frutose 1,6-bisfosfatase,
inibio essa que reforada pela frutose 2,6-bifosfato.

FRUTOSE 2,6-BISFOSFATO
A enzima bifuncional que controla a sntese e degradao da frutose 2,6-
bisfosfato contm uma zona reguladora, seguida de um domnio cinase e de um
domnio fosfatase que so reciprocamente regulados. A fosforilao activa o
domnio fosfatase da enzima, o qual degrada a frutose 2,6-bisfosfato, enquanto a
desfosforilao activa o domnio cinase da enzima, que sintetiza esse composto.
Quando o nvel de glucose baixo, um aumento da concentrao de glucagina
activa a sntese de cAMP, o que conduz fosforilao da enzima; a frutose 2,6-
bisfosfato degradada, o que reduz a sua concentrao. A gluconeognese
estimulada porque, em baixos teores, a frutose 2,6-bisfosfato activa a frutose 1,6-
bisfosfatase. Quando o nvel de glucose elevado, a enzima desfosforilada; a
frutose 2,6-bisfosfato sintetizada e, como este composto em teores elevados
um inibidor da frutose 1,6-bisfosfatase e activador da fosfofrutocinase, a gliclise
estimulada

Etapa 4
O ltimo passo na via da gluconeognese a converso de glucose 6-fosfato em
glucose pela aco da glucose 6-fosfatase. Esta enzima est presente na
membrana do retculo endoplasmtico do fgado e do rim, mas est ausente no
msculo e no crebro. A sua associao membrana importante para a sua
funo, uma vez que o substrato hidrolisado e passa para o prprio retculo
endoplasmtico. Formam-se vesculas a partir da membrana do retculo
endoplasmtico e difundem-se para a membrana plasmtica, com a qual se
fundem, lanando o seu contedo na corrente sangunea.

83
Brigitta Cismasiu | FMUL 2010/2011

GLUCOSE 6-FOSFATASE
A glucose 6-fosfatase no est sob controlo alostrico. No entanto, o KM para o
substrato, glucose 6-fosfato, consideravelmente maior do que a sua
concentrao normal no organismo. Como resultado, a glucose 6-fosfatase
apresenta uma dependncia quase linear de actividade em relao s
concentraes de glucose 6-fosfato e, portanto, diz-se que est sob um controlo
ao nvel do substrato.

Experincia

Qual seria a consequncia da existncia de fosfoenolpiruvato


carboxicinase no tecido muscular?
EXPERINCIA
Comparao de ratos normais (tipo selvagem) com ratos transgnicos (com a
fosfoenolpiruvato carboxicinase activa no msculo).

A gliconeognese ocorre at frutose 1,6-bisfosfato, porque a enzima que


catalisa a reaco seguinte requer outra enzima especfica (frutose 1,6-
bisfosfatase).

RESULTADOS
A sobre-expresso da forma citoslica da enzima fosfoenolpiruvato carboxicinase
no msculo esqueltico levou criao de uma linha de super ratos. Esta enzima
est envolvida em duas vias principais, a gluconeognese e gliceroneognese.

84
Brigitta Cismasiu | FMUL 2010/2011

Estes ratos consomem, em mdia, 60% mais alimento e pesam 50% menos do que
a linhagem controlo de tipo selvagem. Exibem um aumento de longevidade e so
capazes de se reproduzir em idades mais avanadas, bem como de executar
tarefas comportamentais de grande resistncia por mais tempo. No acumulam
gordura visceral e cutnea, mas tm mais TAG no msculo. So hiperactivos
devido ao aumento da taxa de metabolizao do ciclo TCA. No se adaptam
escassez de alimentos, apresentam uma desregulao hormonal e sofrem de
problemas renais, pois o ciclo da ureia sobre-usado.

RATOS TRANSGNICOS V F ?
1. No sobrevivem mais de 2 semanas.
2. Tm maior longevidade.
3. Tornam-se obesos.
4. So hiperactivos.
5. So inactivos.
6. Perdem plo.
7. Tm subdesenvolvimento muscular.
8. Tm subdesenvolvimento heptico.
9. Tm subdesenvolvimento renal.
10. Desenvolvem fentipo de Alzheimer.

85
Brigitta Cismasiu | FMUL 2010/2011

Metabolismo do Glicognio
Glicogenlise

A aco cooperativa e repetitiva da glicognio fosforilase e da enzima


desramificadora assegura a degradao completa do glicognio.

A glicognio fosforilase a enzima chave da glicogenlise uma vez que cliva as


ligaes -1,4 entre C1 de um resduo glicosil e C4 do seguinte, atravs da adio
de um fosfato inorgnico, originando molculas de glucose 1-fosfato. A sua
actividade utiliza o fosfato de piridoxal (PLP), um derivado da vitamina B6. A
glucose 1-fosfato no se difunde para fora da clula e facilmente convertida em
glucose 6-fosfato pela fosfoglucomutase.

No entanto, a glicognio fosforilase termina a clivagem quando chega a 4 resduos


de uma ramificao. A clivagem das ligaes -1,6 exige a aco de outras duas
enzimas: a transferase, que transfere um bloco de 3 resduos da ramificao que
est a ser degradada para outra mais exterior, e a -1,6-glucosidase ou enzima
desramificadora, que hidrolisa a ligao -1,6, libertando glucose, que
fosforilada a glucose 6-fosfato pela hexocinase.

O fgado contm a enzima hidroltica glucose 6-fosfatase que cliva o grupo fosforil
da glucose 6-fosfato, para formar glucose livre e o fosfato inorgnico. Esta enzima
no existe na maioria dos outros tecidos, o que faz com que a glucose 6-fosfato
fique retida para a produo de ATP.

86
Brigitta Cismasiu | FMUL 2010/2011

Glicognese

Consiste na sntese do glicognio a partir de glucose.

O incio da sntese de glicognio precisa de glicogenina, um oligossacrido que


sofre um processo de auto-glicolisao, em que catalisa a adio de 8 molculas
de glucose doadas pela UDP-glucose.

A glicognese utiliza uridina difosfato glucose (UDP-glucose) em vez da glucose 1-


fosfato. A UDP-glucose uma forma activa de glucose e sintetizada a partir da
glucose 1-fosfato e uridina trifosfato (UTP), numa reaco catalisada pela UDP-
glucose pirofosforilase.

Esta reaco seria reversvel se o pirofosfato (PPi) no fosse rapidamente


hidrolisado em 2 Pi. Esta hidrlise altamente exergnica, tornando a reaco
total praticamente irreversvel e direccionando-a no sentido da formao da UDP-
glucose.

Uma unidade UDP-glucose adicionada ao C4 terminal do glicognio para formar


a ligao -1,4 e a UDP retirada da molcula de glicognio em crescimento,
numa reaco catalisada pela glicognio sintetase (enzima chave da regulao da
sntese de glicognio).

Uma enzima ramificadora necessria para criar as ligaes -1,6 que fazem do
glicognio um polmero ramificado. Uma ramificao criada quebrando uma
ligao -1,4 de forma a transferir um bloco de 7 resduos para uma cadeia mais
interior. Estes 7 resduos tm que provir de uma cadeia de mais de 11 resduos de
forma a deixar pelo menos 4 que podem assim sofrer a aco da glicognio
sintetase.

87
Brigitta Cismasiu | FMUL 2010/2011

Doenas Associadas ao Metabolismo do Glicognio

Von Gierke (glucose 6-fosfatase)


Acumulao de glicognio heptico (hepatomeglia);
Inabilidade de corrigir a hipoglicmia.

Pompe (-1,4 glicosidase*)


Acumulao de glicognio;
Insuficincia cardiorrespiratria;
Morte prematura.

Cori (enzima desramificadora)


Glicognio com ramificaes curtas, resultantes da aco da glicognio
fosforilase;
Hipoglicmia;
Hepatomeglia.

Andersen (enzima ramificadora)


Glicognio com cadeias muito longas no ramificadas;
Morte prematura.

McArdle (glicognio fosforilase muscular)


Incapacidade de realizar exerccios intensos.

Hers (glicognio fosforilase heptica)


Acumulao de glicognio heptico (hepatomeglia);
Inabilidade de corrigir a hipoglicmia.

Tarui (fosfofrutocinase muscular)


Glicognio com cadeias muito longas no ramificadas;
Morte prematura.

*Enzima lisossomal que hidrolisa os segmentos lineares de glicognio (via


secundria do metabolismo do glicognio).

Correlao Clnica

Doena de Von Gierke


provocada pela deficincia da glucose 6-fosfatase heptica, intestinal e renal.
Manifesta-se por hipoglicmia em jejum, uma vez que o glicognio no
eficientemente desramificado a glucose, acidmia lctica, j que o fgado no
consegue utilizar eficazmente o lactato para a gluconeognese e porque este
produzido inapropriadamente em resposta glucagina, hiperuricmia, devido ao
aumento da degradao de purinas no fgado, e hipelipidmia, porque h um

88
Brigitta Cismasiu | FMUL 2010/2011

aumento de disponibilidade de lactato para a liplise e de mobilizao de lpidos


do tecido adiposo.

As manifestaes clnicas podem ser bastante diminudas pela administrao de


refeies ricas em glcidos ao longo do dia, de modo a prevenir a hipoglicmia.

89
Brigitta Cismasiu | FMUL 2010/2011

Via dos Fosfatos de Pentose


Importncia da Via
1. Permite a formao de NADPH, necessrio sntese de cidos gordos e
esteris;
2. Transforma a glucose em pentoses, produzindo ribose 5-fosfato para a
sntese de nucletidos;
3. Permite a oxidao da glucose a CO2 com formao de ATP, desde que
possa ocorrer a reoxidao do NADPH a NADP+.

um processo essencialmente aerbio, pois a reoxidao das coenzimas


reduzidas s possvel atravs do sistema transportador de electres ou de
reaces de biossntese que utilizem NADPH e gerem NADP+ (ex.: biossntese do
colesterol ou dos cidos gordos).

Reaces da Via dos Fosfatos de Pentose


Fase oxidante: oxidao e descarboxilao da glucose 6-fosfato em ribose 5-
fosfato com formao de NADPH. catalisada, em parte, pela glucose 6-fosfato
desidrogenase (G6PD).

O NADPH pode ser usado para reaces biossintticas redutoras e a ribose 5-


fosfato como precursor da sntese de nucletidos


Fase no oxidante: transferncia de grupos com 3 tomos de carbono
(transaldolisao) e com 2 tomos de carbono (transcetolisao) por uma srie de
interconverses.

H formao de alguns intermedirios da gliclise: frutose 6-fosfato e


gliceraldedo 3-fosfato.

O NADPH pode seguir vrios destinos: (1) biossntese de cidos gordos, colesterol,
nucletidos e neurotransmissores, reduo do glutatio e monooxigenases do
citocromo P450 (enzima heptica).

90
Brigitta Cismasiu | FMUL 2010/2011

O NADH e o NADPH, no metabolismo, tm geralmente destinos diferentes: o


NADH ser oxidado pela cadeia respiratria para gerar ATP, enquanto o NADPH
actua como dador de hidrognio e de electres nas biossnteses redutoras.

As vias de sntese dependem da interveno de coenzimas na forma reduzida


sobretudo NADPH e de ATP; a transformao de NAD+ em NADH e de NADP+ em
NADPH ocorrem nas vias de degradao.

Importncia do NADPH
1. Proteco contra espcies reactivas de oxignio
Ao aceitar electres, o oxignio fica muito instvel, reagindo com outras
molculas e causando danos inespecficos. O NADPH actua ao nvel do
glutatio, uma molcula que, na sua forma reduzida (glutationa), capaz
de eliminar o H2O2 e formar gua. Os eritrcitos so clulas especialmente
susceptveis ao oxignio reactivo, pelo que tm mecanismos prprios de
proteco: o decrscimo da actividade da G6PD diminui o tempo de vida
dos eritrcitos.

2. Citocromo P450* usa o NADPH para hidroxilar


utilizado o poder reactivo do O2, de forma controlada, pelo citocromo
P450 (associado ao RE), para hidroxilar molculas estranhas, tornando-as
mais polares, logo, mais solveis na corrente sangunea. Em princpio,

91
Brigitta Cismasiu | FMUL 2010/2011

qualquer molcula solvel acaba por ser eliminada nos rins. As molculas
pouco polares alojam-se nas membranas e entre os lpidos.
* Grupo de enzimas que metabolizam compostos potencialmente txicos,
incluindo medicamentos e metabolitos endgenos, como a bilirrubina,
principalmente no fgado.

3. Formao de espcies reactivas para ataque qumico (papel imunitrio)


O NADPH fora o aparecimento de espcies reactivas de O2 num
ambiente controlado (fagossoma), que destroem o corpo estranho.

4. Produo de xido ntrico


O xido ntrico, gs temperatura ambiente, provoca vasodilatao e
agregao de plaquetas, funciona como neurotransmissor, tem funes
de analgesia e est presente nos lisossomas.

Regulao da Via dos Fosfatos de Pentose


O factor de regulao mais importante desta via o nvel de NADP+, que o
aceitador de electres na oxidao de glucose 6-fosfato a cido 6-fosfoglicrico.

Devido ao efeito do teor de NADP+, no citosol, sobre a velocidade da fase


oxidante, fica assegurada uma relao muito estreita entre a produo de NADPH
e a sua utilizao nas redues metablicas e, dessa forma, regulado o valor do
quociente NADP+/NADPH.

Se o organismo requer maior quantidade de ribose 5-fosfato do que de


NADPH (a biossntese de protenas predomina sobre a de lpidos) apenas
funciona a fase no oxidante da via. Nestas condies, a frutose 6-fosfato e
o gliceraldedo 3-fosfato so transformados em ribose 5-fosfato sem
formao de NADPH;
No caso contrrio, a ribose 5-fosfato formada na fase oxidante pode ser
convertida em frutose 6-fosfato e gliceraldedo 3-fosfato, e da em cido
pirvico. Por este processo gera-se ATP e NADPH.

92
Brigitta Cismasiu | FMUL 2010/2011

Correlao Clnica

Deficincia de G6PD
A deficincia de glucose 6-fosfato desidrogenase diminui a capacidade de
formao de NADPH e, consequentemente, de glutationa.

Os eritrcitos obtm energia apenas atravs da gliclise e potenciais redutores


pela via dos fosfatos de pentose. Estando essa via comprometida, no h
produo de NADPH reflectindo-se na ausncia de proteco contra agentes
oxidantes.

Os anti-malricos e as favas produzem grandes radicais muito reactivos, que no


conseguem ser eliminados e que interagem com a membrana dos eritrcitos,
provocando a sua lise (anemia hemoltica).

Esta doena caracteriza-se pelo aparecimento de ictercia, devido acumulao


de bilirrubina, principal produto do metabolismo do heme da hemoglobina, e de
valores elevados de LDH na sangue, ambos os factores indicando hemlise.

93
Brigitta Cismasiu | FMUL 2010/2011

Metabolismo dos Nucletidos


Os nucletidos desempenham uma variedade de funes importantes nas clulas:
Transportadores de energia qumica, como o ATP e o GTP;
Monmeros para sntese de cidos nucleicos DNA ou RNA;
Componentes de cofactores, como NAD+, FAD, coenzima A, etc.;
Mediadores fisiolgicos, como cAMP e cGMP;
Precursores de: estrutura 5 cap de mRNA (GTP), tetrahidrobiopterina
(GTP);
Intermedirios activados, como UDP-glucose, S-adenosilmetionina, etc.;
Efectores alostricos.

Degradao de Nucletidos

Degradao de Nucletidos Purnicos


O catabolismo de ribonucletidos e desoxirribonucletidos purnicos conduz
produo de cido rico.

A hipoxantina e a
xantina so bases
purnicas mais raras.

Em diferentes organismos, o cido rico metabolizado em diferentes produtos.


Os primatas so capazes de excretar cido rico pela urina, enquanto que a
maioria dos mamferos converte este cido em alantona por aco da urato
oxidase.

94
Brigitta Cismasiu | FMUL 2010/2011

Degradao de Nucletidos Pirimidnicos


O catabolismo de ribonucletidos e desoxirribonucletidos pirimidnicos conduz
produo de amnia, que eliminada pelo ciclo da ureia, e de malonil-CoA.

Este processo de
degradao utiliza parte
das reaces envolvidas
na -oxidao de cidos
gordos de cadeia mpar.

Sntese de Nucletidos

Com a excepo do ATP, as reservas celulares de nucletidos no representam


mais de 1% do que necessrio para transcrio e replicao. A sntese de
nucletidos ocorre em simultneo com estes processos, podendo mesmo limit-
los.

A biossntese de nucletidos pode ocorrer por duas vias metablicas:


1. A sntese de novo a partir de precursores metablicos (ribose activada +
aminocidos + ATP + CO2);
2. A via de reciclagem, que reutiliza bases e nuclesidos libertados a partir
da degradao de cidos nucleicos (ribose activada + base/nuclesido).

As biossnteses de novo de nucletidos purnicos e pirimidnicos ocorrem atravs


de vias distintas. Em ambas, ocorre primeiro a sntese de ribonucletidos que so
ento convertidos em desoxirribonucletidos.

95
Brigitta Cismasiu | FMUL 2010/2011

As vias de biossntese de novo partilham alguns precursores, como o caso de


aminocidos e do 5-fosforibosil-1-pirofosfato (PRPP), uma forma activada da
A ribose 5-fosfato
ribose 5-fosfato. obtida a partir da via
dos fosfatos de
Na sntese de novo de nucletidos purnicos, o pentose.
anel purnico sintetizado passo a passo sobre o
PRPP, a partir de diferentes precursores.

Os eritrcitos no so capazes de efectuar esta via, sendo dependentes das vias


de reciclagem.

Biossntese de Novo de Nucletidos Purnicos

96
Brigitta Cismasiu | FMUL 2010/2011

A partir de um intermedirio comum, o inosinato (IMP), obtm-se AMP e GMP


por vias distintas. As snteses de AMP e GMP requerem a hidrlise de GTP e ATP,
respectivamente, o que permite uma produo equilibrada daqueles nucletidos.

Biossntese de Novo de Nucletidos Pirimidnicos


Na sntese de novo de nucletidos pirimidnicos, o anel pirimidnico sintetizado
parte e posteriormente conjugado ao PRPP, sendo que os diferentes tomos
que o compem provm de diferentes precursores.

De incio, ocorre a formao de carbamoil-fosfato no citoplasma por aco da


carbamoil-fosfato sintase II (CPS II), sendo a glutamina o dador de azoto.
O carbamoil-fosfato
ainda usado no ciclo da
ureia, onde sintetizado
na mitocndria por
aco de outra isoforma
da carbamoil-fosfato
sintetase, a CPS I.

Por conjugao com o aspartato, origina-se o anel pirimidnico sob a forma de


orotato.

O anel pirimidnico ento conjugado ao PRPP, obtendo-se uridilato (UMP).

97
Brigitta Cismasiu | FMUL 2010/2011

A partir de UMP pode-se obter o trifosfato de uridina (UTP). Numa reaco


catalisada pela citidilato sintetase, o UTP pode-se converter em trifosfato de
citidina (CTP).

Converso dos Monofosfatos de Nuclesidos em Trifosfatos de


Nuclesidos
Os monofosfatos de nuclesidos (NMP) so fosforilados em difosfatos de
nuclesidos (NDP) por NMP cinases, que so em geral especficas para uma base,
mas inespecficas para a pentose.

Os NDP podem ser convertidos em NTP pela NDP cinase que inespecfica tanto
em relao base como pentose.

Em geral, estas reaces envolvem o ATP, como NTP dador, dado encontrar-se em
maiores concentraes que os restantes NTP.

Os Ribonucletidos so Percursores dos Desoxirribonucletidos


Os difosfatos de ribonucletidos so convertidos em difosfatos de
desoxirribonucletidos por reduo da ribose em desoxirribose pela aco da
ribonucletido redutase. A reduo requer dois tomos de hidrognio que
podem ser cedidos por duas vias distintas.

98
Brigitta Cismasiu | FMUL 2010/2011

Biossntese do Timidilato
A sntese de DNA no utiliza fosfatos de uridina, mas sim de timidina.
A desfosforilao de
dUTP em dUMP
muito eficiente para
manter baixos os
nveis de dUTP e
evitar a sua
incorporao no DNA.

Para tal, o desoxitimidilato (dTMP) obtido a partir dos difosfatos de


desoxicitidina e desoxiuridina.

Via da Reciclagem
A degradao de nucletidos conduz libertao de bases ou nuclesidos, que
podem ser reusados para a sntese de nucletidos purnicos ou pirimidnicos por
vias de reciclagem.

Por exemplo, os nucletidos purnicos podem ser sintetizados a partir das bases
livres em reaces catalisadas:
Pela adenosina fosforibosiltransferase

Pela guanina-hipoxantina fosforibosiltransferase (HGPRT)

Correlaes Clnicas

Gota
A gota caracteriza-se por nveis elevados de cido rico no sangue e tecidos,
devido a vrios defeitos metablicos associados (1) sobreproduo de
nucletidos purnicos ou (2) menor excreo de cido rico.

Dada a sua baixa solubilidade, o cido rico tem tendncia a depositar-se sob a
forma de cristais de urato de sdio. Esta deposio ocorre preferencialmente em
articulaes e rins.

Entre outras opes, a teraputica desta doena pode


incluir:
1. A excluso da dieta alimentar de alimentos
ricos em nucletidos, como fgado;
2. A administrao de alopurinol convertido pela
xantina oxidase em oxipurinol.

Este permanece ligado ao stio activo da enzima, inactivando-a. Nestas condies,


excreta-se xantina e hipoxantina.

99
Brigitta Cismasiu | FMUL 2010/2011

Sndrome de Lesch-Nyhan
Est associada deficincia gentica da HGPRT. Aparece em crianas de sexo
masculino que apresentam:
1. Produo excessiva de cido rico associado a hiperuricmia;
2. Perturbaes neurolgicas como fraca coordenao motora, atraso
mental e um comportamento hostil.

O tratamento com alopurinol permite apenas aliviar os sintomas associados


sobreproduo de cido rico.

100
Brigitta Cismasiu | FMUL 2010/2011

Frutlise
A frutose metabolizada apenas no fgado (onde direccionada para reposio
de glicognio heptico e sntese de TAG), vesculas seminais (fonte de energia dos
espermatozides) e cristalino (excesso de glucose produz sorbitol, que se acumula
e provoca desequilbrio osmtico, com entrada de gua: perda de viso).

Correlao Clnica

Retinopatia Diabtica
A retinopatia diabtica uma das principais causas de cegueira adquirida em todo
o Mundo.

A captao de glucose pela retina independente de insulina, pelo que os nveis


de glucose neste tecido tendem a estar em equilbrio com a glicmia. A
retinopatia, quando associada a diabetes, uma complicao resultante
essencialmente de situaes de hiperglicmia recorrentes e prolongadas, que
provocam alteraes metablicas na retina (aumento da actividade da via dos
poliis e da produo de produtos finais de glicolisao avanada, stress
oxidativo) e inviabilizam progressivamente as clulas e estruturas vasculares
associadas.

A conjugao de todos estes factores induz a destruio de pericitos e clulas


endoteliais, ruptura da barreira hemato-retiniana, hipoperfuso tecidual e
espessamento da membrana basal associada aos vasos sanguneos, promovendo
alteraes fisiolgicas e anatmicas na retina, que explicam a perda de viso
progressiva ou, em casos graves, definitiva dos doentes que sofrem de retinopatia
diabtica.

101
Brigitta Cismasiu | FMUL 2010/2011

Liplise ou -Oxidao
A utilizao de cidos gordos para produo de energia varia consideravelmente
de tecido para tecido e depende do estado metablico, ou seja, alimentado ou
jejum, exerccio ou repouso.

A maioria dos tecidos consegue utilizar os cidos gordos como combustvel. Os


cidos gordos so a principal fonte de energia dos msculos esqueltico e
cardaco. O encfalo no faz -oxidao porque os cidos gordos no conseguem
atravessar a barreira hematoenceflica; os eritrcitos tambm so incapazes de
obter energia desse modo, devido ausncia de mitocndrias, local onde se
processa a -oxidao.

Libertao dos cidos Gordos

Os cidos gordos so mobilizados a partir de adipcitos em resposta a


mensageiros hormonais como a adrenalina e a glucagina. Estas molculas ligam-
se a receptores da membrana plasmtica dos adipcitos e que levam activao
da guanilato ciclase, que produz cAMP a partir de ATP. O cAMP activa a protena
cinase A, que fosforila e activa lipases responsveis pela que hidrlise dos TAG.

O glicerol absorvido pelo fgado e fosforilado e oxidado para formar


diidroxiacetona fosfato, que isomerada a gliceraldedo 3-fosfato (intermedirio
da gliclise e gliconeognese). Pode ser convertido a glucose ou piruvato.

Os cidos gordos no so solveis no plasma e por isso ligam-se albumina


(protena transportadora presente no plasma) para serem transportados para os
tecidos.

102
Brigitta Cismasiu | FMUL 2010/2011

cidos Gordos da Dieta

Os TAG alimentares so pouco degradados por lipases no estmago e passam


praticamente intactos para o duodeno. O suco pancretico alcalino segregado
para o duodeno aumenta o pH do ambiente, permitindo a hidrlise dos TAG pela
lipase pancretica e por esterases inespecficas, que hidrolisam as ligaes
steres de cidos gordos.

Estes processos dependem da presena de sais biliares, que


actuam como detergentes para emulsionar os TAG e facilitar a
actividade hidroltica das lipases e esterases. Os cidos gordos
de cadeia curta so absorvidos directamente nas vilosidades
da mucosa intestinal, enquanto que cidos gordos de cadeia
longa, que so menos solveis, formam micelas mistas com os
sais biliares e so transportadas desta forma para a superfcie
das clulas epiteliais que cobrem as vilosidades.

Nas clulas epiteliais, os cidos gordos so condensados com o glicerol para


formar novos TAG. Estes agregados de TAG com lipoprotenas formam partculas
chamadas de quilomicras, que so depois transportadas para o sistema linftico e
para a corrente sangunea, onde circulam para o fgado, pulmes, corao,
msculos e outros rgos. Nestes locais, os TAG so hidrolisados e libertam os
cidos gordos, que podem ento ser oxidados numa via metablica altamente
exergnica, a -oxidao.

Activao dos cidos Gordos

Antes de serem oxidados, os cidos gordos tm de ser activados. Esta activao


requer energia fornecida pelo ATP e ocorre na membrana mitocondrial externa
por aco da acil-CoA sintetase. O cido gordo convertido a acil-CoA e
libertado no citosol.

Entrada do Acil-CoA para a Matriz Mitocondrial

As enzimas responsveis pela oxidao dos cidos gordos localizam-se na matriz


mitocondrial, mas a membrana mitocondrial impermevel CoA e seus
derivados. Os cidos gordos so, ento, transportados para a matriz mitocondrial
atravs da carnitina.

O grupo acil transferido da CoA para a carnitina pela carnitina aciltransferase I,


na membrana mitocondrial externa. O grupo acil carnitina e a carnitina livre so
transportados por antiporte pela carnitina-acilcarnitina translocase. Uma vez na

103
Brigitta Cismasiu | FMUL 2010/2011

matriz mitocondrial, o grupo acil novamente transferido para a CoA pela


carnitina aciltransferase II.

A carnitina aciltransferase I um importante ponto de regulao da liplise, uma


vez que controla a entrada de cidos gordos na mitocndria e, como tal,
determina o fornecimento de substrato para a -oxidao na matriz mitocondrial.

-Oxidao

O acil-CoA pode entrar na via da -oxidao, que conduz a um novo acil-CoA com
menos 2 tomos de carbono que o anterior. A degradao de um acil-CoA
processa-se numa sequncia repetida de 4 reaces e gera FADH2 e NADH.

104
Brigitta Cismasiu | FMUL 2010/2011

Reaces da -Oxidao
Oxidao
O acil-CoA oxidado pela acil-CoA desidrogenase, uma flavoprotena que usa o
FAD como aceitador de electres, originando enoil-CoA (com uma dupla ligao
entre C2 e C3) e um FADH2

Hidratao
A dupla ligao entre C2 e C3 sofre hidratao por aco da enoil-CoA hidratase,
formando L--hidroxiacil-CoA.

Oxidao
O grupo hidroxilo convertido numa cetona pela L-hidroxiacil-CoA desidrogenase,
originando -cetoacil-CoA e NADH.

Tilise
Quebra da cadeia de -cetoacil-CoA por uma tiolase, originando um acetil-CoA e
um acil-CoA reduzido em 2 tomos de carbono.
Esta molcula reduzida de acil-CoA vai entrar num novo ciclo de oxidao. Na
maioria dos tecidos, o acetil-CoA usado no ciclo TCA e o FADH2 e o NADH vo ser
reoxidados pela via da fosforilao oxidativa, com produo de ATP.

Balano Energtico
Cada ciclo da -oxidao produz um acetil-CoA, um FADH2 e um NADH. O
rendimento da oxidao de cada acetil-CoA no ciclo TCA 10 ATP e a fosforilao
oxidativa de cada FADH2 e NADH 1,5 e 2,5 ATP, respectivamente.

A oxidao dos cidos gordos fornece grande quantidade de energia para a


gluconeognese.

-Oxidao de cidos Gordos com Nmero mpar de Carbonos


Os cidos gordos com nmero mpar de tomos de carbono so oxidados pela -
oxidao. Os produtos finais da clivagem final so acetil-CoA e propionil-CoA.

O propionil-CoA carboxilado a metilmalonil-CoA e convertido a succinil-CoA.

-Oxidao de cidos Gordos Insaturados


A maioria dos cidos gordos nos TAG e fosfolpidos so insaturados, tendo uma
ou mais duplas ligaes (ex.: cido palmitoleico). Estas ligaes esto na
configurao cis e no podem sofrer aco da enoil-CoA hidratase. Duas enzimas
auxiliares so necessrias na oxidao de cidos gordos insaturados: uma
isomerase e uma redutase.

105
Brigitta Cismasiu | FMUL 2010/2011

-Oxidao no Peroxissomas
A -oxidao de cidos gordos de cadeia muito longa, isto , com 24 ou mais
tomos de carbono, realizada inicialmente no interior dos peroxissomas.
Quando a cadeia reduzida a um tamanho igual a 22 tomos de carbonos, a -
oxidao pode continuar no interior da mitocndria.

Por outro lado, se a dieta contiver muitos lpidos, a -oxidao peroxissomal Peroxissoma heptico
ocorre no fgado em taxas elevadas.
Este tipo de oxidao permite que
os cidos gordos sejam oxidados
mesmo em condies de baixa
concentrao de oxaloacetato,
circunstncia em que o acetil-CoA
mitocondrial, no conseguindo
entrar no ciclo TCA, se acumula para
formar corpos cetnicos.

Correlao Clnica

Deficincia de Carnitina
As mutaes ao nvel da carnitina podem afectar a carnitina aciltransferase I ou a
carnitina-acilcarnitina translocase.

A carnitina um transportador de cidos gordos de cadeia longa para dentro da


membrana mitocondrial interna. Se os seus nveis forem muito baixos, os cidos
gordos no passam para dentro da mitocndria, pelo que no se forma acil-CoA e,
consequentemente, este no vai seguir a via da -oxidao. No se formando
acetil-CoA, este no pode ser utilizado para cetognese. Os cidos gordos no so
metabolizados e acumulam-se no fgado, provocando hepatomeglia.

106
Brigitta Cismasiu | FMUL 2010/2011

Cetognese
Corpos Cetnicos

Se predominar a oxidao dos lpidos sobre o catabolismo glicdico (jejum ou


diabetes), a concentrao de oxaloacetato baixa (gluconeognese) e o acetil-CoA,
no podendo entrar no ciclo TCA, tem outro destino: convertido a acetoacetato,
-hidroxibutirato e acetona, designados de corpos cetnicos.

Observa-se tambm a formao anormal de corpos cetnicos quando o regime


alimentar exageradamente rico em lpidos, os quais fornecem um excesso de
acetil-CoA que a clula no consegue utilizar (dieta Atkins).

A formao de corpos cetnicos ocorre no fgado (e no crtex renal, em menor


quantidade) e a sua acumulao d-se no sangue (cetonemia) e na urina
(cetonria).

Cetognese

A sntese de corpos cetnicos ocorre na matriz mitocondrial. A acetona


produzida em pequenas quantidades e, sendo voltil, exalada (pulmes, lngua,
etc.). O acetoacetato e o -hidroxibutirato so transportados pelo sangue para
tecidos extra-hepticos, onde so convertidos a acetil-CoA e oxidados no ciclo
TCA, fornecendo muita da energia requerida pelos tecidos esqueltico, cardaco e
crtex renal. O crebro, que preferencialmente usa glucose, pode adaptar o seu
consumo ao acetoacetato ou -hidroxibutirato em situaes extremas de jejum
prolongado.

107
Brigitta Cismasiu | FMUL 2010/2011

Reaces da Cetognese
Condensao
Duas molculas de acetil-CoA condensam para formar acetoacetil-CoA. Esta
reaco, catalisada por uma tiolase, inversa da reaco final da -oxidao.

Condensao
O acetoacetil-CoA reage com um acetil-CoA, originando -hidroxi--metilglutaril-
CoA (HGM-CoA) e CoA, reaco catalisada pela HGM-CoA sintase.

Clivagem
O HGM-CoA clivado a acetil-CoA e acetoacetato pela HGM-CoA liase.

O acetoacetato reversivelmente reduzido pela -hidroxibutirato desidrogenase


a -hidroxibutirato. A extenso desta reaco depende da razo NADH/NAD+
intramitocondrial. Durante o jejum, essa razo relativamente elevada devido ao
NADH formado por -oxidao, favorecendo a formao de -hidroxibutirato.

Uma pequena quantidade de acetoacetato sofre descarboxilao espontnea ou


pela aco da acetoacetato descarboxilase, originando acetona, mas a sua
formao negligencivel em condies normais. No entanto, em concentraes
elevadas (como acontece em estados mais graves de diabetes), a acetona pode
atingir nveis de concentrao suficientes para ser detectada na respirao, pelo
hlito caracterstico.

Utilizao de Corpos Cetnicos


O principal local de produo de acetoacetato e -hidroxibutirato o fgado. Estas
substncias difundem-se das mitocndrias hepticas para o sangue e da so
transportados para os tecidos perifricos. O msculo cardaco e o crtex renal
usam acetoacetato preferencialmente glucose.

Em contrapartida, a glucose o principal combustvel do crebro e dos eritrcitos


em pessoas com uma dieta equilibrada. No entanto, o crebro adapta-se
utilizao de acetoacetato durante o jejum e diabetes. Em jejum prolongado, 75%
das necessidades de combustvel do crebro so supridas pelos corpos cetnicos.

108
Brigitta Cismasiu | FMUL 2010/2011

A -hidroxibutirato desidrogenase converte o -hidroxibutirato a acetoacetato


por uma reaco de oxidao, com formao de NADH. O acetoacetato ento
convertido a acetoacetil-CoA pela -cetoacil-CoA transferase, que usa o succinil-
CoA como fonte de CoA. Por fim, uma tiolase converte o acetoacetil-CoA em duas
molculas de acetil-CoA, que entram no ciclo TCA para produo de energia.

Diferenas entre Sntese e Utilizao de Corpos Cetnicos


As enzimas chave da sntese de corpos cetnicos, HMG-CoA sintase e HMG-CoA
liase, so expressas unicamente no fgado e no crtex renal.

A enzima chave da utilizao de corpos cetnicos, -cetoacil-CoA transferase,


est presente nos tecidos que consomem esses compostos e, portanto, ausente
no fgado.

Estas diferenas asseguram que os corpos cetnicos sejam sintetizados no fgado


e utilizados noutros tecidos.

Os Animais no Conseguem Converter cidos Gordos em Glucose


importante notar que os animais so incapazes de realizar a sntese de glucose a
partir de cidos gordos, isto , no podem converter o acetil-CoA em piruvato ou
oxaloacetato. A reaco que gera acetil-CoA a partir de piruvato irreversvel.
Dois tomos de carbono do grupo acetil do acetil-CoA entram no ciclo TCA, mas
dois tomos de carbono abandonam o ciclo nas descarboxilaes catalisadas pela
isocitrato desidrogenase e -cetoglutarato desidrogenase. Por conseguinte, o
oxaloacetato regenerado, mas no sintetizado de novo quando a unidade
acetil do acetil-CoA oxidada pelo ciclo TCA.

109
Brigitta Cismasiu | FMUL 2010/2011

Lipognese
Quando os glcidos provenientes da dieta se encontram em excesso relativamente
necessidade de produo de energia e sntese de glicognio so convertidos a
cidos gordos no fgado, durante o estado alimentado. A glucose a fonte de
carbono para a sntese de cidos gordos (via acetil-CoA) e de equivalentes
redutores (NADPH) necessrios a este processo. Outros substratos, como os
aminocidos, podem tambm contribuir para a lipognese.

Transporte Atravs da Membrana Mitocondrial

A sntese de cidos gordos ocorre no citosol, pelo que o acetil-CoA tem de


atravessar a membrana mitocondrial. Uma vez que esta no previamente
permevel a grupos acetil, o transporte de acetil-CoA atravs da membrana
interna realizado pelo citrato.

O citrato formado na matriz mitocondrial por condensao de acetil-CoA com


oxaloacetato. Quando est presente em concentraes elevadas, o citrato
transportado para o citosol, onde sofre clivagem pela ATP-citrato liase. O
potencial redutor NADH concomitantemente convertido a NADPH.

Assim, o acetil-CoA e o oxaloacetato so transferidos da mitocndria para o


citosol custa da hidrlise de uma molcula de ATP, com produo de NADPH.

O oxaloacetato formado na transferncia de grupos acetil para o citoplasma


devolvido para a mitocndria. A membrana mitocondrial interna impermevel
ao oxaloacetato. Assim, ocorre uma srie de reaces que geram a maior parte do
NADPH necessrio sntese de cidos gordos. Primeiro, o oxaloacetato reduzido
a malato, no citoplasma, pelo NADH, reaco catalisada por uma malato
desidrogenase. De seguida, o malato sofre descarboxilao oxidativa por aco da
enzima mlica. O piruvato formado atravessa a membrana mitocondrial, onde
carboxilado a oxaloacetato pela piruvato carboxilase.

Deste modo, uma molcula de NADPH gerada por cada molcula de acetil-CoA
transferida da mitocndria para o citosol. O NADPH adicional necessrio sntese
de cidos gordos proveniente da via dos fosfatos de pentose.

110
Brigitta Cismasiu | FMUL 2010/2011

Formao de Malonil-CoA
A sntese de cidos gordos a partir do acetil-CoA requer um intermedirio
activado, o malonil-CoA, que formado por carboxilao do acetil-CoA pela acetil-
CoA carboxilase. A reaco requer ATP e bicarbonato como fonte de CO2.

Inicialmente, o CO2 liga-se coenzima biotina, usando energia proveniente da


hidrlise do ATP; o CO2 ento transferido para o acetil-CoA.

Sntese de cidos Gordos

A sntese de cidos gordos d-se atravs de sucessivas adies de unidades com 2


tomos de carbono do acetil-CoA para a extremidade activada de uma cadeia em
crescimento, reaces catalisadas por um complexo multienzimtico, a sintase de
cidos gordos. O produto final um cido gordo saturado com 16 tomos de
carbonos, o cido palmtico, e todos os outros cidos gordos so produzidos
atravs da sua modificao.

111
Brigitta Cismasiu | FMUL 2010/2011

Reaces da Sntese de cidos Gordos


Elongao
Os grupos acil do acetil-CoA ou do malonil-CoA so transferidos por uma acetil
transferase ou uma malonil transferase, respectivamente, para a protena
transportadora de acilo (ACP), que tem propriedades semelhantes CoA,
formando acetil-ACP e malonil-ACP.

Condensao
O acetil-ACP e o malonil-ACP reagem para formar acetoacetil-ACP, reaco
catalisada pela enzima condensadora.

Reduo
O acetoacetil-ACP reduzido a D--hidroxibutiril-ACP pela -cetoacil-ACP
redutase.

Desidratao
O D--hidroxibutiril-ACP desidratado pela -hidroxiacil-ACP desidratase para
formar crotonil-ACP.

Reduo
O crotonil-ACP reduzido pelo NADPH a butiril-ACP, atravs da aco da enoil-
ACP redutase.

Elongao
O butiril-ACP reage com outro malonil-ACP (que foi entretanto transferido para a
enzima condensador) e, aps outra sequncia de reaces, origina uma cadeia de
cido gordo alongada de dois tomos de carbono. O ciclo repete-se por mais cinco
vezes, formando palmitoil-EC (16 C).

Hidrlise
O palmitoil-ACP sofre a aco da tioesterase, originando palmitato livre.

O produto da sntese convertido a palmitoil-CoA no retculo endoplasmtico,


preparando-o para modificao ou incorporao em lpidos complexos.

Sintase de cidos Gordos


A sintase de cidos gordos dos mamferos contm duas subunidades iguais,
estando cada uma delas desdobrada em trs domnios que, no seu conjunto,
apresentam 7 centros catalticos.
O domnio 1 a unidade de ligao ao substrato e de condensao. Contm
a acetil transferase, a malonil transferase e a -cetoacil sintase (enzima
condensadora);
O domnio 2 a unidade de reduo. Contm a protena transportadora de
acilo, a -cetoacil redutase, a -hidroxiacil desidratase e a enoil redutase;
O domnio 3 a unidade de libertao do palmitato e contm a tioesterase.

112
Brigitta Cismasiu | FMUL 2010/2011

Esta associao permite a coordenao mais rpida e eficaz de vrias snteses


enzimticas. Alm disso, as molculas que sofrem a sua aco so muito
hidrfobas, pelo que no se conseguiriam difundir facilmente por vrias enzimas
individuais.

Etapa Limitante Feedback


Parte do sinal de sada de um
A acetil-CoA carboxilase catalisa a etapa limitante da sntese de cidos gordos. sistema transferida para a
Esta enzima activada pelo citrato ou isocitrato, o que representa processo de entrada deste mesmo
sistema, com o objectivo de
feedforward da lipognese (o citrato exportado da mitocndria para produzir diminuir, amplificar ou
acetil-CoA citoslico que inicia essa via metablica), e inibida por acil-CoA de controlar a sada do sistema.
cadeia longa, resultando numa inibio por feedback da lipognese pelo produto
final desta via, e AMP. Feedforward
Resposta a uma alterao no
ambiente de um sistema, de
Ocorre igualmente uma regulao por fosforilao-desfosforilao: a fosforilao modo a manter o estado
por uma protena cinase converte a enzima para a forma inactiva e a aco da desejado (actual) desse
fosfatase torna-a activa. sistema.

Muitas das enzimas que fosforilam a acetil-CoA carboxilase so controladas por


sinais hormonais. A insulina estimula a sntese de cidos gordos enquanto a
glucagina e a epinefrina tm o efeito inverso.

113
Brigitta Cismasiu | FMUL 2010/2011

Sntese de TAG

Quase todos os tecidos convertem cidos gordos em TAG, mas esse processo
acontece em maior extenso nos tecidos heptico, adiposo e muscular. O tecido
adiposo um rgo especializado na sntese, armazenamento e hidrlise de TAG
e o principal local de reserva de energia a longo prazo. Os TAG so armazenados
no citosol dos adipcitos em gotculas rodeadas por uma monocamada de lpidos
membranares e perilipina, em contnua sntese e degradao. Os msculos
esqueltico e cardaco tambm armazenam TAG, mas apenas para consumo
prprio.

Os cidos gordos envolvidos neste processo provm da dieta, do tecido adiposo


atravs da corrente sangunea ou da sntese de novo (que segue o catabolismo
glicdico).

Os TAG so sintetizados na maioria dos tecidos a partir de acil-CoA e glicerol 3-


fosfato, um percursor do glicerol. O glicerol 3-fosfato proveniente da reduo
da diidroxiacetona fosfato (no estado alimentado) ou da gliceroneognese (em
jejum). No fgado, existe uma fonte adicional deste composto: a fosforilao do
glicerol pela glicerol cinase.

Os cidos gordos so activados atravs da sua converso a steres de CoA. Na


sntese de TAG ocorrem duas acilaes sequenciais do glicerol 3-fosfato que
originam cido lisofosfatdico e cido fosfatdico. O cido fosfatdico hidrolisado,
com a remoo do seu grupo fosfato pela fosfatidato fosfatase, para produzir
diacilglicerol, que posteriormente acilado a triacilglicerol.

114
Brigitta Cismasiu | FMUL 2010/2011

A sntese de TAG segue uma via diferente nas clulas epiteliais do intestino
grosso. Estas clulas usam 2-monoacilglicerol e cidos gordos livres, resultantes
da digesto de TAG pela lipase pancretica. Uma enzima da mucosa intestinal
acila estes monoacilgliceris usando acil-CoA como substrato. Os diacilgliceris
resultantes so ento acilados a triacilgliceris, que so armazenados em
quilomicra.

A mobilizao cidos gordos armazenados para produzir energia iniciada com a


hidrlise dos TAG. Vrias lipases catalisam essa reaco e a sequncia de hidrlise
das trs cadeias acilo determinada pela especificidade das lipases envolvidas.

Gliceroneognese

A libertao de cidos gordos livres do tecido adiposo uma adaptao


metablica crtica ao jejum. No entanto, a quantidade de cidos gordos libertados
excede a quantidade usada para produo de energia nos outros tecidos. Assim, a
maioria desses cidos gordos so reintegrados no tecido adiposo como TAG.
Tanto o fgado como o tecido adiposo desempenham papis importantes nesse
processo.

Num estado alimentado, o glicerol 3-fosfato para a sntese de TAG provm da


glucose atravs da gliclise. Em jejum, a entrada de glucose no tecido adiposo
est limitada porque a concentrao de insulina baixa e a glucose utilizada por
outros tecidos. Neste estado, o glicerol 3-fosfato sintetizado atravs da
gliceroneognese.

115
Brigitta Cismasiu | FMUL 2010/2011

Esta via , essencialmente, uma abreviao da gluconeognese, na qual o malato


formado no ciclo TCA abandona a mitocndria e convertido a oxaloacetato no
citosol. A fosfoenolpiruvato carboxicinase converte o oxaloacetato a
fosfoenolpiruvato, que convertido a diidroxiacetona e da a glicerol 3-fosfato,
que usado na sntese de TAG.

116
Brigitta Cismasiu | FMUL 2010/2011

Sntese de Fosfolpidos
Existem duas classes principais de fosfolpidos membranares: glicerofosfolpidos e
esfingolpidos. possvel construir muitas espcies diferentes de fosfolpidos
atravs da combinao de vrios cidos gordos e cabeas polares com um
esqueleto de glicerol ou esfingosina.

Em geral, a montagem de fosfolpidos a partir de precursores simples requer:


1. Sntese do esqueleto (glicerol ou esfingosina);
2. Ligao de cidos gordos ao esqueleto, atravs de uma ligao ster ou
amida;
3. Adio de uma cabea hidrfila ao esqueleto, atravs de uma ligao
fosfodister;
4. Alterao ou troca da cabea para originar o produto fosfolipdico final
(nalguns casos).

Nas clulas eucariticas, a sntese de fosfolpidos ocorre principalmente na


superfcie do retculo endoplasmtico liso e da membrana mitocondrial interna.
Alguns fosfolpidos recm-formados permanecem no local de sntese, mas a
maioria destinada a outras localizaes celulares.

117
Brigitta Cismasiu | FMUL 2010/2011

Colesterol
O colesterol um esteride que modula a fluidez das membranas das clulas
animais e o precursor das hormonas esterides como progesterona,
testosterona, estradiol e cortisol.

Sntese de Colesterol

Todos os 27 tomos de carbono do colesterol so derivados de um nico


percursor, o acetil-CoA, que provm de vrias fontes: (1) -oxidao de cidos
gordos, (2) oxidao de aminocidos cetognicos, como a leucina e a isoleucina, e
(3) reaco da piruvato desidrogenase.

O poder redutor sob a forma de NADPH fornecido, principalmente, pela glucose


6-fosfato desidrogenase e pela 6-fosfogluconato desidrogenase atravs da via dos
fosfatos de pentose.

A sntese de colesterol ocorre no citosol e no retculo endoplasmtico e dirigida


pela hidrlise de ligaes tioster de alta energia do acetil-CoA e ligaes
fosfoanidrido do ATP.

118
Brigitta Cismasiu | FMUL 2010/2011

Fases da Sntese do Colesterol


Sntese de mevalonato a partir de acetato
I. Duas molculas de acetil-CoA sofrem condensao, catalisada por uma
tiolase, formando acetoacetil-CoA;
II. O acetoacetil-CoA condensa com uma terceira molcula de acetil-CoA,
originando -hidroxi--metilglutaril-CoA (HGM-CoA), reaco catalisada
pela HGM-CoA sintase. Esta isoenzima citoslica distinta da isoenzima
mitocondrial que catalisa a sntese de HGM-CoA na formao de corpos
cetnicos;
III. O HGM-CoA reduzido por duas molculas de NADPH a mevalonato, com
interveno da HMG-CoA redutase, uma protena membranar integral do
retculo endoplasmtico liso, que o principal ponto de regulao desta via
metablica.

Converso do mevalonato a isopentenil-pirofosfato


I. Trs molculas de ATP transferem trs grupos fosfato para o mevalonato,
formando mevalonil 3-fosfato-5-pirofosfato;
II. O fosfato ligado a C3 e um grupo carboxilo abandonam a molcula,
originando isopentenil pirofosfato (5 C);
III. O isopentenil pirofosfato pode isomerizar, formando dimetilalil pirofosfato.

Polimerizao de cinco molculas de isopentenil-pirofosfato para formar


esqualeno
I. O isopentenil pirofosfato e o dimetilalil pirofosfato sofrem uma
condensao cabea-cauda, formando geranil pirofosfato (10 C);
II. O geranil pirofosfato condensa com outro isopentenil pirofosfato,
originando farnesil pirofosfato (15 C);
III. Duas molculas de farnesil pirofosfato sofrem condensao cabea-cabea,
formando esqualeno (30 C).

Ciclizao do esqualeno e converso do ncleo esteride em colesterol


I. A esqualeno monooxigenase acrescenta um tomo de oxignio do O2
extremidade da cadeia do esqualeno, formando esqualeno 2,3-epxido. O
NADPH reduz o outro tomo de oxignio a H2O.
II. As duplas ligaes do esqualeno 2,3-epxido esto posicionadas de tal
forma que formada uma estrutura cclica, o lanosterol, que contm 4
anis caractersticos do ncleo esteride;
III. O lanosterol finalmente convertido a colesterol por uma srie de 20
reaces, que incluem a migrao e a remoo de grupos metil.

A maior parte do colesterol sintetizada no fgado. Uma pequena fraco


incorporada nas membranas dos hepatcitos, mas a maioria exportada sob a
forma de colesterol biliar, cidos biliares ou steres de colesteril.

Os cidos biliares e seus sais so derivados de colesterol relativamente hidrfilos


que ajudam na digesto lipdica. Os steres de colesteril so formados pela aco
da acil-CoA-colesterol aciltransferase, enzima que catalisa a transferncia de um
cido gordo a partir da CoA para o grupo hidroxilo do colesterol, tornando-o mais
hidrfobo. Os steres de colesteril so transportados em lipoprotenas at
tecidos que utilizem colesterol ou so armazenados no fgado.

119
Brigitta Cismasiu | FMUL 2010/2011

Regulao da Sntese do Colesterol


A sntese de colesterol um processo complexo e energeticamente dispendioso,
por isso vantajoso que um organismo o regule de modo a complementar a
ingesto alimentar do colesterol.

Nos mamferos, a produo do colesterol regulada pela concentrao de


colesterol intracelular e das hormonas glucagina e insulina. O passo limitante na
via do colesterol (e um importante local de regulamentao) a converso de
HMG-CoA em mevalonato, reaco catalisada pela HMG-CoA redutase.

A HMG-CoA redutase regulada a vrios nveis:


1. A taxa de transcrio do gene que codifica a HMG-CoA redutase
controlada pela protena de ligao ao elemento de resposta a esteris
(PLERE). Este factor de transcrio liga-se a uma pequena sequncia de
DNA situada no lado 5 do gene da redutase, o elemento de resposta a
esteris (ERE). Na forma inactiva, a PLERE encontra-se na membrana do
retculo endoplasmtico, associada a uma protena sensora de colesterol.
Quando os nveis de colesterol diminuem, a PLERE liga-se ao ERE do gene da
HMG-CoA redutase, bem como a vrios outros genes codificantes de
enzimas que medeiam a absoro e sntese de colesterol e cidos gordos
insaturados, amplificando a sua transcrio.
2. A taxa de traduo do mRNA da HMG-CoA redutase inibida por
metabolitos derivados do mevalonato e pelo colesterol proveniente da
dieta.
3. A degradao da HMG-CoA redutase rigorosamente controlada. Esta
enzima tem dois domnios: o seu domnio citoplasmtico realiza catlise e o
seu domnio membranar recebe sinais que levam sua degradao. O
domnio membranar pode sofrer alteraes estruturais em resposta ao
aumento da concentrao de esteris, como o colesterol, que tornam a
enzima mais susceptvel protelise.
4. A fosforilao diminui a actividade da HMG-CoA redutase. Esta enzima
inibida pela protena cinase activada por AMP (AMPK). Assim, a sntese de
colesterol cessa quando o nvel de ATP baixo.

120
Brigitta Cismasiu | FMUL 2010/2011

Transporte de Colesterol e TAG pelo Organismo

O colesterol e os steres de colesteril, como os triglicerdeos e os fosfolpidos, so


essencialmente insolveis em gua, mas precisam de ser transportados a partir do
tecido de origem para os tecidos nos quais sero armazenados ou consumidos. O
seu transporte pelo plasma sanguneo faz-se sob a forma de lipoprotenas,
complexos macromoleculares de protenas
transportadoras especficas, apolipoprotenas, com
vrias combinaes de fosfolpidos, colesterol, steres de
colesteril e TAG.

As apolipoprotenas so sintetizadas e segregadas pelo


fgado e pelo intestino e tm como funes a
solubilizao dos lpidos hidrfobos e a sinalizao
celular.

As lipoprotenas so classificadas, de acordo com a densidade crescente, em:


quilomicras, quilomicras remanescentes, lipoprotenas de muito baixa
densidade (VLDL), lipoprotenas de densidade intermdia (IDL), lipoprotenas de
baixa densidade (LDL) e lipoprotenas de alta densidade (HDL).

121
Brigitta Cismasiu | FMUL 2010/2011

Ciclo Exgeno
Os TAG, o colesterol e outros lpidos obtidos da dieta so transportados a partir
do intestino sob a forma de quilomicra. Estas partculas tm uma densidade
muito baixa porque os TAG constituem aproximadamente 90% do seu contedo.
A apolipoprotena B- 48 (apo B-48) constitui um invlucro esfrico anfiptico em
redor do glbulo de gordura; a face externa deste invlucro hidrfila. Os TAG
so libertados por hidrlise pelas lipoprotena lipases. Estas enzimas esto
localizadas no revestimento dos vasos sanguneos dos msculos e outros tecidos
que utilizam cidos gordos como combustvel e na sntese de lpidos. O fgado
absorve os resduos ricos em colesterol, originando quilomicra remanescentes.

Ciclo Endgeno
Os TAG e o colesterol hepticos em excesso relativamente s prprias
necessidades do fgado so exportados para o sangue sob a forma de VLDL. Essas
partculas so estabilizadas por duas apolipoprotenas: apoB -100 e apoE. Os TAG
so igualmente hidrolisados por lipases nas superfcies capilares. Os produtos
remanescentes, ricos em steres de colesteril, so as IDL, que tm dois destinos:
metade absorvida pelo fgado para processamento e a outra metade
convertida a LDL, pela remoo de mais TAG.

As LDL so os principais
transportadores de colesterol no
sangue. O seu papel o
transportar colesterol para os
tecidos perifricos e regular a
sntese de novo de colesterol
nesses locais.

J as HDL captam o colesterol


libertado no plasma por clulas em necrose ou apoptose e por membranas
celulares em reciclagem, um processo denominado transporte reverso do
colesterol. Uma aciltransferase presente nas HDL esterifica esse colesterol, que
transportado para o fgado.

122
Brigitta Cismasiu | FMUL 2010/2011

Endocitose Mediada por Receptores


Cada partcula de LDL na corrente sangunea contm apoB-100, que reconhecida
por receptores da superfcie especficos, receptores de LDL, em clulas que
consomem colesterol. A ligao de LDL a um receptor inicia a endocitose, que
incorpora a LDL e seu receptor na clula, dentro de um endossoma.

O endossoma funde-se, eventualmente, com uma lisossoma, que contm enzimas


capazes de hidrolisar steres de colesteril, libertando colesterol e cidos gordos
para o citosol. A apoB-100 da LDL tambm degradada a aminocidos, que so
libertados para o citosol, mas o receptor de LDL escapa degradao e
devolvido superfcie da clula, para funcionar novamente na captao de LDL.

O colesterol que entra nas clulas por esta via pode ser incorporado em
membranas ou reesterificado pela acil-CoA-colesterol aciltransferase para
armazenamento sob forma de gotculas lipdicas citoslicas.

A acumulao de colesterol intracelular em excesso evitada pela reduo da


taxa de sntese do colesterol, quando este est disponvel em quantidade
suficiente a partir de LDL no sangue.

O receptor de LDL tambm se liga apoE e desempenha um papel significativo na


captao heptica dos quilomicra e VLDL remanescentes. No entanto, se os
receptores de LDL no esto disponveis, estas lipoprotenas continuam a ser
captadas pelo fgado, embora a LDL no. Isto indica a presena de um sistema de
back-up de endocitose mediada por receptores de quilomicra e VLDL
remanescentes, como, por exemplo, a protena relacionada a receptor de
lipoprotenas (LRP), que liga apoE, bem como a uma srie de outros ligantes.

123
Brigitta Cismasiu | FMUL 2010/2011

Aterosclerose e Doenas Cardacas


Nveis sricos elevados de colesterol contribuem para a formao de placas
aterosclerticas. Este excesso de colesterol est presente na forma de partculas
de LDL.

As HDL tm como funo o transporte de colesterol pelo organismo. Estas


lipoprotenas ligam e esterificam o colesterol libertado dos tecidos perifricos e
transferem steres de colesteril para o fgado ou os tecidos que utilizam o
colesterol para sintetizar hormonas esterides. Um receptor especfico medeia o
encaixe da HDL nesses tecidos.

A razo de colesterol sob a forma de LDL para HDL pode ser usado para avaliar a
susceptibilidade de desenvolvimento de doenas cardacas.

Se as LDL ficarem muito tempo em circulao (por exemplo, por no


aparecimento dos receptores superfcie das clulas hipercolesterolmia
familiar), ficam sujeitas oxidao por espcies reactivas de O2 e provocam danos
na parede dos capilares. Os macrfagos endocitam as LDL em excesso na
circulao, formando clulas espumosas. Estas clulas acumulam-se num
determinado local num capilar, formando redes de fibrinognio e libertando
citocinas, o que leva diminuio progressiva do dimetro da parede do vaso e,
por fim, ao seu bloqueio (aterosclerose).

124
Brigitta Cismasiu | FMUL 2010/2011

cidos Biliares
Os cidos biliares, produto do metabolismo do colesterol, podem ser primrios
(sintetizados nos hepatcitos) e secundrios (modificao no intestino delgado).

Os cidos biliares so sintetizados no fgado, armazenados e concentrados na


vescula biliar, e em seguida libertados no intestino delgado. So os principais
componentes da blis e solubilizam os lpidos da dieta. A solubilizao aumenta a
rea superficial efectiva de lpidos, com duas consequncias: (1) uma maior
superfcie exposta aco digestiva das fosfolipases e (2) os lpidos so mais
facilmente absorvidos pelo intestino.

125
Brigitta Cismasiu | FMUL 2010/2011

Correlao Clnica

Hipercolesterolmia Familiar
causada por uma mutao no receptor da LDL, pelo que a apo100 no vai ser
reconhecida e o colesterol no especificamente endocitado, acumulando-se no
sangue.

A abordagem passa pela administrao de teraputica farmacolgica


antidislipidmia, com um inibidor da HMG-CoA redutase, que diminui a sntese
endgena de colesterol, e dieta, que elimina a ingesto de colesterol exgeno.

126
Brigitta Cismasiu | FMUL 2010/2011

Ciclo do Azoto
O azoto existe predominantemente num estado oxidado no meio ambiente,
principalmente como N2, na atmosfera, ou como io nitrato (NO3-), nos solos e
oceanos. A sua aquisio por sistemas biolgicos acompanhada pela sua
reduo ao io amnio (NH4+) e incorporao em ligaes orgnicas como grupos
amina ou amida.

Nenhum animal capaz de qualquer fixao de azoto ou assimilao de nitrato,


pelo que so totalmente dependentes de plantas e microrganismos para a sntese
de compostos orgnicos azotados, como aminocidos e protenas.

Os animais libertam o azoto em excesso sob a forma reduzida, quer como NH4+
quer como compostos orgnicos azotados, como a ureia. A libertao de azoto
ocorre tanto durante a vida como em consequncia da decomposio
microbiolgica aps a morte.

127
Brigitta Cismasiu | FMUL 2010/2011

Metabolismo dos Aminocidos


Alm de serem os constituintes das protenas, os aminocidos so tambm os
precursores de outros compostos azotados, aos quais podem fornecer no s o
seu tomo de azoto, mas tambm os seus tomos de carbono.

Transaminao

Trata-se da transferncia reversvel do grupo amina de um aminocido para um


com um -cetocido.

As aminotransferases catalisam a transferncia de um grupo -amina de um


aminocido para um -cetocido, originando um novo aminocido e um novo -
cetocido.

A glutamato aminotransferase catalisa a transferncia de um grupo amina para o


-cetoglutarato, com formao de glutamato, na via do catabolismo.

Existe um grande nmero de aminotransferases, especficas dos diferentes


aminocidos, sendo algumas muito abundantes nos tecidos animais. O glutamato
o principal dador de grupos amina e participa em duas importantes reaces,
catalisadas pela aspartato aminotransferase e pela alanina aminotransferase.

Nveis sricos
aumentados de ALT e
AST esto associados
a danos celulares ao
nvel do fgado e
corao.

128
Brigitta Cismasiu | FMUL 2010/2011

Estas duas aminotransferases canalizam, portanto, os grupos amina do aspartato


e da alanina para a formao de glutamato, a partir do qual, no catabolismo, se
formar NH3 por desaminao oxidativa.

Na sntese, o grupo amina do glutamato pode ser transferido para um -


cetocido para formar o aminocido correspondente.

O glutamato , com efeito, um participante preponderante nas reaces de


transaminao.

Desaminao

Pode ocorrer por um processo geral desaminao oxidativa ou por alguns


processos particulares, especficos de determinados aminocidos.

Desaminao Directa
Os grupos amina da serina, da cistena e da treonina podem ser convertidos
directamente a amonaco, visto estes aminocidos conterem grupos hidroxilo ou
sulfidrilo em cadeias laterais.

Tanto na desaminao da L-serina pela aco da serina desidratase, como na da L-


cistena pela aco da cistena dessulfidrase, forma-se piruvato. O grupo
prosttico destas enzimas o pirofosfato de piridoxal.

O mecanismo reaccional de desaminao da treonina semelhante, mas o outro


produto final, alm do amonaco, o -cetobutirato (o qual pode originar
piruvato atravs da aminoacetona).

Desaminao Reversvel
O aspartato pode ser convertido a fumarato por desaminao reversvel.

Trata-se de um processo particular de desaminao. Visto ser uma reaco


reversvel, permite igualmente a fixao de NH3 num composto orgnico;
catalisada pela aspartase.

O fumarato constitui tambm um ponto de entrada no ciclo TCA de metade dos


tomos de carbono da tirosina e da fenilalanina.

129
Brigitta Cismasiu | FMUL 2010/2011

Desaminao Oxidativa
Este processo, essencialmente irreversvel, decorre em vrias etapas. Numa
primeira fase ocorre uma desidrogenao do aminocido a iminocido, reaco
catalisada por uma flavoprotena, e os dois H retirados ao substrato vo reduzir o
FAD a FADH2. O iminocido hidrolisa-se em -cetocido e amonaco. As enzimas
envolvidas neste processo so as aminocido oxidases e a coenzima reduzida
geralmente reoxidada pelo oxignio molecular com formao de H2O.

O glutamato desaminado pela L-glutamato desidrogenase, enzima que catalisa


a reaco reversvel. Esta desaminao corre principalmente no fgado e no rim.

A interconverso entre o glutamato e o -cetoglutarato por desaminao


oxidativa ou por aminao redutora constitui um importante ponto de contacto
entre os metabolismos glicdico e proteico.

A L-glutamato desidrogenase pode utilizar tanto o NAD+ como o NADP+ como


coenzima. O NAD+ utilizado na reaco de desaminao oxidativa, enquanto que
o NADP+ utilizado na aminao redutora.

O sentido da reaco depende das concentraes relativas do glutamato, -


cetoglutarato e amnia e da razo NAD+/NADP+. Por exemplo, aps a ingesto de
uma refeio contendo protenas os nveis de glutamato no fgado so elevados e
a reaco ocorre no sentido de degradao de aminocidos e da formao de
amnia. Esta reaco tambm pode ser utilizada para sintetizar aminocidos a
partir dos -cetocidos correspondentes.

Catabolismo de Aminocidos

A degradao de aminocidos ocorre maioritariamente no fgado. Permite a


eliminao de aminocidos em excesso e, em situaes de carncia energtica, a
utilizao de aminocidos como fonte de energia.

Este processo catablico compreende a remoo inicial dos grupos -amina, que
podem ser canalizados atravs do glutamato, originando cetocidos que podem
ser catabolisados para produo de energia.

130
Brigitta Cismasiu | FMUL 2010/2011

Posteriormente, os esqueletos de carbono resultantes so transformados em


acetil-CoA, acetoacetil-CoA, piruvato ou intermedirios do ciclo TCA. Assim,
possvel formar cidos gordos, glucose e corpos cetnicos a partir dos
aminocidos.

Destino do Grupo Amina


O grupo -amina de muitos aminocidos transferido para o -cetoglutarato,
originando glutamato. Este pode sofrer uma desaminao oxidante, libertando
NH3.

Ciclo da Ureia
O excesso de amnia txico para os tecidos, exigindo uma regulao fina dos
seus nveis no sangue. Em humanos, o estadio terminal de intoxicao por amnia
caracteriza-se pela entrada num estado comatoso acompanhado por outros
sintomas a nvel do crebro.

Estratgias para a toxicidade da amnia:


sequestrao da amnia na mitocndria a sua formao e posterior
utilizao ocorrem neste organelo;
excreo do excesso de amnia;
converso em compostos no txicos.

So vrias as estratgias apresentadas por diferentes animais para a excreo do


excesso de amnia.

Decorrendo exclusivamente no fgado, o ciclo da ureia permite a converso da


amnia em ureia. Pela circulao sangunea, esta posteriormente enviada ao
rim, onde excretada na urina.

O ciclo da ureia comea dentro das mitocndrias hepticas, mas as trs etapas
subsequentes ocorrem no citosol. O primeiro grupo amina que entra no ciclo da
ureia derivado da amnia da matriz mitocondrial. O fgado tambm recebe
alguma amnia atravs da veia porta do intestino, a partir da oxidao bacteriana
de aminocidos.

131
Brigitta Cismasiu | FMUL 2010/2011

As Reaces do Ciclo da Ureia


Condensao
A amnia livre presente no interior das mitocndrias condensa-se com o CO2
(como HCO3-), produto do processo de respirao celular, para formar o
carbamoil-fosfato. Essa reaco catalisada pela enzima carbamoil-fosfato
sintetase I, presente nas mitocndrias e requer a clivagem de duas molculas de
ATP.

Para alm da isoforma mitocondrial, existe ainda uma isoforma citoplasmtica, a


CPS II, envolvida na biossntese de nucletidos.

Condensao (entrada do 1o grupo amina)


O carbamoil-fosfato doa o seu grupo carbamoil ornitina para formar a citrulina,
com libertao de Pi (a ornitina desempenha um papel semelhante ao do

132
Brigitta Cismasiu | FMUL 2010/2011

oxaloacetato no ciclo TCA, aceitando material em cada volta do ciclo). A reaco


catalisada pela ornitina transcarbamoilase e a citrulina passa da mitocndria para
o citosol.

Condensao (entrada do 2o grupo amina)


O grupo amina do aspartato (gerado na mitocndria por transaminao e
transportado para o citosol) condensa com o grupo carbonilo da citrulina,
formando argininosuccinato. Esta reaco citoslica, catalisada pela
argininosuccinato sintetase, requer a clivagem de uma molcula de ATP em AMP
e pirofosfato (PPi).

Clivagem
O argininosuccinato clivado pela argininosuccinase para formar arginina livre e
fumarato. A arginina formada serve como percursor imediato da ureia e o
fumarato entra na mitocndria para se juntar aos restantes intermedirios do
ciclo TCA. Esta a nica etapa reversvel no ciclo da ureia.

Clivagem
A arginase citoslica cliva a arginina para produzir ureia e ornitina. A ornitina
transportada para a mitocndria para iniciar uma nova volta do ciclo da ureia.
Essa enzima encontra- se quase exclusivamente no fgado. Dessa forma, enquanto
outros tecidos, como o rim, podem sintetizar arginina, apenas o fgado pode clivar
a arginina e sintetizar ureia.

A ureia sai do fgado por difuso e transportada no sangue at aos rins, onde
filtrada e excretada na urina. Parte da ureia difunde do sangue para o intestino
onde clivada pela enzima urease bacteriana, formando CO2 e NH3 que , em
parte, excretado nas fezes e, em parte, reabsorvido para o sangue.

Em pacientes com insuficincia renal, os nveis de ureia no sangue aumentam,


aumentando tambm o CO2 e NH3 formado no intestino, o que contribui para a
hiperamoninmia frequentemente observada nesses pacientes. A administrao
oral de neomicina (um antibitico), reduz o nmero de bactrias intestinais
responsveis pela produo de NH3.

Regulao do Ciclo da Ureia


O N-acetil-glutamato um activador essencial da carbamoil-fosfato-sintetase I, o
passo limitante do ciclo da ureia. O N-acetil-glutamato sintetizado a partir do
acetil-CoA e do glutamato, numa reaco activada pela arginina. Assim sendo, a
concentrao intra-heptica de N-acetil-glutamato aumenta aps a ingesto de
uma refeio rica em protenas, favorecendo tanto o substrato (glutamato) como
o regulador da sntese de N-acetil-glutamato. Isso leva a um aumento de
velocidade da sntese da ureia.

Interligao Metablica
A interligao do ciclo da ureia com o ciclo TCA assegura a produo de NADH e
sua utilizao na produo de ATP por fosforilao oxidativa, o que contribui para
a diminuio do custo energtico do ciclo da ureia.

133
Brigitta Cismasiu | FMUL 2010/2011

Defeitos metablicos no ciclo da ureia conduzem ao bloqueio desta via


metablica, gerando um excesso de amnia no organismo. Na deficincia da
enzima argininosuccinato sintetase, a suplementao da dieta alimentar com
arginina possibilita a formao de ornitina e a sua reaco com o carbamoil-
fosfato. Assim, obvia-se parcialmente a eliminao de excesso de amnia.

Para alm do fgado, a degradao de aminocidos pode ainda ocorrer em tecidos


extra-hepticos, como, por exemplo, no msculo. O msculo pode utilizar os
aminocidos como fonte de energia em diversas situaes, como durante
exerccio prolongado ou em jejum. Porm, e como em outros tecidos extra-
hepticos, o msculo no expressa as enzimas do ciclo da ureia.

Destino do Esqueleto de Carbono


A degradao dos aminocidos vai conduzir formao de importantes
intermedirios metablicos que podem ser convertidos em glucose ou oxidados
pelo ciclo TCA.

As converses metablicas que afectam o esqueleto de carbono dos aminocidos


comuns das protenas vo conduzi-los a piruvato, acetil-CoA, acetoacetil-CoA,
fumarato e oxaloacetato.

Consoante os compostos que originam por degradao, os aminocidos so


classificados em dois grupos:
Cetognicos, se originam compostos cetnicos;
Glucognicos, se originam piruvato ou intermedirios do ciclo TCA.

Os aminocidos glucognicos podem, portanto, ser convertidos em


fosfoenolpiruvato e, por fim, em glucose ou glicognio.

O nico aminocido com carcter puramente cetognico a leucina.

134
Brigitta Cismasiu | FMUL 2010/2011

O mesmo aminocido pode


ser simultaneamente
cetognico e glucognico.

Piruvato: alanina, serina, glicina, cistena e treonina;


Oxaloacetato: aspartato e asparagina;
-Cetoglutarato: glutamina, prolina, arginina e histidina;
Succinil-CoA: metionina, isoleucina, treonina e valina;
Corpos cetnicos: leucina e isoleucina;
Fumarato e acetoacetato: fenilalanina e tirosina (aminocidos aromticos).

A degradao da fenilalanina, por hidroxilao pela fenilalanina hidroxilase,


origina tirosina. H interveno de um transportador de electres, a
tetrahidrobiopterina (THB) passa forma diidrobiopterina (DHB) por oxidao. Os
produtos finais so o fumarato e o acetoacetato.

Ciclo da Alanina-Glucose
A alanina obtida por transaminao do glutamato
com o piruvato. Pelo sangue, a alanina
transportada para o fgado onde transaminada
em glutamato, que pode ser convertido em ureia. O
piruvato resultante convertido em glucose, que
conduzida ao msculo.

A glutamina constitui outro transportador de azoto


importante para a detoxificao da amnia
produzida em tecidos extra-hepticos. A glutamina
tambm transportada para o rim, onde
convertida em amnia que directamente
excretada pela urina.

135
Brigitta Cismasiu | FMUL 2010/2011

Sntese de Aminocidos
Os mamferos apenas conseguem sintetizar 10 dos 20 aminocidos comuns; os
outros devem ser obtidos na dieta. Aqueles que podem ser sintetizados so
classificados como no-essenciais, o que significa que no essencial que esses
aminocidos faam parte da dieta. Em contraste, so incapazes de construir os
esqueletos de carbono dos aminocidos essenciais, e, assim, esto dependentes
das fontes alimentares para esses metabolitos. O excesso de aminocidos da dieta
no pode ser armazenado para uso futuro, nem excretado quando no
utilizados. Em vez disso, convertido em intermedirios metablicos que podem
ser oxidados pelo ciclo TCA ou usados para formar glucose.

A assimilao de amnia por plantas e microorganismos efectuada por aco do


ciclo da glutamato sintase, em condies em que a amnia limitante. No
entanto, essa enzima no se encontra presente em animais.

Os animais utilizam frequentemente a glutamina e o glutamato como fontes de


azoto na biossntese de aminocidos e de outros compostos azotados.

A glutamina funciona frequentemente como fonte de azoto para cadeias laterais


de -aminocidos e para outros compostos azotados. O glutamato constitui a
fonte de grupos -amino da maioria dos aminocidos atravs de reaces de
transaminao, constituindo ainda a fonte de azoto noutras reaces.
-aminocido
Para uma maioria dos aminocidos, a adio de grupos -amino envolve reaces
de transaminao no sentido inverso ao utilizado para a remoo dos mesmos
grupos aquando do catabolismo de aminocidos. As reaces de transaminao
asseguram apenas a formao do ismero L.

Os esqueletos de carbono dos aminocidos provm de intermedirios


metablicos da (1) gliclise, (2) da via dos fosfatos de pentose e (3) do ciclo TCA.

A biossntese de aminocidos, est interligada com vrias vias metablicas


permitindo a interconverso entre aminocidos e outras biomolculas.

136
Brigitta Cismasiu | FMUL 2010/2011

O 3-fosfoglicerato, intermedirio da gliclise, pode ser convertido em serina. A


serina pode ser convertida em glicina pela reaco catalisada pela enzima serina
hidroximetil transferase na presena de tetrahidrofolato.

137
Brigitta Cismasiu | FMUL 2010/2011

No fgado de vertebrados, a glicina pode ainda ser obtida numa reaco catalisada
pela glicina sintase na presena de derivado do tetrahidrofolato.

O tetrahidrofolato (derivado do cido flico) constitui um cofactor importante em


reaces que ocorram com a transferncia de grupos monocarbnicos em vrios
estados de oxidao.

A serina pode ser convertida em cistena por reaco com a homocistena


envolvendo enzimas dependentes do cofactor PLP. A homocistena pode ser
obtida a partir da metionina pelo ciclo do metilo activado.

O tomo de enxofre da cistena provm da homocistena, enquanto o esqueleto


de carbono proveniente da serina.

Ciclo do Metilo Activado


Alm da sua importncia como aminocido constituinte das protenas e iniciados
da biossntese proteica, a metionina, aminocido essencial, , sobretudo, um
fornecedor de grupos metilo. O ciclo do metilo activado permite a activao e
transferncia de grupos metilo para vrios substratos.

Um grupo metilo activado por ligao da adenosina do ATP ao enxofre da


metionina.
O grupo metilo activado transferido para um aceitador.
O grupo metilo do N5-metil-tetrahidrofolato cedido homocistena na
presena da coenzima metilcobalomina (derivado da vitamina B12).

138
Brigitta Cismasiu | FMUL 2010/2011

Derivados dos Aminocidos

Creatina
A creatina sintetizada no fgado a partir da metilao do guanidinoacetato, que,
por sua vez, formado no rim a partir dos aminocidos arginina e glicina.

A creatina usada para armazenar energia. O grupo fosfato do ATP transferido


para a creatina, gerando creatina fosfato, atravs da aco da creatina fosfocinase
(CPK).

Esta reaco reversvel, da que quando necessrio energia (ex.: na contraco


muscular), a creatina fosfato doa o seu fosfato ao ADP para formar ATP. Tanto a
creatina como a creatina fosfato so encontradas no msculo, no crebro e no
sangue.

Glutatio
O glutatio, o tripptido -glutamilcisteinilglicina (GSH), tem vrias funes
importantes no organismo:
um redutor;
conjugado com frmacos para torn-los mais solveis em gua;
Est envolvido no transporte de aminocidos atravs da membrana
celular (o ciclo do -glutamilo);
parte da estrutura de alguns leucotrienos;
um cofactor para algumas reaces enzimticas;
um auxiliar no rearranjo de pontes dissulfito das protenas.

O glutatio sintetizado a partir de trs aminocidos: glutamato, cistena e


glicina. Esse processo amplamente regulado pela disponibilidade de cistena.

139
Brigitta Cismasiu | FMUL 2010/2011

Como redutor, ou seja, na forma de glutationa, muito importante na


manuteno da estabilidade das membranas eritrocitrias. O seu grupo sulfidrilo
pode ser usado para reduzir os perxidos formados durante o transporte de
oxignio (glutatio peroxidase).

A forma oxidada do glutatio (GSSG) consiste em 2 molculas unidas por uma


ponte dissulfito. Esta forma reduzida pela glutatio redutase a 2 molculas de
glutationa custa de NADPH.

A via dos fosfatos de pentoses muito importante nos eritrcitos, para a


produo de NADPH, necessrio glutatio redutase.

Correlaes Clnicas

Fenilcetonria
Na fenilcetonria, desenvolvem-se nveis elevados de fenilanina no sangue devido
sua deficiente converso em tirosina, motivada ou por mutaes na enzima
fenilanina hidroxilase ou em enzimas do sistema de regenerao da
tetrahidrobiopterina.

A excreo na urina com cheiro caracterstico resulta da utilizao de uma via


secundria do metabolismo da fenilalanina.

A acumulao de fenilalanina ou dos seus metabolitos pode comprometer o


normal desenvolvimento do crebro, podendo causar danos cerebrais
irreversveis e subsequente retardao mental. O tratamento mais comum baseia-
se num controlo rigoroso da dieta alimentar, por forma a que apenas se
suplemente a fenilalanina e a tirosina suficientes para as necessidades da sntese
proteica.

140
Brigitta Cismasiu | FMUL 2010/2011

Deficincia de Ornitina Transcarbamoilase


Esta doena compromete o ciclo da ureia. Os grupos amina que no so usados na
sntese de aminocidos e outros compostos azotados so convertidos a amnia,
que se deposita nas mitocndrias dos hepatcitos para ser convertido em ureia e,
assim, excretada na urina.

Como o ciclo da ureia de encontra interrompido, a amnia acumula-se nas


mitocndrias, sem poder ser excretada, o que explica a hiperamonmia verificada
nestes pacientes.

A amnia em excesso liga-se ao -cetoglutarato, originando glutamato. Como o -


cetoglutarato amplamente consumido neste processo, h deficincia de
oxaloacetato e o crebro fica em dfice energtico. Por outro lado, o glutamato
um neurotransmissor e o seu excesso tem um efeito neurotxico. A glutamina
acumula-se no crebro, exercendo presso osmtica e aumentando o edema
cerebral.

Hiperhomocisteinmia
A hiperhomocisteinmia caracteriza-se por elevados nveis de homocistena ou
metionina e baixos nveis de cistena no plasma e na urina. Entre as vrias
possveis causas, a mais comum resulta de mutaes na cistationa--sintase.
Indivduos homozigticos apresentam um alto risco para o desenvolvimento de
doenas cardiovasculares.

A diminuio dos nveis de homocistena pode implicar a restrio da ingesto de


metionina e a incluso de suplementos de vitaminas na dieta alimentar:
Converso de homocistena em cistationa na sntese de cistena:
o Vitamina B6: fosfato de piridoxal (PLP).
Metilao de homocistena em metionina:
o Vitamina B12: metilcobalamina;
o cido flico: tetrahidrofolato.

141
Brigitta Cismasiu | FMUL 2010/2011

Inter-relaes Metablicas
Introduo

Mecanismos de regulao diferentes correspondem a


tempos de resposta diferentes;
Nem todas as clulas so muito afectadas por
variaes das disponibilidades/necessidades
energticas;
Diferentes rgos tm reservas e metabolismos
diferenciados;
Algumas clulas, em rgos especficos, so muito
afectadas pela necessidade de regular o organismo
como um todo;
O rendimento energtico associado a diferentes
fontes moleculares de energia varivel.

Fgado
Fornece energia aos tecidos musculares, ao crebro e a outros rgos perifricos,
sendo capaz de mobilizar rapidamente o glicognio nele armazenado e promover
a gluconeognese a fim de mobilizar glucose no sangue.

O fgado desempenha um papel central na regulao do metabolismo lipdico:


Quando os alimentos energticos so abundantes, no fgado que so
esterificados os cidos gordos sintetizados ou fornecidos pela dieta, sob a
forma de lipoprotenas VLDL (principal fonte de cidos gordos do tecido
adiposo);
Na falta de alimento, os cidos gordos so convertidos pelo fgado em
corpos cetnicos.

142
Brigitta Cismasiu | FMUL 2010/2011

Como alimento prprio, o fgado utiliza preferencialmente cetocidos,


provenientes da degradao dos aminocidos, glucose e no utiliza
acetoacetato, porque no o pode reactivar a acetil-CoA. Ou seja, no consome os
combustveis que alimentam outros tecidos.

Logo que o organismo alimentado, o aumento do teor de glucose no sangue


conduz ao aumento da secreo de insulina e inibe a secreo de glucagina. O
msculo e o fgado sintetizam glicognio, e a afluncia de glucose ao tecido
adiposo permite gerar glicerol-3-fosfato para a sntese de TAG.

Decorridas algumas horas, quando o teor de glucose baixa, invertem-se os


efeitos: ocorre a degradao do glicognio, funciona a gluconeognese e ocorre a
hidrlise dos TAG. A glucose obtida serve sobretudo o crebro e os tecidos que
dele dependem essencialmente, e o fgado e o msculo passam a utilizar os cidos
gordos, em vez da glucose, como alimento energtico.

Em perodos prolongados de jejum, as vias metablicas tendem a adaptar-se s


circunstncias atravs de formao de corpos cetnicos no fgado, que servem de
alimento ao crebro e diminuem as necessidades de glucose.

RESUMINDO:
essencial ao fornecimento de energia ao crebro, ao msculo e a outros
rgos perifricos;
Armazena glicognio e mantm nveis estveis de glucose no plasma;
Efectua gluconeognese (sobretudo a partir de lactato e alanina dos
msculos, glicerol do tecido adiposo e aminocidos glucognicos da
dieta);
Sintetiza cidos gordos, que lana na corrente sangunea em VLDL;
Em jejum, converte cidos gordos em corpos cetnicos;
O malonil-CoA e a carnitina aciltransferase I regulam degradao vs. envio
de cidos gordos para tecido adiposo (armazenamento).

Crebro
O crebro no armazena reservas, pelo que precisa de um abastecimento regular
de glucose. Em perodos de jejum prolongado, os corpos cetnicos fornecidos
pelo fgado podem suprir durante algum tempo as necessidades do crebro.

RESUMINDO:
S consome glucose, excepto em jejum muito prolongado (corpos
cetnicos);
Consome 60% da glucose total necessria em estado de repouso;
[ ] ; ponto de abrandamento
da gliclise ponto crtico para o crebro.

Msculo
Utiliza como fontes de energia a glucose, os cidos gordos e os corpos cetnicos,
mas prefere a glucose e sintetiza e armazena o glicognio de que necessita. Este
glicognio rapidamente convertido em glucose-6-fosfato para que fique
aprisionado nas clulas.

143
Brigitta Cismasiu | FMUL 2010/2011

No msculo activo a maior parte do cido pirvico da gliclise reduzida a cido


lctico, sendo este convertido pelo fgado em glucose, que lhe devolvida (Ciclo
de Cori). Alm disso, formam-se quantidades elevadas de alanina, por
transaminao do cido pirvico.

O msculo em repouso utiliza, sobretudo, cidos gordos (e tambm corpos


cetnicos, no caso do msculo cardaco).

RESUMINDO:
S consome glucose, cidos gordos e corpos cetnicos;
Ao contrrio do crebro, armazena muito glicognio ( das reservas
totais);
No armazena glucose 6-fosfato: a glucose no sai das clulas e
guardada para exerccio sbito;
Num msculo muito activo, o ritmo da gliclise supera o ritmo do ciclo
TCA. Forma-se lactato (vai para o fgado, forma-se glucose ciclo de Cori);
Em repouso, os cidos gordos so a principal fonte de energia;
Os corpos cetnicos so a fonte energtica principal do corao.

Tecido Adiposo
Sintetiza, armazena e mobiliza os TAG, que constituem uma enorme reserva
energtica. A principal tarefa do tecido adiposo a activao dos cidos gordos
(sintetizados pelo fgado) pela acetil-CoA e a sua transferncia para o glicerol.

O principal intermedirio da biossntese de TAG o glicerol-3-fosfato, que provm


da reduo da diidroxiacetona-fosfato a partir da glucose atravs da gliclise. O
tecido adiposo incapaz de fosforilar o glicerol endgeno, requer glucose para a
sntese dos TAG.

A hidrlise em cidos gordos e glicerol catalisada por lipases. No tecido adiposo,


os TAG so continuamente hidrolisados e ressintetizados. O glicerol exportado
para o fgado e a maior parte dos cidos gordos, no caso de haver abundncia de
glicerol-3-fosfato, reesterificada, e caso contrrio, libertada para o plasma.

, portanto, o nvel de glucose no interior das clulas adiposas que determina se


os cidos gordos so libertados no sangue.

RESUMINDO:
uma enorme reserva de TAG;
Esterifica cidos gordos em TAG e hidrolisa-os (mas a sntese de cidos
gordos ocorre no fgado);
O glicerol 3-fosfato formado a partir da glucose (falta a enzima para
fosforilar glicerol directamente);
A glucose um metabolito indispensvel (determina um passo regulador
importante: abundncia de glucose = sntese de TAG);
A hidrlise de TAG um processo controlado por lipases reguladas por
hormonas atravs de fosforilao;
O glicerol resultante da hidrlise exportado para fgado.

144
Brigitta Cismasiu | FMUL 2010/2011

Ciclo jejum-alimentado

Estado Bem-Alimentado
Atravs da alimentao, podemos
obter glucose, aminocidos e lpidos.
A glucose passa das clulas epiteliais
do intestino para o fgado, atravs da
veia porta. Os aminocidos so
parcialmente metabolizados no
intestino delgado antes de seres
libertados para a veia porta.
Quilomicras contendo triacilgliceris
so secretadas pelas clulas
epiteliais do intestino no sistema
linftico, da so transportadas para
o ducto torcico e, posteriormente,
para a veia subclvia, de onde so
enviadas para o resto do corpo.

No fgado, a glucose pode ser convertida a glicognio, atravs da


gluconeognese, ou a piruvato e lactato, por gliclise. Pode ainda ser usado na
via dos fosfatos de pentose para a produo de NADPH, usado em processos
sintticos. O piruvato pode ser oxidado a acetil-CoA, o qual, por sua vez, pode ser
convertido a triacilglicerol ou oxidado a CO2 e gua pelo ciclo TCA. Muita da
glucose obtida atravs da alimentao passa pelo fgado para chegar a outros
rgos, incluindo o crebro, que quase completamente dependente da glucose
para produo de ATP, os eritrcitos e a medula renal, que apenas realizam
gliclise, o tecido adiposo, que a converte em glicerol, para a produo de
triacilglicerol, e os msculos, que a convertem em glicognio ou usam-na na
gliclise e no ciclo TCA. O lactato e o piruvato produzido por gliclise noutros
tecidos so usados pelo fgado e oxidados a CO2 ou convertidos a triacilglicerol.
No estado bem-alimentado, o fgado usa a glucose e no a emprega na
gluconeognese. Assim, o ciclo de Cori (converso da glucose em lactato em
tecidos perifricos seguido de converso do lactato em glucose no fgado)
interrompido neste estado.

As clulas intestinais usam alguns aminocidos como fonte de energia, mas


transportam a maioria para a veia porta. O fgado ainda retira alguns aminocidos
absorvidos antes de serem distribudos para os outros rgos. Isto
especialmente importante para os aminocidos intestinais, os quais so
necessrios a todas as clulas para a sntese de protenas. O fgado metaboliza
aminocidos, mas os valores de Km das enzimas envolvidas elevado, pelo que os
aminocidos tm de estar presentes em concentraes elevadas para que o
catabolismo seja significante. Pelo contrrio, os valores baixos de Km das tRNA
charging enzymes asseguram que a sntese proteica ocorra quando esto
presentes aminocidos. Os aminocidos em excesso podem ser completamente
oxidados a CO2, ureia e gua, ou os seus intermedirios usados na lipognese. Os
aminocidos que abandonam o fgado so usados na sntese proteica ou
produo de energia noutros tecidos.

145
Brigitta Cismasiu | FMUL 2010/2011

O TAG proveniente da dieta alcana a corrente sangunea como quilomicras, nas


quais actua a lipoprotena lipase ligada superfcie das clulas endoteliais no
lmen dos capilares de vrios tecidos, mas particularmente no tecido adiposo. A
lipase hidrolisa uma larga poro do TAG presente nas quilomicras. Os cidos
gordos libertados so absorvidos pelos adipcitos, esterificados pelo glicerol 3-
fosfato (proveniente da glucose via gliclise) formando TAG, e armazenados em
gotculas de lpidos nessas clulas. As quilomicras remanescentes aps a digesto
pelas lipoprotenas lipases so removidas do sangue atravs do fgado. Os TAG
presentes nessas quilomicras so hidrolisados por uma lipase lisossomal. Os
cidos gordos libertados so esterificados para formar TAG. Os TAG assim
formados e os TAG provenientes da sua sntese a partir de glucose e aminocidos
so armazenados em very low-density lipoprotein (VLDL) e secretados no sangue.
Tal como as quilomicras, so transformados em gotculas de lpidos.

As clulas do pncreas respondem ao influxo de glucose e aminocidos no


estado alimentado. Quando a glucose entra nas clulas , a sua oxidao aumenta
a concentrao de ATP, fecha os canais de potssio sensveis a ATP, despolariza a
clula e aumenta os nveis de clcio intracelular, levando libertao de insulina.
As clulas libertam insulina durante e depois de comer, o que essencial para o
metabolismo dos nutrientes pelo fgado, msculo e tecido adiposo.

Fases Iniciais de Jejum


O lactato, o piruvato e a alanina
so desviados da oxidao e sntese
de cidos gordos para a formao
de glucose, completando o ciclo de
Cori. O ciclo da alanina, na qual o
carbono e o azoto retornam ao
fgado na forma de alanina, assuem
um papel importante nesta fase.

Jejum
Uma vez que nenhum combustvel alimentar entra no intestino e o pouco
glicognio que resta mantido no fgado aps 10-12h de jejum, o organismo est
dependente da gluconeognese heptica, primeiramente a partir do lactato, do
glicerol e da alanina. O ciclo de Cori e o ciclo da alanina desempenham papis
importantes, mas no fornecem molculas de carbono para a sntese de glucose,
uma vez que estas vias transferem energia da oxidao de cidos gordos no fgado
para os tecidos perifricos que no conseguem oxidar TAG. Os cidos gordos no
podem ser usados para a sntese de glucose, porque no existem uma via de
converso do acetil-CoA em glucose.

146
Brigitta Cismasiu | FMUL 2010/2011

O glicerol, um subproduto da
liplise no tecido adiposo, um
importante substrato para a sntese
de glucose. No entanto, so as
protenas, especialmente a partir
do msculo esqueltico, que
fornecem a maior parte do carbono
necessrio para a sntese de
glucose. As protenas so
hidrolisadas dentro das clulas
musculares e a maioria dos
aminocidos so parcialmente
metabolizados, dos quais a alanina
e a glutamina so libertadas em
grandes quantidades. Os outros so
metabolizados para dar
intermedirios (piruvato e -
cetoglutarato) que podem formar alanina e glutamina. Cadeias ramificadas de
aminocidos so uma importante fonte de azoto para a produo de alanina e
glutamina no msculo. As cadeias ramificadas de -cetocidos, produzidas a partir
das cadeias ramificadas de aminocidos por transaminao, so parcialmente
libertadas no sangue e captadas pelo fgado, que sintetiza glucose a partir da
valina, corpos cetnicos da leucina, e glucose e corpos cetnicos da isoleucina.

Grande parte da glutamina libertada a partir do msculo usada pelo epitlio


intestinal, pelos linfcitos e pelos macrfagos. A glutamina um importante
combustvel para os entercitos e os linfcitos, os quais se dividem rapidamente
e, portanto, requerem-na para a sntese de pirimidinas e purinas. Neste processo,
a glutamina convertida em glutamato, que transaminado com o piruvato para
formar -cetoglutarato e alanina. No ciclo TCA, o -cetoglutarato convertido a
malato, e este em piruvato. Esta via denominada glutaminlise.

A sntese de glucose no fgado durante o jejum est intimamente ligada sntese


de ureia. A maioria dos aminocidos pode libertar o azoto do grupo amina por
transaminao com -cetoglutarato, formando glutamato e um novo -cetocido,
que pode ser utilizado para a sntese de glucose. O glutamato fornece compostos
azotados necessrios sntese de ureia: amnia, a partir de desaminao
oxidativa pela glutamato desidrogenase, e aspartato, da transaminao do
oxaloacetato pela aspartato aminotransferase. Uma importante fonte adicional
de amnia e de substncias precursoras da ornitina, tais como a citrulina, a
mucosa intestinal.

Por causa dos baixos nveis de insulina no sangue durante o jejum, a liplise
muito activa no tecido adiposo, aumentando o nvel de cidos gordos no sangue,
que so usados em preferncia glucose por vrios tecidos. No corao e no
msculo, a oxidao dos cidos gordos inibe a gliclise e a oxidao do piruvato.
No fgado, fornece a maior parte do ATP necessrio gluconeognese. Muito
pouco acetil-CoA gerado pela oxidao de cidos gordos no fgado
completamente oxidado, sendo convertido em corpos cetnicos pelas
mitocndrias hepticas. Os corpos cetnicos (acetoacetato e -hidroxibutirato)
so libertados na corrente sangunea e so uma fonte de energia para muitos
tecidos. Os cidos gordos no so oxidados pelo crebro, pois no podem cruzar a

147
Brigitta Cismasiu | FMUL 2010/2011

barreira hematoenceflica. Quando a sua concentrao no sangue


suficientemente alta, os corpos cetnicos funcionar como combustvel
alternativo. No entanto, so incapazes de suprir completamente a necessidade de
glucose pelo crebro. Os corpos cetnicos tambm podem suprimir a protelise e
a oxidao de aminocidos de cadeia ramificada no msculo e a diminuio de
libertao de alanina. Isto diminui a atrofia muscular e reduz a quantidade de
glucose sintetizada no fgado. Enquanto so mantidos nveis elevados de corpos
cetnicos pela oxidao heptica de cidos gordos, h menor necessidade de
glucose, de aminocidos gliconeognicos e de degradao de tecido muscular.

Fases Iniciais do Estado Realimentado


Os TAG so metabolizados tal como foi
descrito para o estado alimentado. Em
contraste, a glucose pouco extrado
pelo fgado durante este perodo. Na
verdade, o fgado permanece no modo
gliconeognico por algumas horas aps a
alimentao. No entanto, ao invs de
fornecer glucose para circulao
sangunea, a gluconeognese heptica
fornece glucose 6-fosfato para a
glicognese. Isso significa que o
glicognio heptico no inteiramente
reposto aps jejum por sntese directa a
partir da glucose sangunea. Pelo
contrrio, a glucose catabolisada em
tecidos perifricos a lactato, que convertido no fgado em glicognio pela
gluconeognese, ou seja, indirectamente:

A gluconeognese a partir dos aminocidos que entram no intestino tambm tem


um papel fundamental no restabelecimento dos nveis normais de glicognio
heptico pela via indirecta. Aps o declnio da taxa de sntese de glucose, o
glicognio heptico sustentado pela sntese directa a partir do glicognio
sanguneo.

Interaces Metablicas entre rgos


Apenas o epitlio intestinal converte glutamina
a citrulina, uma vez que o nico tecido que
expressa a glutamato redutase dependente de
ATP necessria a esta converso. A citrulina
produzida no intestino passa pelo fgado e
convertida pelo rim a arginina, que pode ser
convertida em creatina ou libertada no sangue.
O fgado utiliza arginina sangue para gerar
ornitina, que expande a capacidade do ciclo da
ureia durante os perodos de maior consumo de
protenas. O fgado converte irreversivelmente
a ornitina em glutamato, por transaminao e oxidao.

148
Brigitta Cismasiu | FMUL 2010/2011

A sntese de ureia no fgado dependente da sntese de citrulina pelo intestino e


sntese de arginina pelo rim. A arginina tambm usada por muitas clulas para a
produo de xido ntrico (NO), um activador da guanilato ciclase, que produz
cGMP, um importante segundo mensageiro.

A arginina gerada a partir de citrulina no rim pode ser metabolizada usada na


sntese de creatina. A primeira enzima dessa via, a glicina transamidinase (GTA),
encontrada predominantemente no crtex renal, no pncreas e no fgado. A
creatina circula at outros tecidos, principalmente nos msculos, onde serve
como um reservatrio de alta energia quando fosforilada a creatina fosfato. Esta
sofre uma converso no-enzimtica a creatinina, que libertada na corrente
sangunea e eliminada atravs da filtrao renal.

O glutatio (GSH) um tripeptdeo


importante na detoxificao de perxidos
gerados endogenamente e compostos
qumicos exgenos. O fgado desempenha
um papel importante na sntese de GSH a
partir de glutamato, cistena e glicina. A
GSH heptica libertada na corrente
sangunea e na blis. O rim elimina uma
quantidade substancial de GSH plasmtica.

Os entercitos so capazes de absorver a GSH


biliar excretada no lmen intestinal.

A carnitina derivada de residuos de lisina


que sofrem N-metilao. O rim e o fgado, em
menor grau, so os nicos tecidos que podem
sintetizar a carnitina, e, assim, fornecer outros
tecidos, especialmente o msculo e o
corao, com a carnitina necessria
oxidao de cidos gordos.

Necessidades Energticas, Reservas e Homeostasia Calrica


As reservas energeticas so consumidas entre as refeies e durante a noite para
manter a glicmia. Embora a capacidade de mobilizar rapidamente o glicognio
seja realmente muito importante, as nossas reservas de glicognio so minimas
em relao s de gordura. As reservas de gordura so apenas consumidas durante
um jejum mais prolongado. As reservas proteicas tambm podem ser usadas
como uma reserva de energia, porque fornecem aminocidos para oxidao, mas
no so to dispensveis como a gordura e o glicognio e so utilizadas pelo
organismo mais relutantemente.

A constante disponibilidade de combustveis no sangue denominada


homeostasia calrica, o que significa que, independentemente de se estar bem
alimentado, em jejum ou em inanio, os nveis sanguneos de combustveis nas
clulas mantm-se dentro de certos limites.

149
Brigitta Cismasiu | FMUL 2010/2011

As concentraes de glucose no sangue so controladas dentro de limites muito


apertados, enquanto que as concentraes de cidos gordos e de corpos
cetnicos podem variar de acordo com uma ou duas ordens de magnitude,
respectivamente.

A glucose cuidadosamente regulada devido necessidade absoluta do crebro


para este substrato. Se a glicmia baixar muito (<1.5 mM), coma e morte seguir-
se-o se a concentrao de glucose no for restaurada. Por outro lado, a
hiperglicmia deve ser evitada porque a glucose perdida na urina, resultando
em desidratao e, por vezes, em coma hiperosmolar hiperglicmico.
Hiperglicmia crnica resulta na glicao de uma srie de protenas, o que
contribui para as complicaes da diabetes.

As mudanas nos nveis de insulina/glucagina so cruciais para a manuteno da


homeostasia calrica. No estado bem alimentado, a razo insulina/glucagina
elevada, favorecendo o armazenamento de glicognio e TAG. No estado de
inanio, a baixa razo insulina/glucagina estimula a liplise, a protelise e a
gliconeognese.

Homeostasia da Glucose

150
Brigitta Cismasiu | FMUL 2010/2011

FASE I
o estado bem alimentado, em que a glucose fornecida pelos glcidos da dieta.

FASE I
Quando essa fonte se esgota, a glicogenlise mantm os nveis de glucose no
sangue. Aps o glicognio se esgotar, a gliconeognese a partir de lactato, glicerol
e alanina assume uma importncia crescente.

FASE III
A gliconeognese heptica a principal fonte de glucose no sangue.

FASE IV
Aos vrios dias de jejum, a dependncia da gliconeognese diminui. Os corpos
cetnicos acumularam-se em concentraes suficientemente elevadas para que
entrem no crebro e suprimam as suas necessidades energticas. A
gliconeognese renal tambm se torna importante nesta fase.

FASE V
Ocorre aps o jejum muito prolongado de indivduos extremamente obesos e
caracterizada por uma dependncia ainda menor da gliconeognese. As
necessidades energticas de quase todos os tecidos so satisfeitas pela oxidao
de cidos gordos ou corpos cetnicos.

Enquanto as concentraes de corpos cetnicos forem altas, a protelise ser


bastante restrita, e vai haver conservao das protenas musculares e das
enzimas. Esta situao continua at que praticamente toda a gordura tenha sido
usada e os nveis de corpos cetnicos diminudo. Num derradeiro caso, o corpo
utiliza a protena muscular.

RESUMINDO:

151
Brigitta Cismasiu | FMUL 2010/2011

Estados Hormonais e Nutricionais

Obesidade
A gordura corporal origina-se
principalmente da dieta. Apenas
pequenas quantidades so sintetizadas
no fgado e transportadas ao tecido
adiposo ou sintetizadas nos adipcitos.
A obesidade resulta de comer demais.
Resulta de estar no estado bem-
alimentado por muito tempo, por causa
da quantidade de alimentos
consumidos. A fase de jejum do ciclo
jejum-alimentado demasiado curta
para usar a gordura armazenada no
corpo durante a fase de alimentao.
A sndrome metablica, fortemente
associado ao desenvolvimento de
aterosclerose, refere-se a um
conjunto de problemas clnicos
incluindo obesidade abdominal,
presso arterial e concentrao de
lpidos elevadas e resistncia
insulina.

A obesidade causa frequentemente


resistncia insulina, por diminuio
da afinidade dos receptores de
insulina ou respostas anormais ao
sinal dado por essa hormona.
Normalmente, os nveis plasmticos
de insulina esto muito elevados em
indivduos obesos, o que um sinal de
desenvolvimento de diabetes tipo 2.

A distribuio anatmica da gordura corporal


tem uma grande influncia sobre os riscos de
sade associados. O excesso de gordura
localizado na rea abdominal central do corpo
denominado obesidade do tipo masculino,
andride ou em forma de ma, e est associada
a um maior risco para hipertenso, resistncia
insulina, diabetes, dislipidmia e doena
coronria. Pelo contrrio, a gordura distribuda
nas extremidades inferiores em torno das ancas
ou da regio gltea denominada obesidade do
tipo feminino, ginide ou em forma de pra, e
relativamente benigna.

152
Brigitta Cismasiu | FMUL 2010/2011

Os tipos regionais de gordura descritos so bioquimicamente diferentes. As


clulas de gordura abdominal so muito maiores e tm uma maior taxa de
turnover de lpidos do que as clulas de gordura inferior. Os adipcitos
abdominais so, tambm, hormonalmente mais responsivos do que os adipcitos
das pernas e das ndegas.

Geralmente, os homens perdem peso mais facilmente que as mulheres, j que


tendem a acumular a gordura abdominal facilmente mobilizvel. Alm disso, as
substncias libertadas a partir da gordura abdominal so absorvidas atravs da
veia porta e, assim, tm acesso directo ao fgado. Os cidos gordos absorvidos
pelo fgado podem levar resistncia insulina e ao aumento da sntese de
triacilgliceris, que so libertados como VLDL. Em contraste, os cidos gordos
livres da gordura gltea entram na circulao sangunea e no tm nenhuma
aco preferencial sobre o metabolismo heptico.

Quando os triacilgliceris so depositados nos adipcitos, as clulas sofrem algum


aumento de tamanho. No entanto, a capacidade de expanso dos adipcitos
limitada, e quando o seu tamanho mximo atingido, este divide-se. Assim, a
obesidade envolve um aumento tanto no nmero como no tamanho dos
adipcitos e, uma vez adquiridas, estas clulas nunca so perdidas. Assim, quando
um indivduo obeso perde peso, o tamanho dos adipcitos reduzido, mas o seu
nmero no afectado.

Dieta
Perder peso requer um balano negativo, o que significa que preciso consumir
menos calorias do que as que so gastas durante o dia. O consumo de menor
quantidade de comida com a mesma composio de macronutrientes tem pouco
efeito no ciclo jejum-alimentado. O papel dos diferentes tecidos vai ser o mesmo
que no estado bem-alimentado, com a excepo de que menos glicognio e TAG
so armazenados e a mudana para o estado de jejum ocorre mais depressa aps
as refeies.

Outro tipo de dieta baseia-se na reduo de ingesto de gordura,


especificamente, reduzindo, assim, a quantidade de calorias ingeridas. De novo,
vai haver menor armazenamento de TAG e uma mudana mais rpida para o
estado de jejum, a no ser que a diminuio da ingesto de gorduras seja
compensada com um aumento do consumo de glcidos.
Na dieta Atkins, tambm denominada dieta cetognica, h consumo elevado de
protenas, moderado de gorduras e muito baixo de glcidos. O estado de jejum
pouco afectado, mas a quase ausncia de glcidos na dieta requer que o fgado se
mantenha gluconeognico e cetognico no estado alimentado. Em resposta s
refeies, ocorre uma pequena elevao da glucose no sangue e um aumento
menor que o normal da insulina. Os aminocidos em excesso so convertidos a
glicognio, glucose e corpos cetnicos. Grandes quantidades de aminocidos
provenientes do intestino reduzem a sua libertao a partir de tecidos perifricos.
Os cidos gordos que chegam ao fgado sob a forma de quilomicra remanescentes
so convertidos a corpos cetnicos, de modo a fornecerem energia para a
gluconeognese. A necessidade de energia na forma de corpos cetnicos nos
tecidos perifricos balana a sua produo no fgado, no ocorrendo cetoacidose.

153
Brigitta Cismasiu | FMUL 2010/2011

Diabetes Mellitus Tipo 2 ou Insulino-Resistente


Indivduos com diabetes tipo 2 so resistentes insulina e tm uma produo
insuficiente desta hormona para ultrapassar essa resistncia. A maioria dos
pacientes obesa e, apensar de os seus nveis de insulina poderem ser elevados,
no so to elevados como os de uma pessoa no-diabtica, mas semelhantes a
indivduos obesos. Por isso, esta forma de diabetes tambm uma forma de
falncia das clulas . Apesar de o corpo produzir insulina, no suficiente para
controlar a produo de glucose pelo fgado ou para promover a sua absoro
pelos tecidos perifricos, especialmente msculo esqueltico. O aumento normal
de frutose 2,6-bisfosfato e a regulao da fosfoenolpiruvato carboxicinase no
ocorre. A translocao de vesculas intracelulares transportadoras de GLUT4 para
a membrana plasmtica em resposta insulina diminuda nos micitos e
adipcitos.
A hipertriacilglicerolmia
caracterstica e normalmente resulta
de um aumento de VLDL sem
hiperquilomiconmia. Isto ocorre
devido s rpidas taxas de sntese de
novo heptica de cidos gordos e
desvio dos cidos gordos que
chegam ao fgado para TAG e VLDL.
Lipognese e gluconeognese
concomitantes no deveriam
acontecer, mas o defeito da via de
sinalizao da insulina que controla a
gluconeognese impede a supresso da produo heptica de glucose face dos
altos nveis de insulina. Uma via de sinalizao mais receptiva que controla a
sntese e esterificao de cidos gordos leva sobreproduo de TAG.

O tratamento passa, numa primeira instncia por dieta, exerccio e perda de peso.
Se a glicmia no puder ser controlada deste modo, recorre-se prescrio de
medicamentos ou insulina exgena.

Diabetes Mellitus Tipo 1 ou Insulino-Dependente


Em contraste com a diabetes tipo 2, h uma ausncia completa de produo de
insulina pelo pncreas em indivduos com diabetes insulino-dependente. Uma vez
que a razo insulina/glucagina no
consegue aumentar, o fgado est
permanentemente no estado
gluconeognico e cetognico e no
consegue controlar eficazmente a
glicmia. De facto, dado que a
gluconeognese heptica
contnua, o fgado contribui para a
hiperglicmia no estado bem-
alimentado. No tecido muscular e
adiposo, o GLUT4 permanece
sequestrado dentro das clulas.

154
Brigitta Cismasiu | FMUL 2010/2011

Uma gluconeognese acelerada, alimentada pela protelise descontrolada no


msculo esqueltico, mantm a hiperglicmia, mesmo em jejum. A liplise
descontrolada nos adipcitos aumenta os nveis sanguneos de cidos gordos e a
cetognese no fgado. A cetoacidose diabtica desenvolve-se devido
acumulao de corpos cetnicos e ies de hidrognio. A hipertriacilglicerolmia
consequncia de as VLDL e as quilomicras no serem degradadas pela
lipoprotena lipase, cuja expresso est dependente da insulina.

Assim, nestes pacientes, todos os tecidos desempenham um papel catablico


prprio do estado de jejum, apensar da ingesto adequada ou at excessiva de
combustvel. Com efeito, o organismo fica encurralado nume estado permanente
de jejum, o que leva a que haja um grande desperdcio dos tecidos do corpo e,
finalmente, a morte, a menos que seja administrada insulina. A insulina exgena
o nico medicamento eficaz na diabetes tipo 1.

Exerccio Aerbico e Anaerbico


O exerccio aerbico exemplificado pela corrida de longa distncia; o exerccio
anaerbio pela corrida ou levantamento de peso.

EXERCCIO ANAERBIO
Durante o exerccio anaerbio, h pouco
cooperao entre os tecidos do organismo.
Os vasos sanguneos dos msculos so
comprimidos durante o pico da contraco;
assim, as suas clulas so isoladas do resto
do corpo e dependem, em grande medida,
do seu prprio armazenamento de
glicognio e fosfocreatina. A fosfocreatina
uma fonte de fosfatos de alta energia para
a sntese de ATP, at que a glicogenlise e a
gliclise sejam estimuladas. Na falta de oxignio, a gliclise torna-se a principal
fonte de ATP. O lactato acumula-se no sangue porque a taxa de produo de
lactato no msculo excede a taxa de utilizao de lactato para a sntese de glucose
no fgado. Em situaes normais, o crebro no utiliza lactato como combustvel
porque a sua concentrao muito inferior ao KM do sistema transportador de
lactato atravs da barreira hematoenceflica. No entanto, quando a concentrao
de lactato aumenta at um centro nvel, durante o exerccio exaustivo, este pode
penetrar a barreira hematoenceflica e tornar-se um importante combustvel do
crebro. Este processo ajuda a eliminar tanto o lactato do sangue como a baixar a
sua acidez. Alm disso, a sua utilizao muito eficiente, uma vez que o crebro
evita, deste modo, o consumo de energia pelo fgado na converso de lactato a
glucose.

EXERCCIO AERBIO MODERADO


Durante o exerccio aerbio moderado, grande parte da energia derivada da
gliclise do glicognio muscular. O contedo muscular de glicognio pode ser
aumentado pelo exerccio extenuante, que esgota o glicognio, seguido de
repouso e uma dieta rica em glcidos. H tambm estimulao da oxidao de
aminocidos de cadeia ramificada, produo de amnia e libertao de alanina a
partir do msculo em exerccio.

155
Brigitta Cismasiu | FMUL 2010/2011

EXERCCIO AERBIO PROLOGADO


Um indivduo bem-alimentado no armazena glucose e glicognio suficientes para
fornecer a energia necessria ao exerccio aerbio prolongado. O quociente
respiratrio, a razo entre dixido de carbono exalado e oxignio consumido,
diminui durante a corrida de longa distncia. Isso indica a alterao progressiva da
degradao de glicognio para a oxidao de cidos gordos. A liplise aumenta
gradualmente medida que os depsitos de glucose se esgotam e, tal como em
jejum, os msculos oxidam cidos gordos preferencialmente glucose. No
entanto, contrariamente ao que acontece em jejum, h pouco aumento da
concentrao sangunea de corpos cetnicos, j que a cetognese heptica
equilibrada pela oxidao muscular de corpos cetnicos.

Consumo de lcool
O etanol absorvido no intestino. Difunde-se muito rapidamente por todas as
clulas e metabolizado por duas vias. A primeira tem lugar no fgado.

A primeira reaco, catalisada por lcool desidrogenases, ocorre no citosol:

A segunda, catalisada pela aldedo desidrogenase, ocorre na matriz mitocondrial:

A velocidade da lcool desidrogenase relativamente da aldedo desidrogenase e


a capacidade de regular mais eficazmente os nveis adaptativos da primeira,
determina a acumulao de acetaldedo, e suas consequncias.

156
Brigitta Cismasiu | FMUL 2010/2011

O consumo de etanol leva a uma acumulao de NADH. Esta concentrao


elevada de NADH inibe a gluconeognese, evitando a oxidao do lactato a
piruvato. As consequncias podem ser acidose lctica e hipoglicmia. O excesso
de NADH tambm inibe a oxidao dos cidos gordos, j que o seu objectivo
metablico criar NADH para a produo de ATP pela fosforilao oxidativa, mas
um consumidor de lcool tem essas necessidades supridas pelo metabolismo do
etanol. Assim, o NADH em excesso cria as condies ideais para a sntese de
cidos gordos, levando acumulao de TAG fgado, condio conhecida como
"fgado gordo".

A segunda via, dependente do citocromo P450, gera acetaldedo e,


subsequentemente, acetato, enquanto oxida poder redutor biossinttico, NADPH,
a NADP+. Uma vez que usa oxignio, esta via gera radicais livres que danificam os
tecidos. Alm disso, porque o sistema consome NADPH, o glutatio antioxidante
no pode ser regenerado, agravando o stress oxidativo.

As mitocndrias hepticas tm uma


capacidade limitada para oxidar o
acetato a CO2, porque o ciclo TCA
inibido por elevadas concentraes de
NADH e ATP durante a oxidao do
etanol. Muito do acetato formado a
partir de etanol escapa do fgado para
o sangue. Praticamente todos os
outros tecidos o podem oxidar a CO2
pelo ciclo TCA.

O acetaldedo, intermedirio na formao de acetato a partir do etanol, tambm


pode escapar do fgado e rapidamente forma ligaes covalentes com grupos
funcionais de compostos biologicamente importantes.

157
Brigitta Cismasiu | FMUL 2010/2011

158
Brigitta Cismasiu | FMUL 2010/2011

Regulao do Metabolismo
Estratgias de Regulao

De modo a ajustar a sntese e degradao de metabolitos s necessidades


fisiolgicas num dado perodo, o fluxo de metabolitos ao longo de uma via
metablica determinado pelas enzimas envolvidas.

Assim, a estratgia principal de regulao alterar a actividade da enzima que


catalisa a reaco mais lenta (enzima-chave). Essa regulao conseguida de trs
formas:
1. Controlo da transcrio (induo/represso)
Altera a sntese da enzima por alterao do mRNA. um mecanismo
mediado por protenas reguladoras que actuam no DNA. Os genes contm
locais de ligao a estas enzimas nos promotores. um mecanismo lento.
2. Interconverso
A enzima est ligada ao centro activo na forma activa. convertida
forma activa por uma enzima activadora (sinalizao e mediao por 2os
mensageiros). A fosforilao dependente do ATP catalisada por uma
cinase e a desfosforilao por uma fosfatase.
3. Modulao atravs de ligantes
o mecanismo mais rpido. dividido em trs nveis de regulao:
Disponibilidade de percursores: vias que produzem/degradam
percursores ou compartimentalizao (condio selectiva para as
enzimas de cada compartimento);
Disponibilidade de coenzimas: outras vias (independentes) que
regeneram a coenzima;
Ligantes (alostricos): ligam-se a locais diferentes do centro activo e
podem ser produtos intermdios ou finais da reaco, metabolitos
de outras vias, percursores, etc.

Correlao Anatmica

O sistema porta-heptico responsvel pelo direccionamento do sangue a partir


do trato gastrointestinal para o fgado. As substncias absorvidas no intestino
delgado viajam primeiramente para o fgado, para serem processadas, antes de
continuar para o corao. O fgado muito irrigado: a veia porta divide-se em
microcanais e praticamente todas as clulas hepticas so irrigadas.

159
Brigitta Cismasiu | FMUL 2010/2011

Regulao Hormonal

Insulina e Glucagina na Regulao dos Metabolismos


As hormonas peptdicas, especialmente a glucagina e a insulina, desempenham
um papel essencial na regulao integrada dos metabolismos glicdico e lipdico.

A insulina segregada nas clulas do pncreas na presena de um teor elevado


de glucose. Actua atravs de uma cascata de protenas cinases no dependentes
do cAMP, que conduz ao aumento do teor de glicognio sintase a e diminuio
do teor de fosforilase a.
1. Favorece a formao de glicognio, tanto no fgado como no msculo;
2. Inibe a gluconeognese no fgado;
3. Acelera a gliclise no fgado, favorecendo a sntese de cidos gordos;
4. Promove a entrada de glucose no msculo e nas clulas adiposas,
resultando na sntese e armazenamento de TAG;
5. Estimula a sntese proteica no msculo e inibe a degradao intracelular das
protenas.

A glucagina segregada tambm no pncreas, nas clulas , em condies de


baixo teor de glucose. A sua actividade baseada na activao de cascatas de
protenas cinases pelo cAMP e uma lipase.
1. Favorece a degradao do glicognio e inibe a sua sntese: tal como a
epinefrina, faz aumentar o teor em fosforilase a e baixar a concentrao de
glicognio sintase a;
2. Favorece a gluconeognese no fgado: a activao de uma cascata a partir
do cAMP faz baixar a concentrao de frutose 2,6-difosfato, o que conduz
activao da gluconeognese e inibio da gliclise;
3. Inibe a sntese dos cidos gordos: (1) a produo do cido pirvico diminui,
(2) a glucagina inibe a actividade da acetil-CoA carboxilase e, alm disso, (3)
a hidrlise dos TAG no tecido adiposo reduzida.

A epinefrina e a norepinefrina so segregadas pela adrenal-medula em resposta a


baixos teores de glucose no sangue. A sua aco anloga da glucagina, mas o
seu efeito sobre a degradao do glicognio mais intenso no msculo do que no
fgado. Contrariam tambm o consumo de glucose pelo msculo, favorecendo o
de cidos gordos libertados pelo tecido adiposo. Alm disso, a adrenalina tambm
estimula a secreo de glucagina e inibe a secreo de insulina.

160
Brigitta Cismasiu | FMUL 2010/2011

Vitaminas

As vitaminas so compostos orgnicos que no podem ser sintetizados em


quantidades adequadas pelos seres humanos e, portanto, devem ser fornecidos
pela dieta.

As nove vitaminas hidrossolveis so: cido flico (B9), cobalamina (B12), o cido
ascrbico (C), piridoxina (B6), tiamina (B1), niacina (B3), riboflavina (B2), biotina (B7)
e cido pantotnico (B5). As quatro vitaminas lipossolveis so: A, D, K e E.

As vitaminas so necessrias para realizar funes celulares especficas. Muitas


das vitaminas hidrossolveis so precursoras de coenzimas das enzimas do
metabolismo intermedirio. Ao contrrio das vitaminas hidrossolveis, apenas
uma vitamina lipossolvel, a vitamina K, tem funo de coenzima. Estas vitaminas
so libertadas, absorvidas e transportadas com os lpidos da dieta. Eles no so
imediatamente excretadas na urina, pelo que quantidades significativas so
armazenadas no fgado e no tecido adiposo. De facto, o consumo de algumas
vitaminas em excesso pode levar acumulao de quantidades txicas destes
compostos.

cido Flico
O cido flico ou folato, que desempenha um papel fundamental no metabolismo
de carbono, essencial para a biossntese de vrios compostos. O tetrahidrofolato
recebe tomos de carbono de doadores como a serina, a glicina e a histidina, e
transfere-os para intermedirios na sntese de aminocidos, purinas e timidina
monofosfato (TMP).

Cobalamina
A cobalamina ou vitamina B12 necessria nos seres humanos para duas reaces
enzimticas essenciais: a metilao da homocistena a metionina e a isomerizao
de metilmalonil-CoA que produzido durante a degradao de alguns
aminocidos e cidos gordos com nmero mpar de tomos de carbono. Quando
a vitamina est em falta, existe acumulao de cidos gordos anormais, que so
incorporados nas membranas celulares.

161
Brigitta Cismasiu | FMUL 2010/2011

cido Ascrbico
A forma activa da vitamina C o cido ascrbico, que actua como agente redutor
em vrias reaces. A vitamina C tem um papel importante como coenzima nas
reaces de hidroxilao, como por exemplo, a hidroxilao dos resduos prolil e
lisil de colagnio. A vitamina C , portanto, necessrio para a manuteno do
tecido conjuntivo normal, bem como para a cicatrizao de feridas. A vitamina C
tambm facilita a absoro de ferro no intestino.

Piridoxina
A vitamina B6 um termo colectivo para os derivados da piridina (piridoxina,
piridoxal e piridoxamina), que diferem apenas na natureza do grupo funcional
ligado ao anel. A piridoxina existe principalmente em plantas, ao passo que o
piridoxal e a piridoxamina so encontrados em alimentos obtidos de animais.
Todos os trs compostos podem servir como precursores do fosfato de piridoxal
(PLP), que actua como coenzima de um grande nmero de enzimas,
particularmente aquelas que catalisam reaces envolvendo aminocidos
(transaminao, desaminao, descarboxilao e condensao).

Tiamina
O pirofosfato tiamina (TPP) a forma biologicamente activa da vitamina B1,
formada pela transferncia de um grupo pirofosfato da adenosina trifosfato (ATP)
para a tiamina. O pirofosfato de tiamina serve como coenzima na formao ou
degradao de -cetis pela transcetolase e na descarboxilao oxidativa do -
cetocidos.

Niacina
As formas biologicamente activas da niacina ou cido nicotnico so nicotinamida
adenina dinucletido (NAD+) e o seu derivado fosforilado, a nicotinamida adenina
dinucletido fosfato (NADP+). A nicotinamida, um derivado do cido nicotnico,
contm uma amida em vez de um grupo carboxilo e obtida atravs da dieta.
Esta facilmente desaminada no organismo e, portanto, nutricionalmente
equivalente ao cido nicotnico. O NAD+ e o NADP+ servem como coenzimas em
reaces de oxidao-reduo nas quais a coenzima sofre reduo do anel
piridnico ao aceitar um io hidreto (H-). As formas reduzidas das coenzimas so
NADH e NADPH, respectivamente.

Riboflavina
As duas formas biologicamente activas da riboflavina so a flavina
mononucletido (FMN) e a flavina adenina dinucletido (FAD), formadas pela
transferncia de uma molcula de adenosina monofosfato do ATP para a FMN.
Tanto a FMN como a FAD so capazes de aceitar reversivelmente dois tomos de
hidrognio, formando FMNH2 ou FADH2. A FMN e a FAD formam ligaes fortes
(por vezes, covalentes) com flavoenzimas que catalisam a oxidao ou reduo de
um substrato.

162
Brigitta Cismasiu | FMUL 2010/2011

Biotina
A biotina actua como coenzima nas reaces de carboxilao, nas quais serve de
transportador de dixido de carbono activado. A biotina covalentemente ligada
aos grupos -amina dos resduos de lisina de enzimas dependentes de biotina.

cido Pantotnico
O cido pantotnico um componente da coenzima A, que participa na
transferncia de grupos acilo. A coenzima A contm um grupo tiol que, ao reagir
com os cidos carboxlicos, forma tiosteres, de modo a actuar como
transportador de grupos acilo (ex.: succinil-CoA e acetil-CoA). O cido pantotnico
tambm um componente da sintase de cido.

Vitamina A
Os retinides, uma famlia de molculas que inclui as formas naturais e sintticas
da vitamina A (retinol, retinal, cido retinico e -caroteno), so essenciais para a
viso, reproduo, crescimento, manuteno de tecidos epiteliais. O cido
retinico, derivado da oxidao do retinol da dieta, medeia a maioria das aces
dos retinides, com excepo de viso, que depende do retinal, o aldedo
derivado do retinol.

Vitamina D
As vitaminas D so um grupo de esteris que possuem uma funo semelhante s
hormonas. A molcula activa, calcitriol ou 1,25-dihidroxicolecalciferol (1,25-diOH-
D3), liga-se a receptores intracelulares. O complexo 1,25-diOH-D3-receptor
interage com o DNA no ncleo das clulas-alvo e estimula selectivamente a
expresso do gene ou reprime especificamente a transcrio de genes. A aco
mais proeminente do 1,25-diOH-D3 a regulao dos nveis plasmticos de clcio
e fsforo.

Vitamina K
O papel principal da vitamina K a modificao ps-traducional dos vrios
factores de coagulao, na qual serve de coenzima na carboxilao de certos
resduos de cido glutmico presentes nestas protenas. A vitamina K existe sob
vrias formas, como, por exemplo, filoquinona ou vitamina K1, nas plantas, e
menaquinona ou vitamina K2, na flora bacteriana intestinal.

Vitamina E
As vitaminas E so um grupo de oito tocoferis naturais, dos quais o -tocoferol
o mais activo. A principal funo da vitamina E como um antioxidante na
preveno da oxidao dos componentes celulares no enzimticos, como os
cidos gordos poliinsaturados, pelo oxignio molecular e pelos radicais livres.

163
Brigitta Cismasiu | FMUL 2010/2011

Correlaes Clnicas

Anemias Nutricionais
As anemias nutricionais, causadas pela ingesto inadequada de um ou mais
nutrientes essenciais, podem ser classificadas de acordo com o tamanho dos
eritrcitos ou o volume corpuscular mdio. A anemia microctica, causada pela
falta de ferro, a anemia nutricional mais comum. A outra anemia nutricional
importante a anemia macroctica, que resulta de uma deficincia de cido flico
ou de vitamina B12. Esta anemia tambm denominada megaloblstica, uma vez
que a deficincia de cido flico ou de vitamina B12 provoca a acumulao de
precursores eritrocitrios imaturos de grandes dimenses, os megaloblastos, na
medula ssea e no sangue.

Anemia Perniciosa
A deficincia de vitamina B12 raramente resultado da sua ausncia na dieta.
muito mais comum encontrar esta situao em pacientes que no conseguem
absorver a vitamina a partir do intestino, resultando em anemia perniciosa. A
doena mais frequentemente resultado de uma destruio auto-imune das
clulas parietais gstricas, que so responsveis pela sntese de uma glicoprotena
denominada factor intrnseco. Normalmente, a vitamina B12 obtida na dieta liga-
se ao factor intrnseco no intestino. O complexo cobalamina-factor intrnseco
percorre o intestino e, eventualmente, estabelece ligaes com receptores
especficos na superfcie das clulas da mucosa do leo. A cobalamina ligada
transportada para dentro da clula da mucosa e, posteriormente, para a
circulao sangunea. A falta de factor intrnseco impede a absoro da vitamina
B12, resultando nesta anemia.

164
Brigitta Cismasiu | FMUL 2010/2011

Casos de Estudo
[in Toy, E., Seifert, Jr., Strobel, H., Harms, K. (2008) Case Files Biochemistry. 2
edio, The McGraw-Hill Companies]

Caso 1

A 15-year-old African-American female presents to the emergency room with


complaints of bilateral thigh and hip pain. The pain has been present for 1 day and
is steadily increasing in severity. Acetaminophen and ibuprofen have not relieved
her symptoms. She denies any recent trauma or excessive exercise. She does
report feeling fatigued and has been having burning with urination along with
urinating frequently. She reports having similar pain episodes in the past,
sometimes requiring hospitalization. On examination, she is afebrile (without
fever) and in no acute distress. No one in her family has similar episodes. Her
conjunctiva and mucosal membranes are slightly pale in coloration. She has
nonspecific bilateral anterior thigh pain with no abnormalities appreciated. The
remainder of her examination is completely normal. Her white blood cell count is
elevated at 17.000/mm3, and her hemoglobin (Hb) level is decreased at 7,1 g/dL.
The urinalysis demonstrated an abnormal number of numerous bacteria.

What is the most likely diagnosis?


What is the molecular genetics behind this disorder?
What is the pathophysiologic mechanism of her symptoms?

Caso 2

A 68-year-old female in a hypertensive crisis is being treated in the intensive care


unit (ICU) with intravenous nitroprusside for 48 hours. The patients blood
pressure was brought back down to normal levels; however, she was complaining
of a burning sensation in her throat and mouth followed by nausea and vomiting,
diaphoresis, agitation, and dyspnea. The nurse noticed a sweet almond smell in
her breath. An arterial blood gas revealed a significant metabolic acidosis. A
serum test suggests a metabolite of nitroprusside, thiocyanate, is at toxic levels.

What is the likely cause of her symptoms?


What is the biochemical mechanism of this problem?
What is the treatment for this condition?

Caso 3

A 29-year-old male presents to the emergency department with complaints of


dark-colored urine, generalized fatigue, myalgias, and weakness after completing
a marathon. The patient states that this was his first marathon. He has no
significant medical history and denies any medications or drug use. On

165
Brigitta Cismasiu | FMUL 2010/2011

examination, he appears moderately ill and is afebrile with normal vital signs.
Physical exam reveals diffuse musculoskeletal tenderness. Urinalysis revealed
large amounts of blood (hemoglobin and myoglobin), and serum creatine
phosphokinase (CPK) was significantly elevated, as well as the potassium level on
his electrolytes. The serum lactate level was markedly elevated.

What is the most likely diagnosis?


What is the most appropriate treatment?
What is the biochemical basis for the markedly elevated serum lactate
level?

Caso 4

A 50-year-old Hispanic female presents to your clinic with complaints of excessive


thirst, fluid intake, and urination. She denies any urinary tract infection symptoms.
She reports no medical problems, but has not seen a doctor in many years. On
examination she is an obese female in no acute distress. Her physical exam is
otherwise normal. The urinalysis revealed large glucose, and a serum random
blood sugar level was 320 mg/dL.

What is the most likely diagnosis?


What other organ systems can be involved with the disease?
What is the biochemical basis of this disease?

Caso 5

A 2-year-old black girl is being seen by the hematologist after her pediatrician
found her to be severely anemic with splenomegaly and jaundice. Her mother
gives a possible history of a blood problem in her family but doesnt know for
sure. Her hemoglobin electrophoresis was normal, and the complete blood count
(CBC) revealed a normocytic anemia. The platelet and white blood cell counts are
normal. On the peripheral smear, there are many bizarre erythrocytes, including
spiculated cells. A diagnosis of pyruvate kinase deficiency is made.

What is the biochemical mechanism for this disorder?


How is this disorder inherited?

Caso 6

A 3-year-old boy is brought to the emergency department after several episodes


of vomiting and lethargy. His pediatrician has been concerned about his failure to
thrive and possible hepatic failure along with recurrent episodes of the vomiting
and lethargy. After a careful history is taken, you observe that these episodes
occur after ingestion of certain types of food, especially high in fructose. His blood
sugar was checked in the emergency department and was extremely low.

166
Brigitta Cismasiu | FMUL 2010/2011

What is the most likely diagnosis?


What is the biochemical basis for the clinical symptoms?
What is the treatment of the disorder?

Caso 7

A 48-year-old male presents to the clinic because of concerns about heart disease.
He reports that his father died from a heart attack at age 46, and his older brother
has also had a heart attack at age 46 but survived and is on medications for
elevated cholesterol. The patient reports chest pain occasionally with ambulation
around his house and is not able to climb stairs without significant chest pain and
shortness of breath. The physical exam is normal, and the physician orders an
electrocardiogram (ECG), exercise stress test, and blood work. The patients
cholesterol result comes back as 350 mg/dL (normal 200). The physician
prescribes medication, which he states is directed at the rate-limiting step of
cholesterol biosynthesis.

What is the rate-limiting step of cholesterol metabolism?


What is the class of medication prescribed?

Caso 8

A 1-year-old girl is brought to her pediatricians office with concerns about her
development. She had an uncomplicated birth outside the United States at term.
The mother reports that the baby is not achieving the normal milestones for a
baby of her age. She also reports an unusual odor to her urine and some areas of
hypopigmentation on her skin and hair. On exam, the girl is noted to have some
muscle hypotonia and microcephaly. The urine collected is found to have a
mousy odor.

What is the most likely diagnosis?


What is the biochemical basis of the hypopigmented skin and hair?

Caso 9

A 38-year-old vegetarian (vegan) Caucasian female presents to her primary care


doctor with fatigue and tingling/numbness in her extremities (bilateral). The
symptoms have been gradually getting worse over the last year. Upon further
questioning she reports frequent episodes of diarrhea and weight loss. On exam,
she is pale and tachycardic. Her tongue is beefy red and a neurologic exam reveals
numbness in all extremities with decreased vibration senses. A complete blood
count (CBC) demonstrates megaloblastic anemia.

167
Brigitta Cismasiu | FMUL 2010/2011

What is the most likely diagnosis?


What is the most likely underlying problem for this patient?
What are the two most common causes of megaloblastic anemia and
how would this patients history and examination differentiate the two?

Caso 10

A 46-year-old male presents to the emergency department with severe right toe
pain. The patient was in usual state of health until early in the morning when he
woke up with severe pain in his right big toe. The patient denies any trauma to
the toe and no previous history of such pain in other joints. He did say that he had
a few too many beers with the guys last night. On examination, he was found to
have a temperature of 38,2C and in moderate distress secondary to the pain in
his right toe. The right big toe was swollen, warm, red, and exquisitely tender. The
remainder of the examination was normal. Synovial fluid was obtained and
revealed rod- or needle-shaped crystals that were negatively birefringent under
polarizing microscopy.

What is the likely diagnosis?


How would you make a definite diagnosis?
What is the pathophysiology of this disorder?

Solues

Caso 1
Summary: A 15-year-old African-American female with recurrent bilateral thigh
and hip pain, anemia, and symptoms and laboratory evidence of a urinary tract
infection.

Most likely diagnosis: Sickle cell disease (pain crisis).


Biochemical mechanism of disease: Single amino acid substitution on
hemoglobin beta chain, inherited in an autosomal recessive fashion (1 of
12 African Americans in United States are carriers of the trait).
Pathophysiologic mechanism of symptoms: The sickled red blood cells
cause infarction of bone, lung, kidney, and other tissue from
vasoocclusion.

168
Brigitta Cismasiu | FMUL 2010/2011

Caso 2
Summary: A 69-year-old female with new onset burning sensation in mouth and
throat, nausea and vomiting, agitation, and diaphoresis after a medication error
was noted. Metabolic acidosis is seen on the arterial blood gas. A thiocyanate
level is in the toxic range.

Diagnosis: Cyanide poisoning from toxic dose of nitroprusside.


Biochemical mechanism: Cyanide inhibits mitochondrial cytochrome
oxidase, blocking electron transport and preventing oxygen utilization.
Lactic acidosis results secondary to anaerobic metabolism.
Treatment: Supportive therapy, gastrointestinal (GI) decontamination,
oxygen, and antidotal therapy with amyl nitrite, sodium nitrite, and
sodium thiosulfate.

Caso 3
Summary: 29-year-old marathon runner with acute episode of generalized
myalgias, weakness, fatigue, and dark-colored urine with urine
myoglobin/hemoglobin, hyperkalemia, and significantly increased CPK isoenzyme.

Most likely diagnosis: Rhabdomyolysis (skeletal muscle cell lysis) after


strenuous exercise.
Treatment: Aggressive intravenous hydration to help clear the excess
myoglobin from the serum, and correction of electrolyte abnormalities
and treatment of kidney failure if present.
Biochemical basis for elevated lactate: Nicotinamide adenine
dinucleotide (NADH) levels increase because of the relative lack of oxygen
for muscle, adenosine diphosphate (ADP) and adenosine monophosphate
(AMP) concentrations rise in the cytoplasm, leading to an increased flux of
glucose through the glycolytic pathway in the muscle, causing pyruvate
levels to increase. Pyruvate is reduced by NADH to lactate in a reaction
catalyzed by lactate dehydrogenase. Lactate is transported out of the
muscle cell to the blood.

Caso 4
Summary: 50-year-old obese Hispanic female presents with polydipsia,
polyphagia, and urinary frequency and elevated random blood sugar of 320
mg/dL.

Diagnosis: Type II diabetes.


Other organ systems involved: Cardiovascular, eye, peripheral nerves,
gastrointestinal, kidney.
Biochemical basis: Insulin resistance as a result of a postinsulin receptor
defect. The insulin levels are normal or increased as compared with
normal individuals; however, the insulin is not recognized, and thus the
glucose levels remain elevated.

169
Brigitta Cismasiu | FMUL 2010/2011

Caso 5
Summary: A 2-year-old black girl has normocytic anemia, jaundice, splenomegaly,
and peripheral smear showing spiculated cells. A family history of similar
symptoms is possible.

Biochemical mechanism: Pyruvate kinase deficiency usually will manifest


clinical symptoms on red blood cells (RBCs) with no apparent metabolic
abnormalities in other cells. Insufficient adenosine triphosphate (ATP) is
produced in the red cell and its membrane is affected, is rigid and
removed by the spleen.
Inheritance: Autosomal recessive.

Caso 6
Summary: A 3-year-old boy with failure to thrive and possible hepatic failure. He
presents with hypoglycemia and recurrent episodes of nausea and vomiting after
ingestion of foods high in fructose.

Diagnosis: Fructose intolerance.


Biochemical basis of disorder: Because of a genetic disorder, the hepatic
aldolase B enzyme is defective, and functions normally in glycolysis but
not in fructose metabolism. Glucose production is inhibited by elevated
fructose 1-phosphate. When fructose is ingested, severe hypoglycemia
results.
Treatment: Avoid dietary fructose.

Caso 7
Summary: A 48-year-old male with strong family history of heart disease and now
angina and exertional dyspnea presents with a significantly elevated cholesterol
level. A medication is prescribed that is directed at the rate-limiting step of
cholesterol biosynthesis.

Rate-limiting step: The enzyme hydroxymethylglutaryl-CoA reductase


(HMG-CoA reductase) catalyzes an early rate-limiting step in cholesterol
biosynthesis.
Likely medication: HMG-CoA reductase inhibitor, otherwise known as
statin medications.

Caso 8
Summary: A 1-year-old girl born outside the United States with developmental
delays, hypotonia, hypopigmentation, and foul smelling urine.

Likely Diagnosis: Phenylketonuria (PKU).


Biochemical basis of hypopigmentation: Phenylalanine is competitive
inhibitor of tyrosinase (key enzyme in melanin synthesis).

170
Brigitta Cismasiu | FMUL 2010/2011

Caso 9
Summary: A 38-year-old vegetarian female with gradually worsening fatigue,
neurologic and GI symptoms, and megaloblastic anemia.

Diagnosis: Cobalamin (vitamin B12) deficiency.


Underlying problem: Lack of cobalamin intake with complete vegetarian
diet (vegan).
Causes of megaloblastic anemia: Folate and cobalamin deficiency.
Patients with folate deficiency have similar hematologic and GI findings
but do not have the neurologic symptoms as with cobalamin deficiency.

Caso 10
Summary: A 47-year-old male presents with an acute onset of right toe pain in the
middle of night after drinking alcohol and no history of trauma or any other joint
pain.

Diagnosis: Gouty arthritis.


Confirming diagnosis: Demonstration of the presence of the monosodium
urate crystals within the synovial leukocytes or in material derived from
tophi under polarizing microscopy.
Pathophysiology: Increased conversion of purine bases to uric acid or a
decreased excretion of uric acid by the kidney. Elevated levels of the
insoluble uric acid result in precipitation of urate crystals in the joints.

171

Você também pode gostar